You are on page 1of 343

v

Muc luc

.. .

Li1i n6i diu

C9ng tic vien

xi

1 Bit ding thuc ce So

1.1 BAl dAng thli'c AM-GM

- , - .. ,

3 4 4

13 18 HI 27 34 34 39 44 44 49 53 53 59 61 62 66

1. Ll Bat diug tl!d\: AM-GM vii. ll'ug dung .

1.1.2 Ki thua,t Cosi ngJ.t<;'c dAu .

1.2 Bi\t dling thl1'c CauchY-Schwfll'Z-Holder .

1.2.1 Bat. ding tlnk Cauchy-Schwarz WI IInP; dung 2.2.2 BAt dAng thU'c Holder . . . . . . . . . .

1.3 Bat dang thac Chebyshev . . . . . . . . . . . . 1.3.1 BAt di.ng th1k Chebyshev va lhlg dung . 1.3.2 Kr thutit phan tach Chebyshev ..

1.4 Bit ding thuc v<'li ham 161 . . . . . . . . .

1 04.1 Ham !ai vdi bat din"g thuc Jensen

1.4.2 Ham 16i vi1i kl thu~t xet pnful ttt a bien 1.5 Khai trien Abel va bilt ding thuc hoan vi

1.5.1 Khai trieD Abel . . . . .

L 5.2 BAt ding thuc hoan vi .

1.6 BAt ding thill; doi xlffig 3 bien

lfl.I Bit ding thuc thuan nMt khong co di~u ki,fn _.

1.6.1 Bat dAng thuc d61 xtmg co dj~ll kien .

'21'2

II

1.7 Bat. dAng thu:c V.3. cac da th(it: d5i xttn1j S0 ci'iv .

7l

1.7 1 I.i thuyet v€- Cf\( da th(l'l: etoi xirng ~J (;~P 71

1.7.:.2 Un t.hrrc d(1i Xl.'rng "'0 dip vA. cAl' ,'(n~ dnll!!; I rung (!;"LAi toan hAt

rlAn).', i 11 ,~c 72

1.8 Phuong phap can b&ng h¢ b6

74

LR.l Bai toim rHQ rlA u . 74

l,~ 2 Can b5.ll!l'; h<: s6 vdi bilt dang thtk lien hf' I rung btnh c¢ug V;I

tT{ln~ bmh ullin {AM . G~l} ". 75

U-:.3 Can hhllg h(~ ~O vdi bat dang thlfl' Cauchv - Schwarz - Holder tl-O

1.9 Dao ham Vel. l!ng dung . 1.9.l Kitn thuc Ii thuyct

1.9.2 Khao sat ha.m :;6 mot hi.en . 1.9.3 Khao sat ham nhicu lJi(;n .

1.9.4 :'>,'h) r oug lIlQI ba~ thi toan qu6c t6 2004

83 84 8G

87

L 10 Bai t:;ip ap chl!l~

1.11 \lQt so hal to"; n dang chu Y

2 2 Ban \'(~ s .. ing. t~.I() bil!. (l(uL~ thlk 2.2.1 BItt ding tlurc ell vi, mal

2.2.2 ~·1¢t. each xay dung bt~t. dhng tht"tt: lTI(ji 2.2.3 Til dlltng ruinh - phan bien d(.l1 ket lufiu . 2 2.4 Simg tao hit di\l!g t lure .

105 Ill.') ~Ol 2Dl 20:) 2nli 21lS

2 Sang tao bat dAng thac 2.1 Car hiti toan chon 19C

3 Cac phu'dng philp clnrng rninh bbt diJ:ng thuc 3.1 Phuong phap JOI) bit}Il va dlnh Ii GO]! bi~1I manh

211 ZlZ

:U.2 Phuong pI-I";]) dOll hi~n.J:ol vai (.:;ic hAt ding~lhtk ~ bl~I! 2Hi

3.1.:1 Dinh 11 dAIi bi&1l rnanh S.M V . . . 222

3 1.-4 Dinh II S.}.1. V va mot ,,(i 'lmg dl_\ng 224

3.1.!J Phirong phap eli:})] bi0I1 toan mif~1l . 230

J'vfl.1C h,lc

III

3- 2 Phuong phap ph fin tlch binh phimng S_0.5

233

3,2_1 Bai toan mJ ua u ~:i:i

:~,2.2 Djnh If vt, bi~ll ditin co »cr n'lit pillt(tng phAp ~ O_S va mnt stJ

ki thuat phan t.ich , , , , , . , , , , , .... , .. _. , 239

3-2 3 Nhili!g lrug dung quail trong cua plurong phap S_O,S _ 244

3.2,4 Suy luan tU mQt bai toan 249

:U Phuong phap phan clnrng

254

.3 3_1 Bai toan nur d~ 1I , 254

3.3_2 :--;-hlll nh;)l1 11lOt ua.t uiLJ~ thul,; ul1Ji p;ut: U\} phan clurug 2~;)

3,3.3 Cac bai toan ap dung

34 Phltdng phf;.p quy nap t6ng quat 3-4- 1 Bai to all lIla .Iau ' , , , ,

3.4,2 Phuong phap quy nap t6ng quat va dinh Ii LG.l 3,4,3 Cac bai toan ap dung - - - - - - - - - .

3,4-4 Xay dl,tng ham so va bai toan tong quilt

3-,) Phuong philp f).U' I,h,_mg oat Jiing thl'il; (;13 di0u .

3_6 Nhin lai 5 plnrong phap chirng minh bat db,ng th(tl; 3-6 1 Phuong phap don bien , , , , , .

3_6_2 Phuong phap phan tlch bmh plnrocg S O-S 3-6,3 Phuong phap phan chimg - - - -

3.6.4 Phuong phap quy nap tang quat

3.6,5 Phuong phap sit dung bAt d~ng thuc co di~n

257

262 262

267 276 277

285 287 288 289 290

4 M9t so van de chon IQc v€- bij_t dAng thuc 4-1 Bat u~ng thirc Schur suy rong .

4_2 Nlnrng hat dang tlnre kl 1<;1. T

4,3 BAt ding th(k Nesbitt va mot s6 dang rna TQng 4.4 Suy luan va ph at tritn , . . ,

4,5 Bat Ilil-ng thirc thu~n nghich ,

4_ 5 _1 Ph uong phap tlch phan dOl vdi bat rlilng t.hlit; -

4- J -2 Bat d~lIg thlrC thuan nghich .

291 291 291:$ 303 308 312 312 314

33:i 340 340 341 342 343 344 344 346 346 347 347 348

IV

4.6 Di Um lill giru ~tJ c~p . . . . . . . . 4.6.1 Tro lai van de cO dien , .. 4.6.2 Them m¢t bal tOM 4 bii!n ,

4.7 Ly thuyet cac bO trOi va bat dAng thl'tC Karamats , 4.7, I Cac bi) tro-i va mOt s6 tinh chat lien quan 4,7.2 Blit dAng thuc Karamata

4.8 D&n bif:n kh6ng xac dinh .. ,.

4.8.1 0. ~ b hay a S b .

4.8,2 Don bien khflng xiu; dinh

4,9 Bat dimg that va. de yin de: rna 4.9.1 MQt l(1i gi;li hoan chinh? .

4,9.2 ~Ot tiit u~ug tb& a61 xung thu vi . 4.9.3 (lib + bit: + cia ~ 3 ?

4.Y,4 Bat u&ug tli(it: huall vi t01lg quat 4.9,5 Chi La die hAL dAng ihirc b(lc nhit .~ 4.9,6 Cae dang t6ng bmh phinmg .

4.10 Tan man vrii bat tlhg thitc ' . 4.10,1 Cac (~~P thuan nghich , 4.10.2 Sang to,w b§_t ding thuc

4.l0.3 QUaD di€rn ve mOt bai tOM ba.t ding thuc hay 4.10.4 HQc toan trcn mang . . . , . . . . . , . . , .

317 317 320 320 320 .324 3:i:i 333

Phu luc

Tac gia cac bai toan Tal H~u tham khao .

349 349 350

3

Chtrong 1

.... ?

Bat dang t hrrc Cd sa

Df lam qm:n v{ji bih. GAng t h{(c th] vi~c: nam vihl~ d.c bAt. dim!,,; tlurc co ban If> vo cung quan trong. Tren the gidi co r ~fit nhieu cac ba.t dcing thirc, d.t nhi~ll nhirng dinh If Ii~n quail Wi Mt diutg Hate, dit nhi~u cac ki thni!.t 11116 chimg minh b~t dAng tlurc Ilt'll J~ Lit·t ht-t dtfQl: chung la uifu khong th~. dihl quan troug Ilhat HI clning t a phai hi&u t.hM ro c(u: bAt dang thtrc co ban, d6 cung I':" yCu to quan trong dau tien df ban hoc tot bat dA.ng thirc, Tae gia se nh.aIl rnanh Wi nrnrng hat d~llg thirc h~t 5([C cAn thie-t sau day; Bat dilllg thuc lien he gilla trung blnh cQngo..trung binh nhan {AM· GM), bAt rl~llg tluk Cauchy - Schwarz va t6ng quat III bitt. rtiinf.'; tllln~

Hoi der. Mt ding t ht't'c Chebyshe.v va biit dAng thuc J e.nM:n. '

Day Iii. 4, b~t dAng thtYc quen thuoc trong I;hl!dng trlnh pho thong, nh,tng dJ Him vung duoc chung t;tlng khong phai H~ clifll don gian, d~ dang nhat, la doi vdi cac han 11loi Ldt l.l<iu 1~1I)j quon vai Lit <!fmg, rlnre. Clurong t.1tm ti0n cua cuon ",{u;h cung {;Zt.p cho cac ban kha dfiy au nhullg ki nang su dung 4 b5.t ding thirc do, them rnra la mot 56 bai toan lien quan trii cac bfit dAng thii'c d6i xung 3 bien, bat dimg thu-c hoan vi, plurong phap can bA.ng h¢ ::;6 va kI thuat khai tri01l Abel. Day la chwl)g ell ban va d.t. !Ji~n ftw\ rH'ii voi ('1l.C: b{L!l bor- sinh THCS, ~:;k ban hQr sinh lap 10, 11 muon ren Iuyen ki nfulg cmrng minh bit tHing thirc eua minh.

D6i veii moi b~t ding thtfl; co ban do, tac gia. d~u chon mot ki thuat ap dung u~c bi~t nhAt .:Ie ban doc tlliiy ro hil?u qua. cua cluing. Ch~ug h;;UJ v~i blit uimg that:

AM - GM ta co ki thu~t COS1 n.qJtqc, vol b&t dAng tlnrc Chebyshev ta c6 ki thuat phan tach Chebyshev va voi bAt ding tM'I.: Cauchy - Schwarz ta c6 Mt ddng th-itc Holder. Thea eAch nghi 61R hoc sinh Vii,lt N am th\ blit drmg thrre Holder c6 vc hoi xa 10. vn kho {tp dung, ngav.ca. voi cac ban hoc sinh gioi toan, nhirng C\li:m sach sf! giup cac ban eo mot each nhin khae hem doi vdi bat dimg t.hm: quan trong nay, binh thl.1bng nhu chmh bAt ding thu:c Cauchy - Schwarz v~y. Phan SU UlJIlg dao ham ding lh. phan If thuyet quaIl trong rna cac ban d.n phai narn roo

s ala2···a", .. ,S

4 S + -- > n n ___:.._.:;____:.::........::-

n-I - n-l

=} s ~ (n - l) n-,yii"ia .... a,,_I_

Til' 2 nhan xet trell ta co ui~u phai ehirng minh. Dling tlnrc xay ra khi va chi khi tAt ca cac bien a, . (12, ...• a" bang nhau 0

4 ChlfdJlg 1. Bat d<1ng thuc co sO

1.1 nth d~ng thuc AM-GM

1.1.1 Hilt d~ng thirc AM-GM va img dung

Dinh I)' 1.1 (BAt dAng thU'c AM-GM). Vai moi so th'\tc dUCJng al. a2 •.... an ta co Ml rld1l9 that

0.1 + a2::+-· .. + at. _~f'-----~

---- > vU1U2··-a,..

II

Ddng thuc xriy m khl 110. (hi khi (q = n·~ 0:::: .. - = (In·

CHGN G )'11N1-L RU raug bAt diiug tlilk voi 11 = 2. lli'·U bn.t ding tlnrc dun!!; veri n sb thl ciing dung voi 2n so VI

Do do bat d!ng thuc cling dung khi ti bAng mot !uy thira cC!a 2. Mat khac nell bA.t dAng thac Mmg 'len n s6 thl cflfig dung vdi n - 1 ~o. that V?y ta chi call chon

(I." -= s/{n - 1) , s = al + a2 + ... + «,,-l

BAtd~n!'; t.lrll'l: AM - OM \1':1. bAt. d~n).'; tllt1:e qncn thll¢r va ro l'mp; dung n)rtJ.'; riii, lb. bli.t ding th(ic da.u tien rna. cac ban don phil ghi oM' rAt rc va su dung mot each thanh thao, Khi !)u dung b~t dfutg thuc nay chil~ ta can het ",L'rC chu y tdi dieu kien ella dAng thirc khi al = a2 === ... = a,~ va cAn tach cac h¢ s6 cho phil hop.

Co nhieu CA.ch durnp; mirth bat (limp; t hal: AM - G M. each clnrng minh hay nhit c6 thB la each chirng rninh sl1 dung phirong phap quy nap C(l.1u:hy (nblt clnrng minh tren). C6 Ie vi v~y mil. nhieu nguol Iiim Ian ri1ng Caur:hy phat hien ra h~t dang

thirc na.y. Ong chi ia ngti~i' dua ra chirng minh rat hay eua rntnh chu khong ph,i.i la ngtrai phat hien fa d!.u tien, hAt ding thuc Ilia. chung ta quen gQi la bilt ding thU'e Bunhiacopxki thuc ch~t la phat minh cua 3 nha toan hoc Schwarz, BunhiaCopxki va Cauchy- Thco each goi tf~n dump; cila. the J.!;it'ti. bht. dfi.n~ t.~l({'; BunhiaCopxki cO t1!n la biit dang thirc Cauchy - Schwarz, con bat cl~!lg thuc C6si (hay Cauchy) co ten 1& bAt ding tlurc AM - GM (Arithmetic Means - Geometric Means). Day la m¢t s\,t nharn IAn kha kl I~ va dang ngElc nhien trong mot thui gian dai 1?

Sa.u day Ie. mot so bai to an d~c trung sir dung bat dAng thuc AM - CM.

1,1, BAt diing thl'tC AM-GM

5

VI du 1.1.1. Ch{tng mini! ranq vai mvi $6 thw khang am a, b, c fa c6

1 1 1 9

~+-+-> -

a b c-a+b+c

LOI GIAl. Stl (h,m!!; bAt nAng t}lll't: AM - GM cho 3 s&

(1 1 1) y-.... 3

(a + b + c) - + -b + - ~ 3 v abc 3r:-c: = 9,

a C vabc .

s,h dAng thl1c tang quat hen diroc chimg ~inh hoan toan tursng tlJ

1 1 I n2

- + - + ,.- 1- - > , 0

0,. a2 a" - UI + a2 + _ .. + an

Vi du 1.1.2 (B!it ding thuc Nesbitt). ChUng mink ding vdi m9i so lh'ljc !chong am a, b, c ta co

abc 3

-- + ~-+-- >-,

b-lc c+a a+b~2

LCH GIAL X6t cac bi~u tbik sau

abc s=--+--+--

b+c c+a. a+b'

b c a

M = -- +--+-_, b+c c+a a+b

cab N:::--+--+--

b+c c+a a+b

Ta co M + N = 3. tI_·lM khac thee hilt dAng thuc AM - ClI-I thl

M S a+b b+c c+a 3

+ =--.+-~+--> < b w c c+a a+b - .

N S a+c a+b b+c 3 + =--+--+--> .

b+c c+a a+b-

V~y M + IV + 25 ~ 3 suy fa 25 ? 3_ DAy lh dicll phai dutn!l; rninh. Q

vr du 1.1.3 (BAt ding thli'c Nesbitt 4 -bil!n). ChUng minh rdnq l.uji mpi so thl,lc khong am a, b, r' ta en Mt diin!J thuc

abc d

-- + -- + -- + -- > 2, b+c c+d d+a a+b-

6

CJmong 1. BAt dAng thll'c co "a

Lei CIAL Wit

a b t. d

s=--+--+-- +--, b+c c+d d+a u+b

bed a

M=--+--+--+-b+c c+d d+a. a+b'

, c dub

J.v =--+--+--+--. b+c c+d" d+a a+b

Ta co M + N = 4. Theo hat dang thde AM - GM thi

M S a+b b+c c+d d+a 4

+ =;=--+--+--+-->

b+c c+d d+a a+b - ,

N .~ S = a -I- C + b -t d + a + c + b + d

b+c c+d d +-a n+o

a+c a+c &+d b v d

= --+ ~--+ _. -+-b+c a+dr.+d u+b

4(a + c) 4(b + d)

> + =4.

- a + b + c + d a + b +«; + d

V~y M + N + 2S '2'_ 8 suy ra S ;:: 2. DAng thuc xay ra khi a = b :::;: c = d. 0

Vi du 1.1.4. Cia SU-Ul, a2,'" a,~ Ill; cdc so th'!jC d1fan.Q sao cho al +U2+,.- +a,,:::;: n. Ch'l1:ng ttunh. vOi mo?-i s6 nguyin diJdn_Q k la c6 btlt dang tMc

LOI G IAI. Sit dung bit tling tlnrc AM - G M ta c6

(k - 1 )ak + 1 = ak + r;/' + ... + ak + 1 ;::: k V flier k~ 1) = kale - ~ .

Thay a hoi UI, a:h ... , an r5i tong cac blh d~ng thuc lai ta ,11.(1)<.'

V~y ta chi d.n chthlg minh

SU clung btu ding th\te AM - OM

ak-1 + (k _ 2) =' ak~l + 1 + 1 + .... + l ? (k - 1) "-,y;:;:k-l = (k - l)a.

Thaya bal ai, a2, ,." an rblc(mg ~a!: bAt ~it.llg that: dang tr(~1L lai

a~-l + a~-l + + a~-l + n(k - 2) 2: (k·- l)(al + a2 + ,.' + an) = (k - l)n

1.1. BAt dAllg thttc A.M-GM

7

,1,-1 1.-1. 1;-1

~al +aZ -.-,.,+a,~ ~n,

D~ng thuc xey ra khi Va. chi khi al = a2 ,.. '" = an = l. U

Nhan xet. Til cac chimg minh tren ta suy ra

Voi moi sO nguyen dircng k: Ngoai ra ca 2 bat ding thtrc nay deu dung khi k 2:: I Iii mot so thirc. Neu xct k < 1 ta co bat dAng thuc

m/a! + a~~~ + an ;::- va! + \iii2 + '" + ~,

V n n

Vdi rnoi so thirc dimng m 2: 1. Cat; bau hay tu kiOm ltghl(1ll cac bilt Jaug th(rl: nay hAng phutrng phap CJ trcn, t lnrc chfi-t DO chi lit h¢ qua trw ticp_

Vi d~ 1.1.5 {Ba.t ding thuc AM-GM suy ri?ng). Wi cdc so lhr,tc dU(1nq ai, (J2 ••• " au va Xl, X2, "., X)'t lii cae so Ih1jC kh6ng am co. !!}ng Mng 1 ta r:6

LOl GI.~1. Phuong phap chimg minh fill' dung quy nap Cauchy hoan to an tuong ur nhir voi bal dA!lg t.huc AAf - C.M thong thirong. Tuy nhicn trong trirong hop n ;;;;;; 2 chung t.a tAn mot loi giiiL chi titt hen, Ta phai chimg minh ntu x + ,t/ = 1 va 11. b, J', ,t/ lii cac so tlujc khong am rhl

ax + by ;::: aJ;b~.

eAch lam nan giim Ilh:\t dl)] VO] bAt dAng tlnk nay 1i'!. x{·t v6i s& 111111 ti rili dmyrn , qua gioi han. Hifn nhicn !leu ~,y huu ti thi bai toan dl,l'Qc chung minh thea blit dilng tl!ttc AM - GM c6 (lH~n

Trang do x = m/(m I n) va y = n/{m + n). Con Ili\u x, y tlU1C thl sc ton t ai day cac s6 hiru ti Tn -+ X, .!itt ---> y, T"n + Sn :: 1 va nhir v~y

Chuycn qua gidi han khi n --\ +00 ta duqc ax + by ~ a"'b" , Day chmh b. di~u phai chirng rninh I:l

each chirng minh nay kha co ban, va n~u han chua dl1Qt: hoc v~ gioi han lhl hay t~lII1 chilp uh.('l,ll bat uaug tlntc vdi !;a\: :,;0 Xi h..--(\I tl. Li do rat Ul.lH giau, vt M dinh nghia chinh xac clio luy tillta vci sO mil thue, buoc phai ce dinh nglila giCii han. Cac kien thuc co sO ve gi6i han va ham lien rue sf: dime dinh nghia trong chimng trmh toan ph6 thong lap l l , 12.

8

C1Hlcrng 1. Ba t dAng tht!'c ca ~a

Vi du 1.1.6 (IMO Shortlist 1998). VOi x, '!I, z Iii cac 56 thuc dur:fnQ ('0 tiel! IIdng 1, r.;hltny minh Mt dang th&c S(JU

x3 y3 z3 3

-,---------c--_,,.. + + > -

(l+y)(l+z) (l+z)(l+x) O+x)(l+y) - 4'

LOI G1.&.1. Su dung bAt ding t.hirc AM - GM cho 3 s6

xS 1 + y 1 + z 3x

(1 + y)(l + t) + -8- + -8- ~ 4'

Tirong til ta c6 2 h:h dang thuc VOl y, Z r5i cong !/ili suy ra

x;) y3 z.l X + '!I + z 3

-----+ + > ' --,

(l+y}(l+z) O+z)(l+x) (l+x)(l+y) - 2 4

!\-1~t khac x + 1,1 + z ? 3?fXfiZ = 3 nen ta c6 dpcm: 0

Vi du L 1. 7 (IMO Shortlist 1990). Cia s'£ a, b, e, d iii cae 56 thllc khlJng am thod man ab + be + cd + cia. = 1, ChUng minh

a3 bj cJ dJ 1

~--+ + + >-,

b+c+d c+d+a a+b+d a+b+c ~ 3

Lor GIAl, Sit dung bit ding thuc AM - GM cho 4 s6

a3 b + c + d a 1 2a

b+c+d+ 18 +"6+122!-.f'

Hoan roan t.\fdng tu ta co them 3 bat Ml,lIg th.k vui b. c. d sail 06 cong I~i dltQ'1:

a" bJ c~ d3 a + b + c + d 1

---+ ----+ + > ~-

b+c+d c+d+a a+b+d a+b+c - 3 3

Chu y r~ng ab + be + cd + da = (a + c)(b + d) nell

{a + b + c + d)2 ? 4(ab + be + cd + da} ;;;;;; 4 =:. a + b + e + d ;:- 2.

Thay krt qua nay vao M,t ding thuo (j tren ta co dpc m. D~ng thtre xay ra khi va chi khi a ;;;;;; b = (.: = d = 1/2 0

Vi" d IJ 1.1.8 (Komal Magazine). Ch1ing minh 1Jm m9i a, b, c dUUn.q

1 1 1 27

+ + > 2

a{a-f·b) b(b+c) c(c+a)· 2(a+b+c)

1,1_ BM rfJIlg rh,k At"I-G.Il1

9

Lor CIAI, SiI dung trm: tifp hat dang thuc AM - OM cho vt: trai

I 1 1 3

----,- + + > ----;;r=:===;=""""'~=:::::;::;:==7

a(a + b) b(b + c} c(c + a) - {labe(a + b)(b + c) (c + a)

Do do ta chi r;'\n chirng minh

Thee b§.t dang thuc AM - GM

3Jubc::; {a + b + c)\

33(a + b}(b + cHe + a) ~ (a + b + IJ + c -I c -I- af' ;;; 8(a + b + C)3, NM.n thea vc 2 Lat llftJl!!, t lurc tren ta l!ltv!.: Ji(,u phM darn!!; minh. 0

Vi du 1.1.9. Cho cdc ,~6 th1jc a, b. c thoa man a2 + bZ + (;2 = 3. CMtnq mmh Mt ddng thuc SU11.

Ldl CIAI Ta cung ap dung true ti6p bit ding thuc AM - GM nhir sau

(Ial + [bl -I kl)2::; 3(0.2 + b2 + ['2) = 9 => lal + [bl + lei:::; 3.

Ceng vc 2 bAt ding t lnrc tr~t1 t.a c6 !lieu phai cmmg minh, Ding tlnrc xay ra khi v a chi khi trong 3 so a, b. c c6 2 so bang 1 va 1 s6 bang -1_ 0

Nhan xet. Ban doc thu lam bai to an tren n~u ta bO di cac dau gia tri tuyet dbi, tim max CUR a + b + c - abc, Day 18. mnt bai toan rAt thU V! va khong de,

Vi' du 1.1.10 (Iran MO 199B). Cho cac s6 th1jc dUr:fn.Q a, b, c, d thori man abed = 1, Chitn,q minh rang

., -, "" ( 1 I 1 1 )

a" + b" + c" + d'-' ::::: max a + b + t' + d, ;:; + b + ~ + d .

LaI CIA!. Ta phai clurng minh 2 bat ding t htrc sau

(1)

a3 + b3 + c:> + dJ ~ abc + bed + cdr. + dab (2)

10

Clllwllg 1. 8lit d~I!g t hue CO .~a

Bat dang thirc (L) duoc chirng minh tuung tlj nhu trong vi du 1.1.4. BAt d&ng thll'l: (2) 00 lhxQ'1,; balig t6lJ& 1:8.1.: oilt U~Il~ tinA: sau

a3 + I"i + r3 ;:- 3abc bJ + (3 t rP ~ 3bcd

e"3 I- rl~ + (1:'> ;::: 3eda Chi co da.ng t.hirc khi a = b = c = d "" 1. 0

Vi du 1.1.11 (USA MO 1998). Chung minl: !.1m mQi ;;6 tJu,tc dltong u, b. r:

I 1 1 1

. + -.-'--"_ + <-.

o;J + b3 + abc b'~ + t3 + abc c3 + 03 + abc - abc

Lot CIA!. Ta co nhsn xet sal!

a? + /;2 :2: 2ab =? a3 I bJ ~ ab(a + bJ

a~ a~ c

::::} as + b3 + c'! :::: ah(a + b) + abc = a + b + c

Xay dung them 2 bat d~l\g thUl.: tuong W rCJi c(mg, Il,1.i suy ra ui('u phiii. chung minh

abc abc abc

_____ + _. - + -_. ;_.- < l.

a3 + OJ + abc b~ + c:> + «be ("'1 + a3 + abc _

Dn.IlB rlnre xay f11 chi khi a = b = c. 0

Vi du 1.1.12 (France Pre - MO 2005). Cdc so ihuc dwng :£, y, z thad uuin difu kifn x2 + y"2 + Z2 = 3. Hay chtmg mink

xy + yz + :.: > 3.

z ;r "11-

Lor G IAI. Bmh plnrong 2 ve cua hat diing tlnrc, ta phai chung minh

x2y2 y2z2 Z2X2

-~- + -' -2- + -2- + 2(x2 + l-+ z'~) ~ 3(xZ + y'2 + z"2).

Z X Y

Chu y riing thea bAt ding thirc AM _ eM ta co

( x"2y"2 112 z2 Z'l.:r2) (X2y. 1;'1 Z2 ) (y2 z2 z2 <1;2) (x2 y"2 ;;2X2 )

2 --+--+-- = --+-- + --.'_ -- + --+--

z"2 X2"!,p ;;:2 x2 x2 y. z"2 y2

~ 2(x2 + y2 + z"2).

DAng thul.: xay ra chi khi x ;:;; y = z == 1. 0

N~ xet. Cac ban hay ki~m nghiem blt dAng tJUJ{; sau

... J 3 3 ab be ca

Neu a, b, c dudn.9 thorl m(in a + b + c = 3 thl . - + - + I. ~ 3?

c a. u

VAIL de. t6ng quit se diroc giai quyet (I chuong san ciia cuon sach.

1,1. BAt d.trJg thtk AM-G.H

11

Vi du 1.1.13 (IMO Shortlist 1996). Cae so d1.tdn_q x, y. z co tich bdng L Chiing minh bat dang thuc

xy yz. zs:

5 '+5 0+" ,:5l.

x + xy + 1/' Jj +!V + z" z;J + ZI + x"

La! CIA!. Ta co nhan xct sau

---,-_ = -~-_

1 + .1;)I(X + y) x + y + z

Xay dimg 2 b[l.t ding t.hul: tl1ang; nr voi X,)I roi cong vf. ea 3 b~t ttrHlg; thut; t a C[' ui~u phal chtmg minh. Dang thirc xiiy HI chl khi .r = y = z = 1. 0

Vi du 1.1.14 (Viotit Nam MO). Cd,!' sJ thuc dUlfnq x:. Xz, ... , If> tho« man

I 1 1

--+-~+ ... + --",,1-

1 + Xl 1 + X2 1 + Xu

Hay chun.g mink ri1.ng

Ldl GIAI. Tu gia thl~t bai toan ta c6

1 1 1 £n

-- + -- +,.- + --- = _-

1 + .71 1 + 72 1 + XII I 1 + ;r"

Xn n-1

--> '

1 + Xn - "{Ifi + xd(l + x2) ... (1 + xu-d

Ta c6 n - 1 b.it da.ng thuc tn'dug til vai moi 56 XI, X2, ... , XII-I, sau do nhan cac v~ wang (tng eua n IJAt ,Jill!!; tlnn: Ir{On !.';'Ii suy rot rlpr;m, 0

Vi du 1.1.15 (APMO 1998). Chttn,q mink voi moi z, Y: z dJJdng ta co

(1 + ~) (1 + ?t.) (1 + ~) "2: 2 -+ 2(x.+ J) + z).

y z. x ?fiYi

Lm G IAI. D~ dang nhan thfi.y bAt dAug thuc tren 1~ hR qua tit b!i.t dAng "hue sau

:r y z x+y+z

-+-+-> .

~ z x - ~

UJl CIAI, 51r dung bat dling thrrc AM - GM eho 3 sa

x3 y3

- + y + z ~ 3:r , - + x + z 2 3y ,

yz xz

J Z

_. + x +y;::': 3z. xy

12

Clnlollg 1, B~t ct~)jg thrl'c co .~'o

Ta nhom va SU' dung hiit d!ng thirc AM - OM

(X 11 z) (2X Y) (2Y z) (2Z x) 3x 3y 3z

3 -+=-+- - -+:.... + ....:...+- + -+- >~~+--'-+--

Y z ,r Y Z Z T :r; y - {/XfiZ ?fXiiZ <fXYZ



x y z ~+y+z

=>-+:_+-> ,

Y z x - ?fX)iZ

Dimg t hirc xay ra khi va chi khi a: =; y = z. a

Vi du 1.1.16 (Canada MO 2002). Vai tnri x, y, z; d1(dng, hay ch-ti'n.Q minh

x3 1/3 z3

-+:""_+-2"x+y+z

1/z ;rz Xll

Cong v~ 3 ]J;_\t thing t.ln:tr tlt:u ta j_;U tlit-u phai clnrug ruinh. 0

Vi du 1.1.17 (Macedonia MO 2000)" Chunfj minh. VOl mQi x, y, z dttctnq .1:2 + y2 +Z2 ~ 12(X)) + xz)

Lor GIAI. SIt dung hia ding thuc AM - GM vdi 2 so

.., (y+zf M

x2 + y2 + z2 ~ ;r:< + . 2 ? v 2x(y + z) ,:::1

Vi dl,l 1.1.18. ChUng minh voi moi a, b, c, d d1ldng t(l luon co

l6(o.bc + bed + cda + dab) ~ (a + b + c + d)4,

LI)I CJAI. Sir (h_In."'; bM rthng 1.lnn" AM - GM cho 2 s6

16((l!Jc +- bed 1- cda + dab) :;;;:: 16ab(c + d) + 16cd(a + b) -::: 4(fl + bfJ(c+ d) + 4(c + d)2{a + b) ~ 4(a+ b+ c+aj(a + b){c ~. d)

~ (a + b + c + d):).

D~ng thuc xay fa khi va chi khi a = b ~ c = d. 0

Vi du 1.1.19. Chltng mink riing vOi cdc so dUdng u, b, c co t6n,q Mng 3 thl

a{a+c-2b) b(o+a~2c) c{c+b-Za) 0

-'-----'- + + > .

ab + 1 be·1 1 ca + 1 -

1.1. BtH dAng thtl'c AM-GM

13

Lot GIAI. BAt dAng t huc hMng drnrng vui

a(1 - b) + 1 + o{l - c) + 1+ c(l - a) + I > 3

ab -+ 1 be + 1 t.tl -+ 1 -

u+l b+l l:+1

<=} -- + -- + -- > 3.

a/) + 1 oe + 1 ca + I -

Sir dung hat dAng thuc A/~J - GM cho 3 sO hang CI v~ trai

VT> 3 ~ (a + 1)(0 + 1He + 1) .

- (ab + 1)( be + 1)( ca + r)

Ta sc cmrng minh (a + l}(b + 1)(e + I) ~ (ab + 1 )(bc + 1 )(ca + 1). That vl).y, bat dang tlnrc trtlu urong dnong voi

abc + ab + be + ca + a + 0 + C + 3 "2: a2b2c2 + abc(a + b + c) + flO + be + ca -+ 1 <-.} 3 ~ a2b2c2 -+- ::Iabe.

Bat d~ng tlnrc rren hj~n nhien d'I"ing VI theo b6.t d~ng thrrc AM - GM thl abc s: 1. Diug tll1~C xay r a khi va chi khi a = b = {: = 1. 0

Nhu da noi, chu y quan trong nhat khi su dung hat dang thuc AM - GAl la phiii chon dlmp, M s.A khi !l;hcp rli.p d~ dlin!/; tlnrc co t.h~ xay HI a~iQc, ChAnp, han, d

vi dl,l 1. 1.6 ta kh6ng th~ su dung biit ding thuc .

3 X

)( 'I + (lJ -+ 1) + (z + 1) ~ 3x,

(l+y 1+;;:1

Theo cam gibe dAng th(k x~iy ra khi -T = !/ = z = 1 n€n ta chon dUQc h~ ~ 1/8 d~ cac sa hang bang nhau

xJ 11 + 1 z + 1 3x

~--:--:"----:- + -- + -- > -,

(1 + y){l + ,0) 8 8 - 4

Vai cac bai to an d dang chuan nmr tren, tile It!. co dang thuc khi tat ca cue bien b~ng nhau thi viec ghep cap nhir vay tuong d6i de, nhung voi mot 56 bai toan b~t d&ng t huc kh6ng dOl ximg t hl cbng vi~c nay !It> kho khan hun, t~ phai d \lng ph~(lJJlg phap Cdn bdng hf 56 va phdi gidi ccc phudng trinh. Ban co th6 xcm trong phfin sau, bai viet vc Phudng pMp ciin. bhng h.(! .~J, tr ang 75,

1,,1.2 KI thuat Cosi ngiroc d§.u

Bay gib chung ta sc xcru xct b~t ding tlnrc AM - GM va ruot ki that d~c bictkl thuat Cosi l1gWC dau, Day Iii. mQt trong nhirng ki thuat hay, kneo leo, moi me va An tuong urdu etta bAt d?ng thlfc AM - G M: Hay XCIII cac VI du cu th~ sau

14

Vi du 1.1.20. Cae s6 dltdng a, Ii, (' tluxi man dieu kit!n (l + b + c = 3, ChUng minh MI dlin!) th{tr;

(l IJ C 3

--+--+~->-,

1 + b. I + c2 1 + ,,2 - 2

LOI GU\.L Ta khcng thf dung true ti~p bAt uAng thtrc AM - GM vdi mall ~t:~ vi bat dang thire sau rI6 :-it' d61 chien

a b c abc 3

-- + -~ + -- < - + - + - > - .,~

1 -r b2 1 + c. 1 + a2: 2b 2c 2(1 - 2

Tuy nhicn, nit may ttuin. ta co the dung I{li bat. dang date do thea each kha«

. a ab2 abi nb

-- ""'0;- -~ > a- - ;;;;;;0;--.

I + b·J 1 + b2 - 2b 2

Ta clii ~.I dung b~t, dAng tln'!e AM - GM cbo 2 sA 1 + b2 ~ 2b rj duoi nihil nhirng lai 1:0 1.\l10c mot bAt dtmg thirc t.hu~n chieu? S~( may mdn d day la nu)t each dung nguoc IAa d~ng thirc AM - GM, mot ki thuat d.t ~n nrcng va bAt ngo, Ncu kh6ng :ill dung phinrng phap nay thl btLt dAng that.: trsn se rat kho "it dai,

Tit bAt c1il.ng; t.t1l«~ t1'6n, xny rl.1.m~ 2 Mt diing tIJl~~ t.wng; tl,i vf1i b, c r~i cQng ell. 3 bAt d~ng tlnrc lai suy ra

a b (' ab + be + C(l 3

-- + --, + -- > (1 + I) + " - 2 -,

1 + b2 1 + r:.l 1 + (.C2 - 2 2

vi t a co ab + be + ca < 3. Ditng tilltl: dji xay ra khi (J. = IJ ;;;;;; c = I, [J

V(!i nidi Ii"Ull tren co th~ xiiy dirug mOt bAt lIAIIg thuc tu(mg hI vui 4 56

Vi du 1.1.21, Chitn.Q minh rling VOl a, b. c, d ta cae so th1,tc dudng co tdn.Q bdng 4 la co bUl dd119 thric

abc d

--+--+-~+-->2 1 + b? 1 + c2 1 + IF 1 + a2 - .

va n~1l khong dung n~11 khong dung ki thuat Cosi T!gltt;tc dd-u thl g§.n nhir bai toan nay kh6ng th~ giai dlit;1t theo each thong t hmrng dime. Kl thuat nay thuc S1..r hifll qua voi cac hai toan bat d&ng th~c hoan vi

Vi d l! 1.1.2"2. Chvng minh vai moi ,~A ihld7l_Q a, b, c. d thor!. man di€u kifn (l + b +

c + d "" 4 ta co

a b ~ d

-1 -+-;}-c + -1 ~--;::~-. + 1 + cPa + 1 + a2b ?7 2.

1.1. BAt diing rhtl'c AM-GM

L(JI CIAI. Thee hAt ding thul: AM - GM

a ab2e ab'lc ab.fi

--;;= a- -- > a- -- ==0,---

1 + b'lc 1 + b'lc - 2b,fi 2

bJ(UiC b(a + cc)

> a - -- > (I .. _!._ _ ___:_

- 2 - 4

a 1

=> 1 + b~c ? a - 4: (ab + abc)

Hoan toan tirong tl)' ta co them 3 bat d~ng thrrc sau

b b be + bed c cd + cda d > d _ da + dab

~----,,- > - > c - ---

1 + c?-d - 4 I + c'id - 4' 1 + d2a - 4

Cong vI:. d. 4 M.t <ling th11c tren

abc d

--+ + + >

1 + b'lc I + cZd 1 + lf2a 1 + a2b -

1

~ a + b + e + d - '4 (ab + be + cd + da + abc + bed + cda + dab).

Til bAt ding thuc AM - GM d~ dang suy HI cac bin ding thuc 1

o.b + be + cd + da :S 4(a + b + c + d)2 = 4,

1 a

abe + bed + cda + dab $ 16 (a + b + c + d) == 4.

Do <16

a bed

-1 b2 + 1 Z J + -1 .r?- + 1 2b ?> a. + b + c + d - 2 .:;;; 2,

+ e + c (1 + a-a + a

Dang thuc xay fa khi va chi khi a ;;= b ~ c = d = 1. 0

Vf du 1.1.23. Chitng minh 11m nt91 so ih«c d,ldnq a, b, c, d ta luiin eo

a3 fiJ 2 dJ a + b + c + d

--;:-----== + ~- + -- + > -~---

a2 + b2 b2 + c2 c2 + d2 rF + a2 - 2

Lrn GIAL Sii dung bat d£ng thirc AM - GM v($i 2 s6

as ab2 ab2 b

~-- ;;=;Ci- -- > a- - = a-~.

a2 + b2 a2 + b2 ~ 2ab 2

Xily dung 3 bA.t, dAng th1~C titan!!; tu vdi b, c, d rbi cOn!"; y~ <;A.c bAt. nang tlnrc )~i ta co dieu pbw chirng minh. Ding thlic 'xl\y fa khi t~t di. cac bien bang nhau, 0

MOt b8.t ding thli'c cung dang tren lao

a4 b4 c4 d4 a + b + c + d

a:l + 2bJ + b3 + 22 + dl + 2d3 + d3 + 2a~ ~ 3

15

va2/:> ::; b(2a + 1) 't

uc2/J < a(2c + 1).

16

Ctntoug 1. B:h dAJlg thue co .50

Vi du 1.1.24. Cho u, b, c ~ 0 ~Ja a + b + c = 3- ChUng mink

Lor CIA!. Su dung bieu d6i va bat dci.ng thirc AAI - GM cho 3 56

a2 2ab2 2ab2 2(abfn

----: = (l - > a - ~~ = a - -----'---.---C--

a + 2b"2 a -1- 2b"2 -- 3 ~ 3

Hoau roan i\fcIng til ta CIlIIg to 2 bAt dimg t IlIk

~b_2 _ > b _ ~ (bt)"'/J

b + 2c2 - 3 .

c2 2 2/"1

~~---- > C - ··(ca) "

(" -I- 2a"2 - 3

Do do t a chi l:§.n chii"ng mirih

a + b + c - ~ ((ab)2/J + (bc)2/~ + (I:Xt)2/J) > 1 ~ (ab)2/3 + (bc)2fJ + (ca)2/J s 3

Nhung bat, d~ng tlurc nay hi~n nhien dung. VI theo b(\t dll.ng thtrc A1H -- GAl

-,

a + bb + b ~ 3{ab)2N, b + be + c ~ 3(bcftl. c + ca + a ~ 3(ca)2J~,

ngoai fa. de th§.y 3 ?- ob + be + cu nen tao co di(~u phai clumg minh, DAn!l: tll1~t xilV ra kit! va chi khi a = b = c = 1. 0

K~t q"i!, cua bai toan "..!in dung khi thay gil!. thiet 11 + b + c = 3 bo-i ab + be + ca = 3 hoac .fii. + Vb + .;c == 3, t,ruong hop sau kh6 hen mot chut. Ta co them mot bat ding thue khac cung dang tren

Vi du 1.1.25. Cho a.b, c ~ 0 va a + b + c = 3, C}!1lng minh rimg

a2 b2 {"'l.

--- + --. + -_.- ,- > 1.

a + 210 b + 2cJ C + 2aJ ~

Lbl CrAl. Chung minh tuong tlj, dua bAt dAng thUc ve b~ + cW +a~::; 3.

Cong vc ca 3 bat dang thuc tren ta (:0 dl~u ph<i,i dl((ng rninh. 0

17

Vi du L 1. 26. ChUng minh vm m9i so dWlng 0., b, c co tang Mn.q 3 tM

a+l b+l c+l

-- + _.- -- + .... .. ? 3.

b2 + I c2 + 1 a2 1

Lor CIA!. Theo bAt dAng thrrc AM - OM d~ th!y

a+1 (a.+l)b2 b2(all) ab+b

b2+1 = Q, l- 1 - "-[;2+T" ~ a + I - 2b ;; a + 1 - -2-·

Tuong tl! ta co 2 bat liimg tlnrc )Jl."rl\ v01 b. c roi cong lai

a + 1 II + 1 c + 1 ( al: + b) ( be + c) ( (:11 + a)

--+--+-~> (l+1-~- + b+l-~- + c+d---

b2 + 1 c2 + I a2 + 1 • 2 2 ' 2

a + b + c - ab - be - co.

"" 3 + 2 ';:: 3,

Ding thirc chi xay fa khi va chi khi a "" b = c "'" 1. 0

Sau dAy la bai toan tuong tv vdi 4 bitn so.

Vi du 1.1.27. Ch11n,q minh rang vo-i m9i a, b, e, d dW17h} co tjn,q bJ.ng 4 th1

0.+1 b+1 c+l d+1

-- + -- + -- + -- > 4, b2 + 1 ('2 + 1 tP + 1 (l2 + 1 -

Vi du 1.1.2B. ChUng minh rdn,q vai mpi a, b, c, d !luang co tang bdng 4 thi

111 1

-- + -- + -- + -- > 2. ({2 + 1 b1. + 1 c2 + 1 d2 + 1 -

Lo: CIA!. That vay, ta co u{mlJ gia sau

1 a2 a2 a

~~ ;;;;;; 1 - -- > 1 - - = 1 - -.

0,2 + 1 a2 + 1 - 2a 2

San 06 rhl can lam tHong tl.! voj b, c, d,rol cong lai. 0

Ki thuar Cos« n.Ql1!Jc 1£1. mot ki thuat "mal giup gi8.1 quy~t bai to an thea 16i suy nghl nhe nhang va trong sang. cac ket qua diroc lam bang kl thuat nay noi chung cimg rAt kh6 co th8 lam dime theo each khac, hoar phai lam theo each kha did.

18 Clntoug 1- SA ~ dAng' t11~~r co 1;0

1.2 Bitt diing thirc Cauchy-Schwarz-Holder

1.2.1 Bat dilng tln1c Cauchy-Schwarz va uug dung

Djnh It 1.2 (Bitt ding thuc Cauchy-Schwarz). Vth 2 day s6 th¥(: fiLY 'Ii aI, U2, ... , an va /:PI, U2, .,., bll, !a lu6n c6 Itilt dang thac

Ddng thu( xriy ra khi va .hi kh! (al,a2,--"a,,) va {bl,b21 .. "b,~) ld 2 b¢ ti l~, tU(' Lii t6n t(ti ~·6 th1t(.' k ari (Ii = kbj Vi = 1, n.

CH(f'.iG Ml';H C& 3 du.:h den gian dll~ng rninh bAt cling t huc tren.

each 1. Day la eac crnmg mint! quen thuoc st'r dung phinmg phap tam thirc bac 2, Xet tam thirc sau day

Si\U khi khai trien t.a co

Mat khac vl 1(:;:) ;::- 0 Vx E R nen theo dinh Ii ve dii.lI cua tam tbltt~ h~c 2

Ding thrrc my ra khi phm:mg trlnh f(x) = 0 co nghiem, n6i each khac (aI, a2, ... , flrt) va (bl: b2, , .. , bn) la 2 bo ti le.

each 2. M¢t ca.:!. dump: minh khac cung rat (An nhd, VI ta sc :;.1 dung lai t rong 1 s6 hai t ap. do jit phuung phap ~d dung di\ng Ihac, Ta c(} hitng dAng t lnrc sau

"

(at + a~ + -, + a~)(br + b~ + -, + b~) - (a"lD, + G2b2 + --> + at>bn)" = L (albj - Gjbd2_

;';=1

Va do do hien nhien pha.i co

each 3. NgD3.i ra bAt dAng thirc Cauchy - Schwarz ding co the chirng rninh true

1.2. Blit dAng thuc Cauchy~Schw;uz-HoldeI .

19

ti~p hhng bA,t dA.ug thire AM - GAl chi voi 2 :;u, lIft;.' Iii. mot chirng minh rAt hay va J lkll); lIt: Ill,) rOil!!, cho ku !.I,cUIt!, t i Ilk H otder .

Cho i ChiiY t.':f I de.n n rai ccng v6 d.. n hA.t !ling thuc lai ta 1.:6 k~t qua. Day cling Iii. mot clurng minb rfit nga.n gon. 0

Bilt dAng thirc Caw;hy - Schwarz cung Iii mot h~t dAng t.huc; nit quen thuoc voi elk L!;I1l IH,lt; sinh plio thoug va vi!;'!; lJ1lm chAt Slr U\llJg 1 h;',.h thao bat <1illg tlnk nay I", rat can t.hi~t cho d.t ca ban doc. khong chi voi cac ban t hi hoc Sinh gioi, Olympic; quoc gia, quae t6 rna. ngav d. v0i cac ban on thi vao lcp 10 va thi vao u~i hoc,

Cac M qua. san dily s~ cung 1;0 them cac ung dung khae nhau ella bat doing th\1c qnan t.rong nay.

H~ qua. 1.1. VOi 2 day so ((LI, (12,-- ,a,,) vi! (bl,b2, __ .,b,,), b, =2: 0 'Vi = l,n, a~ + a~ + , .. + a:' > (UI +a2 +.,.+a,J;!

b, b'l - bn- b1 + b2 + '" + b,~

CHUNG MINH_ Ta chi ca.n chang rninh trong t.nrong hop n = 2. Val cac gia tri khac, bat d~[lg thuc dlI(1C suy ra true tiep bang pnucng ph up (j1:LY TlI,l.P, Tuy nhien vdi n = 2 thl ta co bat dimg thlir Cauchy - Schwarz

Ja1 +bi + Ja~ .... b~ ~ v'(aj +a2)2 + (bl + b2)2 <=> (af + a~)(br +, b~) ;;;: {alb, + a2b2)2

DAng tlurc (;ung xay ra khi (aJ,a~, ,,-,Q,t)-vA. (bl,b2, ".,bTI) iii. 2 bo ti Ie, 0

(al + (l2 + --- + a,,)2 :::;; n(a~ + a~ + ... + a~).

20

Chuang 1, Bat dAug thue co ,~(;

CHeNG MINt1. sa d\!ng hilt ding thuc Cauchy - Schwan vdi 2 be) so

(al.a'l, ""a,,) . (1,1, .. ,1),

Trong tI6 b¢ ti6 thu 2 gom n dl 1, 0

Sau day lit. nhttng Ung dung ttHlong gap va ti{'\\ bieu ctia bat d~ng thuc CauchySchwarz Ta xern xet qua cae vi du tl{ dun giAn d~n phuc tap

Vi du 1.2.1 (Math Changel les). Cho cdc so thuc dutmq XI, ;<:2,·, x". co lang Mng 1, Hiiy tim .QiJ. tri nho nhd.I C!((! bl.€u thUc

( 1)2 ( 1 )2 ( 1 )2

Xl + - + X2 + - + ... + x" + -

Xl X2 In

LOI CIAI. Sit' dung bat ding thuc Cauchy - Scluuar z ta co

(XI + J:_)2 + (X2 + _..!:_)2 + ." + (xn + _..!:_)2

Xl I2 In

1 ( 1 1 1 )2

?-: - Xl + X2 + , .. + Xn + - + - + , .. + - ,

n Xl X2 Xn

1 I 1 n2

_ + _ + ... + _ > = n2.

Xl X2 . In - Tl + X2 + ... ..L. In

Tlt 2 hAt ct~ng t lu'rc tn"n tn sny ra

( 1)2 ( 1 )2 ( 1)2 (n2 + 1)'

Xl + ~ + X2 + - + ... + ;r,~ + -- 2": '

Xl X2 Xn n

Diwg thuc xii.y ra khi Va cbi khi Xl ;;;;; X2 == .. , = xn = lin. 0

Vi du 1.2.2. Gin ~n£ a, b, c ia cac so th~(c duong, chi'tnq mink ding

abc 9

-~-+ + >----

(b+c? (c+a)"2 (a+b)2 - 4(a+b+c)

Lot CIA!. Su dung bAt dAng thuc Cauchy - Sciuuar z ta co

(a + b ... c) Cb: C)2 + (c: a)2 + ((1. : b)i) Lai theo btl.t <.laug tlllk Coucln: - Sch'wc;l'z

abc ai b2 c'l. (a+h+c)2 3

-- + -- + -- ;;;;;; + -- + > > -r- ,

b + c a + c a + b ab + tu: ba + be ea + cb - 2(ab + be + cal - 2

(a b r.)2

> --+--+-- b+c c+a u+b

Til' 2 bAt dAng thu.: tren ta c6 dpcrn. Ding thiiIC xay ra khi a = b = c. 0

1-2. Bllt dAng thtl'c Ceuchv-Scawerz-Hohier

21

v r d\l 1.2.3. Cho cac ,,6 duang a, b, t: co U;n.g Mn9 3. Chung minh rlinq

0;2 02 c2

-----"'" + -- + > I.

a + 2b2 b + 2c2 C + 20;2 -

Lal OIAI Bai toan ua. dltQc giai d phan truce bAng ki thuat C6s'i ngu{!c. duu, tuy nhion lai co th!": gi:'l.i khft don !-:i~'in b~[lg b{i.I lIAng tInro Cauchy - Schwarz nhu sau

aZ 62 ('2 (a2 + b2 + (2)'l

-----::-:,-,;:+--+ > .

a + 2bZ b + 2('2 C + 2a2 - a3 + /;13 + c3 + 2(a2b2 + b2c2 + c2a2)

Do do ta chi cAn chirng miuh

(0.2 + b2 + (2)2 -2:: aJ + /;3 + (;:1 + 2((l.2/;2 + b'2c2 + c2a2) 'ri- a4 + b4. + ('4 ~ 0,3 + b3 + c3.

B.§.t. d.\llg tht"rc nay kha quen thuoc, ta Co the dung chrmg rninh hoan toan chi' bang bat. dling thU'<,; CaHchy - Schwarz

3(a3 t· 63 + c3) = (a:1 + b3 + c3){a + b + c) ;:::: (0,2 + /;2 + c2)2 (a2 + b2 +(2){1 + 1 f 1) ~ (Q + b+ <-"?;; 9-

Do till a2 + /;2 + CZ :.::: 3, suy ra a~ -'t- 0.3 + cJ ~ a2 + b2 + c2_

(a4 + b4 + c4) (a2 + /;'2 + c2) ~ (aJ + b3 + cJ)Z => a4 + b4 + c4 ~ a3 + b3 + c3.

Dang thuc chl xay fa khi a = /; = c = 1. 0

OAt. dAng thrrc rren n\t~c dum!!; minh d~ dang hAn!') rAch :;;U dung bti.t rlA..ng thlic Cauchy - Schwarz, tuy nhien vci Mt ding thuc kha tuong tl,t khac

(}2 b2 (2 3

---+--+-->-

Q. + b'l b + (:2 c + a 2 - 2'

Ta chi co tb6 sit dung phuong ph.§.p Cosi n.Q1tI;1C, khong sir dung dlti),~ true tiep duoc bat dang thirc Cauchy - Schwarz.

Vi du 1.2.4 (Vitiit Nam MO 1991). Cia 8U x ~ u -:::: z"2: o. ChJ1ng minh

Ldl CIA!. Theo bAt dAng thuc Cauchy - Schwarz ta c6

Ch ltdllg 1, BAt daHg th ur: ca sa

Mo:tt khac VI T :;:: Y ~ z nell

Tu do ta co di€-u phai chang minh. D~llg thU'c xay fa khi x -= y = z. 0

Vi d\} 1.2.5 (Iran MO 1998). Gia ~''Ii .r , y, z "2: 1 11d ~ I- .~ + ~ = 2, Chttn'9 nuuh. x y z

1 I 1 x ._ 1 y - 1 z - 1

Lor CIA!. VI - + - + - = 2 => -- t- -- + -- = 1.

x y z x y Z

TIJ\'o bAI (tilng tlll~~' Cauclu) - Schwarz ta co

X+Y+Z=(X+Y+Z)(X-l +y-l +~)

T Y Z

:2

~ (~+~+JZ="l)

=> ,j x + ']I + z ;:: ~ + y'i/=-r -t JZ="l'.

Dhg thl1c xey ru khi va chi khi x = Y = z = 3/2, 0

Vi du 1.2.6 (Greece MO 2002). Cho cnr: sJ ttu«: dttdn,q (I., b, c co t6n9 IIdnq 1, CMtng minti bUt ddng th'Uc

abc r.:: r, . r.:;,2

4112+1 + 4c2+1 + 4a2+1 '2: (ava+bvb+cvc) .

ClHJ)O{G Ml:>1J-!. Sil' dung t.nrc tiep bat dil.ng thuc C(wchy - Schwarz

abc a3 b3 el

b'l + 1 + c2 + 1 + a'l: + 1 ;;;;; 4a2b2 + a2 + 4b2c2 + b2 + 4c2 a2 + ['2

2' 4(a2b2 + Pc.':! + c2a'l:) + (12 + b2 + c2 - Phan con lai n'la bai toan, tao chi r.&n chirng rni IIh

4(a2!J'l: + b2c2 + c2a2) + a?' + b2 + (? :s: 1 = (a + b + ef' <=> ab(l ~ 4abl+be(l- 4bc) + ca(1- 4co.) ~ O.

BAt (Hillg thul.: ,lily hi0u nhien UU)lg vi a + b + c ~ 1 ~ ab, be, 00 -:; 1/4. D~ .. ng thuc xay ra khi trong 3 sa a, b. c co 1 56 b~ng I va 2 $6 bAng O. 0

1-2 BAt djng thuc C'<luchy-Schwarz-Holder

23

Vi du 1-2.7 {Crux}. Foi mQi x, u, z ~ 0, chUng minh

v9"+1 + v11+l + Jz'l + 1 :::: V6(x + y + z),

L<11 G I AI. Ap (l\mg h~ qua (1-2) ta co

. ~ + .JYI+l + J;2'+l:::: V(.r + y + z)2 + 32 Then blLt dD.lIg t.hirc A M - GAl ta 06

(.1: + Y + zl + 32:::: 6{x +)f + z ].

DA.n~ tlntr. xay ra chi kit! x = y = z = 1. 0

Vi du 1.2.8. Xci" din"- dilv ki~n cJn va dtl 11di cdc so tlntc TJ, r2, "'1 T,I sao cho

Len CIA!. Cho Xi ;;;; 'i ta BUY ra day (ri) phai thoa man rt + r~ + .. _ + T~ ~ L

Day (llUg Ik cli~u kien att. that vay, nell Jay (r,) t hoa mall di~nl ki~Jl !lay t.hi theo bat dAng thl"(\: Cuuchy - Sdm,Nlr2 t a co

Khang dinh dtLt;!c chung rninh xong. 0

Vi d u 1.2.9 {IMO Shortlist 1993). CMng minh uoi moi 86 duang a, b, c, d

(l bed 2

----+ +~~---+ >-_

b + 2c + 3d c + 2d + 3a d + 2a + 3b a + 2b + 3c - 3

LOI G 1'\ L Then bAt d~ng tlllk Cauchy - Schwan ta co

~ ~ 2 ~

VT == + + ----:----- + -----

ab + 2ac + 3ad be + 2bd + 300 cd + 2ta I 3cb da + 2db + 3dc

{a + h+ c + (W

> ~~~--~~~----~

- 4{ab + be + cd + da + ac + bd)

M(1t khac

(a + b + c + d)2 = 2( ab + be + cd T da + ac + bd) + a2 + b'); + c'J. + d'l

8

~ 3(ab+bc+cd+da+ac+bd).

Tu do ta t:6 dpctn. Dll.ug ttJu\~ xay fa chi khi a = b = c = d. 0

24

Clltwug I. BAt clAuS thuc co sO

Vi du 1.2.10. Ch1ing minh urn mri a. b, c dU(Jrl,g

aSb fh:. 20, abc(a + b + c)

I + ab2 + 1 + bc1 + 1 + w2 ?:: 1 + abc '

Ldl GIAI. val moi $6 time dirong k. theo b~t dAng thirc Cauchy - Schuiar z

a'1 62 c'1 (a + b + c) '2 a + b + c

--+~-+-~>-....::...__-....:;_~

b + kc c + ka a + kb - (k + l}(a + b + c) k « 1 .

Ui.y II; == l/(abc) ta co di~u ph iii clnrng rninh Ding thuc xey fa khi a = b 0:: C, [J Vi du 1.2.11 (Bit ding thuc Nesbitt 6 bi€'n). Chimg minh rdn9 v6i 11I9i a, b, c, d. e, f La car s6 thuc dUdn.q

a b r. d e f

_- + -- +--+ -- + -- +-- >3-

b+c c+d d+e e+J J+« a+b-

CHUNG MU'I]-I, Theo bit Il£ng thtrc Cauchy - Schwarz

abc d e f

--+--+~-+--+--+--

b+c c+d d+e e +/ J+a a+b

a:1 b'l c'2 d2 e2 {:t

--- -. + -- + -- + -- + + _.,;;:.__-

ab+ac bc+bd cd+ce de+df f'./+ea fa+fb

> _ (a +- b + c + d + e -11)2_

ab + be + cd + de + ef + fa + ac + ce + ea + bd + df + jb·

GOi S IS. mau s6 eua phan thUc n6i tri5n

2S = (a + b + c + d + e + f)'i - (0,2 + b'l + c_2 + d2 + e2 + j2 + 2ad + 2bd + 2t:!)' Theo bat dang thuc CalLChy - Schwarz ta c6

a2 + b2 + c2 + d2 + e2 + f'i + 2ad + 2bd + 2cf :::: (a+d)2 t(b+e)2+(d+f)2

1

~ 3(a + b+ c+ f + e + 1)2.

Do do 2S :-:; 2/3{a + b + c + d + e + J)2, ta co dpcm. Ditng thirc xiiy ra chi khi a=b=c=d=e==!- 0

Vi du 1.2,12 (Korea 2002). Hoi day 86 thl!c a), a2, .. -, atl va b., 02, ... , bn thllli man

'2 2 2' b2 b2 b2 1

al + a2 + -,. + an == I + '2 + ". + n == -

ChUng minh Mt ddng !hitc

(a1b2 - a2bl)2 :5 2~(Llbl + a.2b2 + --- + a"b" - II.

1,2, Bat d<~Ilg t hlk C'allchy-Schw<!rz-Holder

25

Lo! CIA). Trong bai nay ta phai 8U dung dang thuc khai tri~n bll.t dang thUc Cu'uchy· Schwarz, au In

T!

;;;;;; L(a.,bJ -ajbJ.! ~ (a,b2 -a2bd2. i,j='

Thco gia thii:t ai + a~ + ... + a;, '-'- br + b~ + ." + b~ ;;;;;; I nen theo b!i.t ding thtk Cauchy - Schwarz ta co 1 :;:::- al b1 + a2~ + ... + a'tbn ;::0: -1. Do do

(1 - alb] -- ll2b.:! - .. - - at,btt){l + alb, + <l2b:z -f , .. + anbn) ;::: (atb2 - a2bt}2.

=> 21(LI/)1 + a2b2 + --- + a"b" - II ~ {aJb2 - (l..JbdZ. U Vi dl.;l 1.2.13 (Crux). Tim yili try nhd nhcit, (:'1iu bie~u thUe:

3a 4b 5c

--+--+--.

b+c c+a u+b

Lo: G IAI SU' dl)ng b:it. d~ng Cauchy - Schwarz

3a 4b 5e· (3 4 5)

-- + -- + -- + (3 + 4 + 5) = (a + b + c) -- + -- + -b+c c+a a+b b+c c+a a+iJ

1 (3 4 5) (J3 + V4 + I5f

;;;;;; 2-(b+c+c+a+«+/)) ~b-+--+--b ~ 2 .

+r. c+a a+

Suy fa gja, tr] nho nhAt ella hi~u thirc bang ~ ( /3 + V4 + y'5)2 - l2.

'. b+c c+a a+b

Dang thm: xay ra khi va chi khi v'3 = -2- = J5' 0

Vi du 1.2.14. Chflng minh rdn,q net! a, b, c ~ 0 va abc = 1 thi

III --+--+--<1.

2+a 2+b 2+c::-

Lei OIAI. BQ.t ding thuc tuong dtwng voi

2 .22

1---+1---+1-~->1

24(1. 2+b 2+c-

abc

~ --- --- + -- + ~~. > l. 2+a 2+b 2+c-

ry

0.=-, .1:

b -= ,.<; • y .

rf.

(>- -

. - .

z

I r:r ("'" -.

l

26

Ton t~i cae s6 thirc z, y. z sao cho a;;;;;; xl)}, b = y/;;, c = ;,;/x. Ta pMi chirng minh

xly y/z ziT

_ .. - + + > 1

2 + ;r/y 2 + yj z 2 + z /.1' -

r y z

~ -- + -'-+ _. - > 1.

x + 2y y + 2z z + 2,r -

Theo bat Uil.llg thuc Cauchy - Schwarz ta co

x y z (x+y+zfc

__ .1' __ +--> ;;;;;; 1.

To + 2y y -I 2z z + 2x - x(x + 2y) +y{y + 2z) + z(z + 2:r)

Di\.LJ~ thlie xay fa khi va chi klii .J: "'" Y = z hay a ;;;;;; b = c: == 1. 0

Vi du 1.2.15. ChUng minh. rilny vdi. moi a, b, ('. d duong co tich hriny 1 Ihi

1 1 1 1

_~--:-+ + + >2.

a(l+b) b(l+c) e(l +d) d(l+a)

Lot GIAL Do abed =;= 1 nell ton tai cac s6 thirc x, y, z, t !;ao cho a = ylT, b = ;0;/)1, (' = t/ z, d = ,r./t, Thay fl, b, c, d hal .T. ", a , t V~l.Q biH- diiJlg tlnrc

1 1 I 1

__ + + + > 2

ylx(l + z/y) zly(l + tjz) t/z(l 1 :Elt.) . x/t(l + y/x) -

x y z t

~ -- + _. - + _- + -- > 2.

1) + Z z + t t + x x ·1 y -

Day II; hoit dang Hnk Nesbitt 4 hi~n dA duoc chirng minh d trl'n, 0

Vi du 1.2.16. ChUng minh .ling neu a, b, c ; d liJ. e(ic ,.,6 tiqtc duanq va 1'4 = obcd ~ 1 Mt ddng tMc sau lutm dung

ab + 1 be + 1 cd + 1 da + 1 4(1 + 1.1)

--+--+--+--> .

n+l b+l c+l d+l - l+T

LD! CIA!. Chu }' rrmg U:I giil. thiet obcd = 1'4 :::: 1 ta suy ra ton tai (';ac ~ thirc dirong x, 11, 2:, t sao cho

Khi do bAt ding nul!; ul1(jC vi6t lni ,.hMi dang

~ + 1 2J. + 1 F1x + 1 ¢£ + 1 4(1''2 + 1)

~ + y z '> --,- _ ____:_

~+l !:..f+l + !1+1 + o:£+t - 1'+1

I Y /

r2;;+x T2t+1! r2x+;:; 1'2y+t~ 4(r2+1)

(""}--+---+ +---.> .

TlI + X rz + y rt + z T.T + t - 1" + 1

1.2. Blit dAng thll'c Ca!1chy-Schwtlrz-Hokler

27

X+Z lI+t x+z y+t

A ;=;-- + -- + -- +--

ry+x 1'z+y l't+z rx+t

z t x y

B ;;;;;;-- + -~ + -- + --.

roy + :r T Z + t rt. + z r-.r + t

Ta phil chang minh

.... 4{r2 + 1)

A+{r~-l)B~ .

1'+1

Thee bat dAng thirc AM - G M dr thay

4r(xy + yz + zt + tx) + 8(n + yt}

= 4{r - 1)(,1; + z)(y + t) + 4{:t + z)()/ j t) + 8.u + 8yt ~(r ·1){,r+1j+z+tf+2(.L+y+z+t)~

S (1' + 1 Hx + '!I + z + tf·

!:id ul,1ug bat daJlg tlnre Cauchy - Schuicrz, chu y d.ng 1" 2 1 ta c6

A::: (x +z) (_1_ + ~l_) +(y+t) (_1_ + ~l_)

ry + x rt + z r x + y r z + t

4(x + z) 4(y + t)

-___;_-----'--+----,------'---

,T + Z + 1'il; rt. y + t + r.r + rz

> 4{x+ 11'+ z + t)2 > _8_

{x + z)2 + {y + t)2 + 2r(x + z)(y + t) - l' + l'

(x + y+ z +t}2

B?_

z(ry +:1:) + t(rz + y) + x{rt + z) + Y(1'X + t)

(:r+Y+Z+t)2 >_4_

r(:ry + 112 + st + t:r} + 2(:r.::: + yl) - r + 1

>

>

Tir 2 bat dAng thirc t ~'ell tot co di~l.I phai clnrng rninh, D?~Jig t hl!'!: xay ra kh: va (hi khia;;;;;;b;;;;;;c=.::.d-r.D

Sail d5.y r.hfmp; ta Sl~ nghien ciru mot. dang md n,np; Ul1(ooj.!; g~p va fa nhicn irng dung c,'la M.t dang tbrrc Cauchy - Schwarz, d6 la bat d~ng thirc Holder,

1.2.2 BAt doing thLi'c Holder

Dlnh Iy 1.3 (Bat d~ng thuc Holder}, Voi m day !:iO durtng {(~1.l,«~,2' .... ,nL,,),

(G2.! , G2.2, .". , r22"J ... (a"" I, (.1.".,2, .... , am.,,) ia co

28

Ch l(dlJ.g 1_ BA r d.1.Ilg t h Uc co !jiJ

Ddng Ihuc xiiy ra kh: n, day do trW(!g lil1g Ii If Bar dling tiuu: Caurhy - Schsoarz Ii! h~ qu« true t'iep tuu Mt driny thuc H older Vdt rn = 2,

H~ qua 1.4. Voi U, b, c, x. 1,1, Z, rn, n, p Id cae so tnuc duonq ta co

(a3 + bJ + ('3)(X3 + y:l + .. 3)(mJ + )1J + p3) ~ (arm + byn + CZp):I,

CHeNG ~·11 [,:H, Thuc ra diiy chi lit hi;l qua true tiep (;113 bAt dilng t h ltc Holda voi m = n = 3_ each chirng minh sal! co th~ lam lai hcan to an tuung h_r vul d~ chirng minn bfit di-ng thtre H older dang dmg quat.

S~ dung b~t diing thuc AM - GM ta co

a3 xJ mJ

aJ + b'J + c3 + .1:3 + yr+"ii + m:l + mJ + rP 3axm

> ,

- vra3 + bJ + 2)(xJ + y3 + z3)(m:l + n3 + p3)

Xay dimg tuong t!J 2 hat ding t.hirc mra vui (b, y, n) va (c, z,p) rai cong thco tirng vt, t,l. I:() uil·u phai clumg ruiuh I:l

Mot hi;> qua. cua bll.t (lin!l; there Holder- thiroug diroc ap dung la

H~ qua 1.5. Vdi dU:!I .~6 dltrtw; (ll, [li, '., (1'1, r:h{tn._q minh

Cmrxc MI:-iH, Sit dung bAt. cliing thite AM - GM ta co

1 1 1 n

-- + -- + -., + -- > --;:,:=======:=======:=

l+al 1+a2 l+(.In - \i(1+ ad(l +a·l)---(l+a.~»

(II ([2 (lt~ n 1aJa2--.a'l

-- + -- + .. , + -- > --;:====~===;==;=====;= l+Ul 1+a2 1+(1" - V(1+ad(1+a2)--.(1+a .. )'

Cong thea timg v~ 2 bAt dAng th(tc tren ta co di~ll phai chirng rninh. 0

Bilt dillg tlnrc H older 103. mOt b~t dimg tlnic mauh va C0 uhif-u itng dung, nlnrng rAt t i~r no klu"'!nv; dltqC rlim!!; pll1-) hiPn a phi> them)!; hil~1l [jay. C~ic II(: !jl1il ril~Jlg () trl~1) de minh hoa cho bat ding t lnrc Holder c.ii v~ deb dHl'ng minh IAI! cii..ch Sif dung, Viec lam quen va thuan t hue voi bAt diing thuc nay cung rAt can thifot. vi tit)' rt.ng co m¢t phat bi~u hui kho nhln. hat dang thirc Holder- hoan toan de hi<Su va irng dung hieu qua vdi nhieu bai toano Scm day 11'1. mot 5& vi d1J-

vr du 1.2.17 (IMO 2001. Pro A2), Vdi m~i a, b, C dll(1ng fa co abc

----;=;;;;;:===;=;= + + > l.

.jo.2 -+ 8bc vb2 t- Sac ';r;2 1- 8ao -

1.2. BAt d.ing th;)c Csuchy-Schwese-Hoklet

29

LOI CiA!. Xct cac bi~u thirc

Theo bAl clfillg t lnrc Holder voi 3 day (he qua lA). ta <:6

Ta bay gio ta chi d.ll chrmg minh ((1 + !J + c)J ~ R, hay

Diug tln'!t.: xay ra chi khi a ;;;;;; b = c. 0

Vi du 1.2.18. ChJ1ng minh nln.Q n~u a, b, c lii cdc sa tJu,lC dJ.t(Jng co tick Wing 1 thi t~ Co cdc Mt dling thue sau d6;y

abc

+ + > 1,

.../7 + b + c ';7 + c + a ';7 + a + b -

a b r

';7 + b2 + cZ +/:; +-c2 + a2 + ';7 + (12 + 1/1 ;::: 1.

Bat dang thUc sat! co dung hay kh017.9 ?

t.oi GIAL Vdi h~t d~ng t huc dim tien, d~t

abc

'1/7 + b + c + ';7 + c + a + ,)7 + a + b ' a(7+b+r.) +b(7 t a ~ c)+c(7+a+b),

Thee In\t ~lfmg: tllU\: Holder ta co

A2B ~ (a+b+cf

Ta phii clnrng rninh (<< + b + c)3 ~ B ~ Z{a + b + c) + 2(ab + be + ca). Thea h~t ding thuc AM - GAl, a + b+ c:;;:: 3~= 3 ncn

(a + b + (')3 ~ 7(a + b + c) + ~ (a + b + c)2 :2: 7(a + b + c) + 2(ab + be + cc).

30

Vui hAt dhg thirc thu 2. ta dar

c~

abc

..J7 + -p --t- c2 + vl7 -+ 0.2 + (:2 + .j7 + a2 + b2' a(7'" b2 -I- ("2) + 0(7+ c2 + (2) + (:(7 + a2 + b"2),

D=;;

Theo bAt dAng t huc Holder t.hi C2 D ? (a + b + c)3,

D"" 7(a+b+c)+(a+b+c)(ab+bc+ca) - 3,

<

(0 + b+ e)3 ,

7(0. + b + c) + 3 - 3 -::; (a -I- b -I- c);}-

Tit d6 suv ra di'::'u plJ:l.i chung rninh. Bal dhg thirc t.htl 3 khong d(\ng_ Th.;i.t "8._r. ta chi can cho a_,O va b;;;;;; c _, +oc. chimg han (1 = 10-.4, Ii = c = 100. 0

Vi du 1. 2 .19 (Junior Banlkan 2000). ChUng mink 1,4i moi o. I!, c dUrfng

aJ bJ eel «2 b2 c2

- + - + - > - + - +-.

b"2 c2 a2 - b c a

Til' 2 bAt. rl:im~ tlllrl~ trfn t~1 co di~l\ phfl.i eiu'mg: minh, 0 Ta co bai t0ii n tong: quat S3.1.I

Vi du 1.2.20. Vd1 a,li, c 14 ('(k M1 IJP,tc ~~ :;::: 1, k EO :V Ihi

ak+ I bk+ I Ck+-1 (Ii< b~' ck

7 + T + 7 :;::: b!,;-l + Ck-1 + ~k-l .

LOI GOII, StT dung hAt ding thac Holder-

(!.: bk I: )

__ a_ I -L-l + -i-1 (b + a + C)k-l ~ (a + b + c)~

bJ.·-1 c~- aN-

ak bk ck

=;. b~-I + ck - 1 + ak-I ;:: n + b + c,

(ak+l b~';'1 Ck+1) k-l ( ak b~- Clc)*

V + ck + -;l"C (0.+ b.+ c) 2: b~-l + Ck-1 + a~-l

Tir 2 btu. d~n~ thuc tren ta rut ra dieu phi1i clnrng ruinh. KCt qua nb.y v!..n d(lng nt:n k hfru ti (chung mint! tirong nr), va do d6 se dung voi rnoi :;5 thirc k ~ L khcng nhat thict k Iii so nguyen 0

1,2_ BAt ddIlg tJli~C CalIChy-Sd:rWMZ-HoJdt:r

31

Vi <11.,1 1.2.21 (Singapore MO 2002). CMrng minh vdi mQ-I sd dttdng a, b, c 3{a3 + b3 + c3)2 ? {a2 + b2 I c:2)3,

1,(.\1 G IA I Si( dung true tiEp uat utuj~ thl"[l' H oldt:r

{aJ + bJ + c3)(aS + b:l + c){l + 1 + 1) > {(l~ + b·l + c2? 0

Vi du 1.2.22 (Crux). Vai cdc s& th1,1c d-uan_q (1,1, U2, ",,11,-, co t6ng bang 1, My!im gia tn nh6 nhoJ cua bilu UUIt

Loi CIA!. Xct cac bicu th{fc

Then b&.t di\.ng thuc Holder ra co

:.I. 3

A _ B ? ((.q + a2 + ... _._ (I,,) =' L

Ngoai ra de th~y

B _. ("2 2 2) 1 _ (al + (l:!. + .. , + a.~)2 _ n - 1

- 1 _ (1.1 + a2 + _" + a .. ::;: - - - --,

n n

V~ . 1 -

Do rlc'i A 2 - . DAng t.lnu: xay [<1 khi va. ~~hl khi Uj ;;;= _. Vi ,.. l, n. 0

Tl-I 11

Vi du 1.2.23. Gho cdc so th'llc khOng am a.b,c co t6ng Mng L Ch{mg minh r6:ng

(J; b c

J~+ 3~+ J~>l.

v It -r- W vb + 2(" V C + 2a --

Lo! G IAI, Xct tac bicu tlll:!I:

abc

S=c... + +~=-

Va + 2b -iYb+2c ;yc + 2a'

P = a(a + 2b) + b(b + 2c} + c(c + 20.) = (a + b + c? = 1.

Theo biit ding tlurc H olde:r

S3.p ;::.: (a + b + c)4 =} S3 ~ {a + b + cf = 1 => s:;::- 1-

DAng thirc xay HI khi va chi khi a = b = c = 1/3, [J

32

Clutaug 1. BM dAllg thuc co sa

Vi du 1.2.24. Ch1l:ng minh IIdt dtinq !hue

{l b c

----;::=;=;;;=====:;=:;:==-=---"'-- + + » /3

-./2b2 + 2('2 - (1.-2 .j2c"l + 2((2 - b1 /202 + 2b2 _ ("2 •

Tnmg d6 a, D, (' Iil. cac 56 tliuc d1tffnq sao rho cae can thuc ton t«i,

Loi GIAI, COl bit;u thuc V{'t tr ai 1ft P. DM

5 ;;;;;; a(2b2 + 2cz - a2) + b{2c2 + 2a~ -- b2) + r.(2o,2 + 2b"i - c~}

;;;;;; 2ab(a +b) + 2bc(b+ c) +2ca(c + a) a3 - b3 - c3.

Theo bat uing thirc H older ~a co

p.P.S2: (a+b+c)3_

Do do ta chi cAn dump, minh

(a+b+c)3;:::3S Bat dii.ug thuc t I'~n nrong dircng voi

4(oJ + bJ + cJ) -I- 6abc ~ 4 {ab(a + b) + bc{b + c) + (:'(1(c + 0.))

Dc co bfit clAng thuc trcn, til. chi dn u)ng Vl~ 2 brlt dii.ng thuc que)"! thuoc sau

3(aJ + b3 + c3 + 3abc) ;::. 3 (ab(a + b) + be(b + c) + ca(c + a)), a3 + b3 + c3 2: 3abc.

Dimg thlrt: xay no. khi va chi khi a = b = c. 0

Vi du 1.2.25. Ch11nq mini: ning nb.( p ~ 2 va a, b, c > 0 tM

3 u:.l + pabc 3 IJ3 + pabc J c3 + pabc

1 + + < a + b + c.

p+ p+l p+l-

Lei CIA 1. GQi P Iii. bi~u t !Jue v~ trai, ap dung LAt tliing thl)(; H older ta Co

(a + b + c) (((.(2 + pbc) + (b:l + pea) + (c2 + pab)) (1 + 1 + 1) ;::: (p + 1)P3_

Do do ta chi cfi .. n chirng minh doug

(a2 + pbc) + (b2 + pea) + (c2 + pab) ::; p; 1 (a + b + c)2_ BAt dAng thirc nay hjGn nhien VI

2 2 p-2 2

VT = (n + b + c) + (p - 2){ab + be + cc) ~ (a + b + cJ + -3-(a + b + c) = V p.

Ding thuc x:ay ra khi a = b = c. 0

33

Vi du 1.2.26 (Ml1 ri?ng IMO 2001 Pro. A2). Cho cdc sd th1c duong XI, X2, . ,., X11' Di).t y~ = XIX";! ... T;-lxi+IX;+2·.;T,,· ChJny tuini: "tit rliiny thu(;

Xn

+ ... + - d~.:""--====== ~ L "--(I x::-I + (n,,-I - l)y"

L<JI GIA!. Dat S lll. vt~ t.rfii ula bAI rlitng t.hll:1" vh

p.:.;; Xl (:r7-1 + (n,,-l - l)yd + £1 (Xz-) + (n,l-I - 1)Y2) + + ... + In (X~-J + (n,,-I - l)Yn)

" + "+ " + (7\.-1 }

= Xl Xi ... + x" n - n XiX2,··X,!.

Thno b:ll. {t.\UP; tlnrc Halder ta co

Ta pMi chimg minh

Tuy nhi(~11 bit. IUlug tlnrc tren I LiC'n nhien dtmg vi sa u khi I ach II + ;1:2 +- .,' + .1:;: ra khoi (Xl + X2!' ... + In)'1 thi cac ~6 hang con ];,l.J n:' the bi€u di~n ct6i xirng thanh cac tong Ion hdn hoac b~ng IIX2."X". ':J

N~u bAt liAng tlurc giii.i dUVt: LAng Holder thi eilHg s~ gii'J.i Ul[l)l: LlLng bat t1Allg tlurc AM - G M, VI thirc til, de clnrng minh bfit dang thuc Holde»: ta CUl1g chi cAn dung b.at ding thtk AM - GM" Ch~g han vdi vf du 1.2.26 ta co th6 dung true ticp bAt ding tlurc AA-f - GM theo each san day

Dat M "" III + U2 +." + a", ap dung bitt d~llg th(K AM - GM cho n so

nXI >- M·

,~ (1) ,,"," " + (" )

"'"' n - Xt L..".,-I·1:; n - n XIX2 .. I" > n

~ .1 / + At" - "

i=l n-y xt-l + (ntl-1 - 1 )Yi

34

Cb,taug 1. Btit doing' tMc co siJ

Nhu v~y t.a chi phai chrmg minh

Ngoai ra sO k = n'l-I - 1 .:i'mg Ill. hAng ,so dimng t.flt nhat {nbo nhfl.t} 0(-\ bAt. OA.n!l; thuc san dung vdi moi 56 thirc dinrng x), :1:2, ... , x"

Xl :1:2 :rn n

"-iri~-l + k1/l + "-~./'r'2-·1- ~"k;~ + .. , + rr .:I:r;;-I + kYn ~ n--Yf+k'

Voi ~;\~ bai tonn t ilrm!1; t lnr?mg t hi bAt cl:An!!; tluk Holder lai dung t huan t icn han bat dAng t.hirc AM - G M vi khollg I;.§.n xCI <Ju: di~u kl~n ding tlnrc, r:bn t.rong trircng hop tong quat ban chi can gbi nho chtrng minh vdi m = TI = 3 lit dl1. Xern them phan sir dung hil.t ding thue H older d~ can h~ng h~ :>0, trang 74.

1.3 BAt diing thirc Chebyshev

1.3.1 BAt dang thl1c Chebyshev va U'ng dung

Djnh ly 1.4 (B.§t dAng thuc Chebyshev). Vdi 2 day 3d th'l!c dan difu tting Ql, a2, ... , aH va bl, b2 •...• 011 ta cO

CHUN G jI.·I1NH. BAng philn t feh tn,tr: ti€p, til. co d~llg thuc sau

n

- L (a, - aJ)(b, - bJ) 2: O.

;.;1=1

Vi cac day (.1), a2, .. " a'l va b), b"], .. " b" don (lieu nen (aj - aj)(bi - bJ) :;:: O.

N iSu 2 dAy (Zl, a2, ... , aT, va b\, b.:!, ... , b, dan dieu nguQ'c chieu thl bt..t diLng t hl~\: tren d6i chien. Chung minh di~u nay hoan roan tuong W. 0

ThOll!; thHvllg !.:;«; Liit u~ljg tlll,A: ubi ;{al\~ v(ii r:ar; lJi~l), lieu vit?r: ~a.V x~V lai ~ax bieD luon ~O thf tlnrc hi~n diroc. Do d6 truce klii r>lr dung bat dfmg thuc nay ta phai co mot bucc sAp x6p lai cac bien rna khcng lam mat t.inh t6ng quat cua bai toan Luu y r~ng dieu nay chi dung khi va chi khi MJ da.ng thdc dln clnrng rninh holm

1.3. BAt ddng ~M'c Chebyshev vol U'ng dung

35

toan ci61 xrrng vui tAt ta cae hi~n_

Bat dilng th((~~ Cht:b'!-jshev co nhicu ling dung rat hay, va noi chung lam cho hai toan du'~!; giili quyf! theo each dan gran han trong kha nhicu truang hop- Chung ta cung xern xet cac vi du san dJ thay ro di~1l nay.

an+) I ~n+1 + + a,,+l > a'~ + an + + an

I '"2 -.. " - 1 2 .. . 11'

CmiNG MI;-':H. Sv dung lilt dKll!!; tli\k Chebyshev cho 2 hi) (tan di¢u aj,a:!. --·,Q.1,

va ai', a:; , , a~ ta co ngay dlhl ph.ii. chrmg minh. 0

Vi du 1.3.1 {Balkan MO). Cho cac 80 thJ,fc dUdng {ll,{lZ, ---,an thoa. man ill + ai + --- + a1l = 1. Ch11ng minh

al (12 an n

-- + -- + ... + ._--- >--

2 {l1 2 - U2 2 - (1" - 2n - i

Lor CIA!. VI b&t. dang tId!(_; dbi x\~ng \-'ui t.it ni. cac bii-n nen t a co th~ gi3. :o;U dINt.: al ~ tt2 ~ -. ;::- a'l_ Khl do

1 I 1

--> --> ... >--.

2 - al - 2 - 0;2 - - 2 - O;r!

Theo hAt dfi.ng thirc Chebyshev thl

1 (1 1 1 )

VT:::: -(111 + o:? + ... + (In) -2-- + -- + ... + --

n - al 2 - a'l 2 - an

Dimg thut: xay ra chi "hi 111 = «2 = -.- = (1), = lin. 0

Vi du 1.3.'2. Cac so d'ltl1ng a, b, c, d co tang Oinh phuang Mn_q 4. ChUng minh

a'L b2 c2 a2 4

-~~ + + + ------_ ... >-.

b+c+d c+d+a d+a+b u+b+c - 3

Loi CrA]. Kh6ng mAt tfnh t6ng quat ta.gia su a ;::- b::.:: c 2! d, khi d6

a2 ~ b'l ~ ,2 ~ dY,

__ 1__ 1 > 1 1

> >.

b+c+d-c+d+a-d+a+b-a+b+c

36

Clmc::mg 1. BM dAng tillie n} sa

~ 2 2 2( 1 1 1 1)

4. VT .> a. + b j- c + d +. + +

- ( ) b+c+d r.+d+a d+a+b a+b+c

16(a2 + b~ + ("2 + (J2} 4J4(a'2 + b2 + C'l + d2)

>- > . --'

- 3(a + b + c + d) - 3 .

Suy ra di~tl phai chirng minh. Dimg thin: xay fa chi khi tt. = b -= c = d = 1. LJ

Vi d1,l1.3.3 (China MO 1996). Cdc so Ih1/c dwn._q a),U2.·,·.(In co t6nq iJll1l.q l . CMmg mitth h/u rlir:ll,r] tiuu:

(·Ii a2 a, y'7i'i + Jii2 + .. , + va;;

_._--. + _.-- + ." + -- > -- .- --_. -

vt-aI ~ ~ - ,JnT '

La] GJAI Khong m§.t t inh tdng quat ta gia. 511 a) :::: 112 >- ... ;::: an· Khi do

1 1 1

fi""'----::""" > ~ > ... > .rr--::- . V 1 - Ul - vI - «2 - - y 1 - au

Sit dung hAt d~ng thdc Chebustie» cho 2 hQ don di~u tren

. (1 1 1 )

n.vT~(a~+(J.'2+ ... +a,,) v'. + f'1"::- +.,,+ 1·'- ,

1 - (11 V 1 - [/2 \0 I - u"

Theo bit di:lllg thuc Cau.chy - Sdm;un thl

n = n(al + a2 + .. , + a,,) ::: (.,fiil + .;a2 + .. - + Ja~l -'* vial + ~ + , .. + .,;a;;-:::; .;n,

,.---,---~---

~ + ~ + ... +.~:::; Vl1{n- a) - a-~ - ... -a,,)

::; In(n - 1).

Do d6

T([ 2 b&t dAng thuc t.fn ta suy ra

1 n2

VT> -.

- n }n(lI - 1)

D~ng tmrc chi xa.y fa khi t§.t d_ cac bi~n deu bang "ban, 0

1.3. Bit d<ing thll'c Cbebysbev vA l~ng di,mS'

371

Vi du 1.3.4 (Crux). Cia su {~ii(' so tbuc a. b, c > 1 thoa man

1 I 1

--+--+--=1

a2 - 1 /)2 - l c2 - J -

CMtng truuh. riing

1 1 1

--+--+--<1. a+1 b+l c+l-

LO] G L~ I Ta ~n! d ung bat ding tturc Chebyshev kct hop voi phucng phap chirng niinh bang phau clnrng (ban hay xern a chirong IV). 1\ 6i each khac, ta s6 chimg minh rang, ntu cac 56 thuc a, b, c > 1 thai man

1 1 1

--+-~+~-=L a+l &+l ('+1 .

Thl bat ding thuc sau luon dung

1 I 1

--+--+~~>L

a2 - 1 b2 - 1 c2 - 1 .

ThM v;,ty. ti't gia thi6t suy ra

a-2 b-2 c-2 a+l+b+l+c+I=O (*)

BAt dring thuc c~ chtrng minh Wong dlwng v(fi

4 - a2 4 - b2 4 - c'l

~l + b~l + -2--1 ~ 0

a - - c-

a-2 a+2 b-2 b+2 c-2 c+2

¢'> --.-- + --.-- + ----- < 0-

a+la-l b+lb-l c+lc-l-

Cbli. ~, rang neu ta co a ~ b ~ c thl'

a-2 b-2 c-2 -->-->-o.+l-bH-c+l'

a+2 b+Z c_'_2

--< --< --, a - 1 - b- 1 - c - 1 .

Do do ta (hi (An ap dung bat dAng thuc Chebyshev eho 2 be ngiroc chieu noi trcn va chu y di,}u kien (,,0) ta co dicu phal clnrng miuh. DAng Un)'!.: xay ra khi vii chi khi a;;;;;; b = c;;;;;; 2_ 0

Vi du 1.3.5 (CWMO 2005). Cho cdc so lhl!c dU(Jng a, b,c,d,e thoci man

1 1 J 1 1

4+a+4+b+4+c+4+d+4+e=L

Chiing minh nlng

abc d e

--+_____.__,_.__:_+--+--+--<L

4 + 02 4 + b2 4 + c2 4 + cr 4 + eZ -

38

C1Hldng 1. Bin ding thuc co so

Lot GI.~.1. Then gia. thiet ta co

l-a 1-b 1-c I-d l-e -- + ........__,_ + -- + -- + -- ;;;;; O. 4+u 4+b 4+c 4+d 4+e

Ta phai cmrng minh

1 1 1 1 1

--+--4 _ .. +--+-- 4+a 4+/; 4+c 4+d 4+e

abc d e

>~-+--+--+~-+-~

. 4 + a2 4 + b' 4 + c2 4 + d2 4 + e2

I-a 1-b 1-[; l-d

<:=:> (4 +a)(4 + a~) + (4+ b)(4 + b2) + (4+ c)(4 + (.2) + (4 + d)(4 + d2) > O.

Gia. sir ra.ng a 2: b ~ c ;:: d :? to> khi d6 de thay

I-a l-·b l-c l-d l-e ~-<~-<--<--<-- 4+a-4+b-4+c-4+d-4+e.

1 1 1 I 1

-- < --<--<-- < --. 4 + a2 - 4 + b'l - 4 + c2 - 4 + (J2 - 4 + f2

Du uu til chi elm all Ul,1Uf!, LIl.t diillg tIn\:!; Chebuslve» cho 2 L{) don ui('u giau: lll'1l b0 c6 diEhl phai chimg minh. Dang thuc xsy ra khi va chi khi a = b = (: = d .:;.: to "" L 0

V[ du 1.3.6. Cia su cdc s6 th\fr;; duong a, b, c thoa man a + b + c + d = 4. Chl1ng

minh bIlt ddng th{lc ..

1 1 1 1

~-- 1---+ + <-

11 + a2 11 + b2 I I + c2 11 + d2 - 3'

LaI G L.i.l. Bilt ding thuc c6 th~ vih lal duo-i dang tirong duong

1 1 1 III 1 I

11 + a2 - 12 + 11 + b2 - 12 + 1l + c? - 12 + 11 + cF - 12 ~ 0

a2 •. 1 b2 ~ 1 c2 - 1 d'l. - 1

11 + (12 + 11 + 62 + 11 + c2 + II + d2 2 0

a+l b+l c+l d+l

~ (a-l}·a2+11 +{b-l)'b2+11 +(1:-1)'('2+11 +(d-l)·d2+11 ;:::0.

ChCl Y rllng n6u a ~ Q ~ c ~ d thi til luon co

a+l 11+1 c+l d+1

....."..--->~-> >....."..---

a2 + 11 - 02 + 11 - c'l. + 11 - d2 + 11 .

Nen ap dung bAt dang thftc Chebyshev cho hI) s& tren va b¢ (a - l , b - 1, c - 1, d - 1) ta c6 dieu phai chirng minh. Ding thttc xii.y ra khi va chi khi a = b = c = d = 1. 0

Tat: !!,ia. :it- kllOHg uf, t~V llhiEu U~ll llhfnlg Lai toan alJ dung true tie-p bAt uillL~ thuc Chebyshev ma lie di sau hon den mot ki thu.(i.t Tilt hay lien {Juan deu bilt. d~ng thil'c nay, ki thuat ph an tach Chebyshev. Cac ban con g~p li;Li nhfmg ling dung d~c biet bon eiia ki thuat nay trong phan sau eva. Li;Lp sach, phan Cac bai twin sang tao.

1.3. Bih aAllg tll!1"c Chebyshev 0't Ii"llg d~,.mg

39

1.3.2 Ki thu~t phan tach Chebyshev

DJ cac ban de hinh dung ve phuong phap nay, hfl..y xern mot vi du sau day Vi du 1.3.7. Cho cec so th1,£c dudng c, b, e, d thod man di€u kif.n

a + b + c + d;;;;;; 11(~ + ljb + lie + lid.

ChI'tng minh bIlt aang thdc

2(a + b + C + Ii} ~ Ju2 + 3 + Jb2+3 + ~ + Jd2+3.

LtiI G I AL N~u uhm thoang qua ve <10 hill. t.:at.: ban s~ cam thay kho khan vi gi;i thi~,t ella bai roan khong that tub"ng rnlnh. N~ll banda bi.~t nhieu ki t huat chimg minh blit ding thuc thi lai chg de bi nMm khi di t heo nhiIng phirong phap rnanh vi du nhu don bien ch;\ng han. Thitt b~t ngo, bai toan tren ducc giai hoan toan don gian bliug bAt ct;\n~ t !Litc Chp_b!/.~hr~1;, I.t~t rrliit'n voi rtlQt ki tim;;\! flp dung kilfl Q;:ic bir:t .

Tao lru nrong irng tirng sO hang cua 2 vtf va chu y ding thul;

3( 2 1)

2a - J a2 + 3 = a - ( >I; )

2a + .j(i2+3

Khong th~ ltp dung hlit ding thuc Chebyshev ngay dUQ'c VI ca til: w va mA.t1 sb cua bi("'" tillTC (ff\l la ('ae ham don dipl1. t!'tllg Thy nhien h.1} dn', y giil thih

-1 1 I 1

- + - + - + - = a + b + r'+d

abc d

~ (a - ~) + (b - ~) + (e - ~)I (11 - ~) ~ 0

(.12 - 1 b2 - 1 c2 - 1 dl. - l

~ -- + -- + -- + -- = 0,

abc d

Ttl It[\l!!:; thut; nay Lau UI)\; (;() tli· thi\..y IlJ.Qt mui li01l Ii\' voi (.). uv la ehung ttl ~0 a 7. ~ 1. Day chinh ttl chia khoa de: ghi.i bai toan, Bttt dijng link tuong dunng vdi

3(a2 - 1) 3(b2 - 1) 3(c2 - 1) 3(d2 - 1)

-...:..,_-=~=+ + + > 0

20, + JU2'+'3 2b + vb? + 3 2c + leI. + 3 2d + vrJ1 + 3 -

[J,2-1 ~2_1 ~-l ..r-l

~ -c.- + -b- + -c- + -d- ~ 0.

2+Jl+~ 2+JI+~ 2+Jl+~ 2+Jl+~

Lt1U Y bat LHu~ tb((t; Qil cho huuu toan dOi Xi'tll~ voi 4 bien a, b, c, d va cac ham so '2

.1: 1 1

/{x) = ---'-- = J~

x x

I

g(XJ=. ~'

2+Vl+~

40

D€ou 18 car ham tang tron R' Ilion theo b;\t ding (Ink Chebuetrev

f(a)g{a) + J{b),q(b) + f(c),q(c) + J{!l),q(d) ~

1 -

i(f(a) + f(b) + ftc) + f(d)){g(a) + 9(0) + g(c) + ,q(d)) "'" O.

VI ta co f(a) + J(b) + f{t:) + I(d) = O. DA.ng thltc xii)' ra khi a = b = (: = d = 1. U

DiPn d.(i.c hi~t nh;.\t d loi giai nay chinh Ia vi~c (hi" t.:i\. tl'( va mill1 moi phan sb tirong ung cac 56 a, b, c, d tlWII~ ((u!!" sau UU Iw)i tUm); b;it u~ug t lnrc ChfbY5h.CI.' Ncu ta dung true tiep thl khong wang lai hicu qua, C6 the khai quat plnrong phap tren thanh dang t6ng quilt VOl n bi61l

De chtmg minh X1Yl + X2Y2 + -,. + XnYn ~ 0 ta co the chirng minh

YI ,(xp:.q) + !h . (X1;a2) , ". + y" . (;t"an)?: 0,

Ul U2 0 a,.

Trang d6 al, (.12, , a" la esc so tittle rna ta phai tim sao cho

La die bo don di(,u cimg (;hi~1.1 dv di~l\ kien b~t dang thirc Chebyshev duoc tho;; man, Khi uu vivc clnrug II rinli btl! o:Alllj tlI\Jt: ban uAu ~i:' Utrl,K quy vt- dllOlg minh 2

bA.t dAng thuc don gian hem til. .

~ + Y2 + + Yll :::: 0,

al (l2 an

xlal + X2(l2 + '" + Xnan ~ D.

M¢t dang thuong su dung c(la ki thuat nay la tip dung vdi dang phan tlurc Gia ~lr ta pM..i chitn!2; rninh hi\..t dii.ng thirc dang

Xl, ~~ + , .. + x" > 0,

Yl Y2 y" -

W Ii t huyh t hlt::t luC}]! nJ t h4 dira Ld t J ~Ilg th uc ve dang nay, t ham (;!J i (;0 t Ii~ chon Yl, Y2, "., Y2 ~ O. Khi d6 til, rli tim dl.l~ so tlurc aj, a2, ---, a" sao cho

In. cal: b¢ don dieu ngiroc chleu. khi do bat dnng t hitc sk duoc chirng minh neu 2

di~u kien sau Ult!,l'1: tbOR man '

alXI + a';lX2 + , .. + UnX" ~ 0,

1 1 1

-+-+ .. ,+-->0_

al PI (l2!12 a'IY" -

41

l'Mng t huong u(li vdi <"<I.e h~t d,\ng ! IHk trong 2 cliiy sf) (x I , ;r.~, "., :[,,) va (Yl, Y. - - -, )f,,) !';{- co rm)t dily ducng ncn khi chou day ((11, o.'J, __ ., Un) dinmg thi chi can clnrng minb mQt trong 2 bli.t c1&ng thirc 1..:11«)1, Trong nhieu tntong hop tu 1:6 (hrQ"c dling th ((0:: a I x I + fl21.'2 I .. - + anI n = 0 nhir trong bai t oau tren,

Tuy nhien vii:;'e chon diroc hI) so nhan ell' (ai, a2, "., Un) kh6ng tb~ may moe va vi th&. vhttul!fi, phAV nay l!ui khi dlll uoi hui :-;'-1 kht"o 100 va t inh S'- V{ du sau u~IY rninh hoa kha r6 ki thul;it chimg minh nay

Vi du 1.3.8 (Crux). Cho nk ,,,6 dUCfn.Q It. 0, C co td119 bclng 3, Ch/(ng minh MI dang

tMtc sau.

1 1 1 3

--+--+--<~. 9-ab 9-bc 9-ca-S

L", GIAI. Dat x = be, y = ca. ;; == ab, Ta phai chimg minh

1 1 1 3

--+--+--<-

9 - x 9 - n 9 - z -, 8

1-:r 1-1,1 1-" (}

~---i- __ + __ > ,

9 - x . 9 - :11 9 -- z --

D~n day til phai tun cac :>6 uJ'1 ay, no; tuong un!; d6 nhaIl vai ell. t if va. mau cac phan Si) I,rell Ta lay a,,, == 6 + z , <l'J = 6 + 1/, a~ = 6 + z

(1-x)(6+;r) + (l-y)(6+yj + (1 ~z)(6_+z) >0.

(9 - x)(6 + J;) (9 - y}(6 + y) (9- z)(6 + z) -

Kh6ng mat tfnh t6ng quat ta gilt sit a ~ b ~ c => z 2:: y ~ x. VI 0. + b + c == 3 nen z + y = a(b + c) -s 9/4, Z + x ~ 9/4, z + y "$ 9/4. Ngoai ra, de thay

(1- x)(6+.T)::: (1 - y){6 + y) ~ (1- z)(6+ z ),

(9 - ~r:)(6 + x) :::; (1 - !I){6 + y) ~ (9 - z)(6 + z),

(1-:r)(6+x). (l-Y)(6+y) (1-z){6+z)

-=--~=-------:- ~ + -------:-;-:;:-------;-

(9 - x)(6 + :r} , (9 - y){6 + y) (9 -- z){6 + z )

~ (~(I - x)(6 + X)) (E (9- .r)I(6 + X))

CuBi cnng. ta (:h; (,:A.n chung rninh btu doing there sau

L (1 - ;;;)(6 + .r ) == 18 - ;;(~ -I- !J + z) - (x2 + r/ + z2) ;::: ()

~ 5(ab + be: + ca) + (a'.!b~ + b2c2 + ,,2aZ) $ 18

{"o} 5(ab + be + cal + (ab + be + r:a)2 :s.; 18 + Babe,

42

G'JH1(1ng 1. BAt dAng ti: uc co sci

S11 dung hal dang thirc quen thuoc

(a + b - (')(b+ c - a)(c::+ a - b):O:; abc,*,*, {3 - 2a)(3 - 2b)(3 - 2c)::; xy;; ¢} 9 + 3abc ;:::: 4(ab + be + ca)-

Thay 3abc ~ 4(ab + be + ca) - 9 vao bat ding thirc tre-n. ta. pM.i chimg minh

5(ab + be + cal + (ab + be + ca)2 ::;; 8(ab + be + cal

~ ab + be + ea ~ 3.

Bit dii.ng thuc tren hi~n nhien dung VI a + b + c = 3, Ding thuc xii.y ra khi va chi khi a = b = c = 1. 0

vr du 1.3.9. Chllng mini: Mt dang thuc 111

-:-;--- + + < 1

c'l. + a + b aZ + b + c b'} + a + C ~

Wi cac 5J th'l/c khOng am G, b, c tuy 11 co t6ng bJng 3.

Lot OrAI- Hay ch6 ). phan tfch sau

ala - I) h{b - 1) c{c - 1)

---,,--''--____;'- + - t > 0

().z _ a + 3 b'l - b + 3 c2 - c -I 3 -

a-I b-l c-l

¢ 3 + ~. 3 3 ~ 0,

a-l+- b-l+-c-l+ abc

G ia su a =:: b '2: c ~ a ~ 1 2': b - 1 ~ c - 1 . Vl a + b + c = 3 Iltn ab, be, cu -::; 3, Do 06

1 1 I

-------.,.-3 ~ 3 ~ -- 3'

a-1+- b-l+- c-l+-

It b c

Ap dung bAt d~ng Ih(tc Chebyshe.v cho 2 UflY tren ta \.'0 dj~u ph a! chung rniuh. Ding tlurc xi!ly fil- khi va chi khi a '"'- b = c -r- L 0

"

Vi du 1.3.10. Chang nunli lxit diing !hUe sau 1J(h a, b, c IiI ccie sa thl!~ kh6ng am

cho trudc

1,3. BAt dAug t.JH~C Ch€~yshe\! vi< Ung (l\lIIg

43

Lor CJA!. Ta tif chirng minh hat clang thuc t,6ng quat hun v0i ITlQi k < 2

~-k ~-ro 2-~

62 + c2 + ka1 + [2 + f)2 + kb2 + a2 + b~ + kc2 ? 0,

That vay, nhan them cac h(' s& urong UU!'\ vao moi phan ,~b

(01 - I1r)(b + c) (b~ - ro.)(c + a) (c:l - ab)(a + bJ

----=-_:__ _ ____::..:...~--- + + . > 0_

{b"l + ("2 -+- ka2l(b + c) (c2 + a2 + kl}2){C+ a) (n2 + b2 + k('2}(u -+- b) -

Khong mh.t nnh t6ng quat, gia sir (1 2': b, khi d6

(a2 _ bc)(b + c) -- (b2 - ca)(c + a) "" (ab + (;'2)(11 - b) + c{a2 - b2) ;;:: 0 (b2 +·c~ + ka2}(b -r- r) - (c1 + a2 + kb2)(c + a)

,--, (b - c) ((12 -+- b2 + c2 - (k - 1)( ab + be + ca}) ::; 0, Vf).y 2 bo t\6 san don dieu nguoe chitlu

(a2 _ bc)(b + c) , (b2 - ca)(c + a) I (cz - ab)(a + b)

(b2 + ('. + krL2)(b + c) , (c2 + a2 + kb2){c + a} , {a2 + b2 + k(2)(a + b), Chu Y ding

l\'"~n tl'r bfLt dAng thuc Chebyshev ta co dieu phai chimg rninh, Ngcai ra k = 2 cimg 11:1 hang sO tot nh<'it d~ bAt dAng dn'ic tren dung. 0

Nhan xet. C6 mOt each chirng rninh khac cho bat dAng thirc tren bang each si'( dung bflt ditD!l; tIll\:!: Cmu:hy - Sctuuar z

(L2 b'J. (4+ b) '1

-- + -- > ~'------~'------

a2 -1- (.:2 b2 + ("2 - (12 + b2 + 2c2 '

b"l c2 (b + c)2

-- + -- > ~_:____._~;__...,..

b2 + a2 c;? + a2 - b2 -+- c2 .~ 2a2 '

cZ a'l (t' + (L)2

-~ + -- > -- -----'-----ci + bi (.12 + b2 - c2 + a'2 + 2b2 '

Chi can cong theo tUng vt: die bti.t dAng tlurc tren ta c6 di~u phi1i clurng rninh. 0

Day lit. mOt chung minh rlu hay vii: doc dao, tuy nhien chi sir dung dl1~C trong tntbng hop k = 2. Ta cung c6 tilt dung phuong phap phan tich blnh phimng S,O_S,

Vi du 1.3.11. ChUng minli 1J8i ml'-i a, b, c khOng am

in" I 8/:w+ Jb2 + 8ea + vc') + Sab S" 3(a+ b i (").

44

HUONG DAN. T1I co bii-n d6i sau day

r-2--sb- 8(02 - b(~)

3a - v a + c = ~.....:....--;=;;<====:= 3a + .ja2 + Sbc

8(0.2 - bc)(b + L")

(3a + ,; a2 + 8bc)(b + c)

Sau d6 s-j'( dung hAt ding thite Chebysile11, chii y dmg t6ng cac hl's6 htmg Il. U K~t thuc cho hai vi;}t CA.C ban hay thu: chung minh mot bat dbng thuc manh hun d~[J!'; thin: Chebyshev thirong g(l.p. Chung minh chi li6t se co a bai sau, trong phan khai tri6n Abel.

Vi du 1.3.12. Gia su C!IC 86 tauc U]. (J_'l, •.. , a., thaa man dieu kien

ChUng minh. riing

1.4 Bitt ding thuc vdi ham 15i

Ham h~i Iil. nujt kliai niem quan trong troop, toan hor. Li rhuyet ham loi d6i voi cue bM J~n!'; tlurc 111. mot Ii thuyet rat 1"ii1.u va rong, rnang nang tinh giai t ich, nen tAt giii.. I.:tUlg chi co gang neu ra nh((ng vAn M Cil.lL t.1Mt, quan trong uhit dO; vdi bAt c1~llg t hirc, V{)i cac han THCS, ham loi van con la mot khai nihn kha xa 111.. nhimg Li~l· ban ~c· kllOll[!, I)h,i,i II} lill!!. vi dii.~' Ii:~ lIl(it kliai Hi~~111 ttwug ,1t,j d~ hil-u vi:I lJ~~I) h:IY tilt dOu till! iip d~mg no t ir bay gid Troug phan nay se l~6 2 muc nho : nat (!im!!: tlnl'C Jensen va bat dang tln:tc ham It,i v{Ji viec xet car: phan t if (; bien.

1.4,1 Ham 16i voi bAt di..ng thitc Jensen

D !nh 15' 1. 5 (nat d~ng th uc J en~en). N (11 f la ham h5i trf!n khoiin_q / thi vUi moi Xl, .i:2, ... , XIl Ella df-u co

() f() ). (:Cl+X2+ ... +X,,)

f·T\ + X2 +, .. +J(x" ;:::nf n .

45

Chung til. van quen vdi vi\'c col bam Ibi f : I _, R li'l ham Iif:n t uc, kha vi ~Ap 2 va r(x) ~ 0 "/;t E' [fl. Tuy uhicn voi ki611 tluk THCS thl ujuh Ii .l eusen. co thf pilat bi~u d,(ni dang dan giilll ViL d~ tip dung hon,

H C + ' ) (x+y)

¢ qua 1.7. .ho f: Ie R ,R [hOI! man f(;o; + f(:,,) ~ 2f ~2~·

Khi d{j lIdi m{)j .ri , ,T2 •.. , .r" Ella c6 bill dring th·de

Vx,:,/E'I.

CHUNG Mrxu. Chung minh dillh Ii tren ta di:mg phmmg phap quy nap Cauclur. Dinh li tl"0n gitql cho viec kiC'llI tr iI btlt U~\\g, 1.IHj(; J e{/,sr·" thuan tien hon !leu ban kh6nl;, hiet vI: d<,1C) ham, Sau day I;:' mot chung rninh ng1'in gQn eho dinh Ii .

De thay. bat d6..ng thuc tren hieu nhien dung I)~U II b. mot luy thira ciia 2 va ncu bat ding thuc dung vdi n == k + 1, til. liy x = Xl + :1:2 ....... + XI.; va Xk+ I = ~ thl

X (X+i!:) X

!(Xl) + /(.7:2) + ... + f(a:ki +! (~) ~ {k + l)_f 'k -+- ~ = {k + 1)f C~:)

Do do t.a di di~u phai clurng minh, 0

(:r + y)

Hj~n nhien n£;u thay di~tI kien f(x) I- f(y) ;::- 21 -2-'

f ( x ; Y_) t III bat ding thU'l: t6ng quat cilng d6i chi~u

bdi f(x) + I(!)) <

I( 1 f(,rl+,T2+."+:C,,)

:tJl + f(;r~1 + ... + fCc,,) ::; 11 11 .

B!i.t dAng th(te Jensen ld\{lng phii 18. mot bat ding thul; qua chat, ulnrng xet mot each t6ng quat. no la cong cu rat d.n thiet va rilt dr ap dung, Day 13. bAt ding U11k rna kill ban mni lam fJlwn VI'; bat dAn~ t.ln·t(~ till rAt rAn pldll ghi nhrl T(\n~ quat han, ta co b;_\t ditng thuc Jensen voi trong so

Dlnh Ii' 1.6 (Bat dilng thac Jensen suy r<)ng). Ne..I.{jl,tl2 •.... QIl la cae.5(1 th1/C khbnq am co t/in.11 bhng 1 1'( r , r I . ,1"2 ..... f" III nie 1>8 th tIc tU)/ 1; IAi 11di mq1. /Ut711 lr1i f triin lap ,0;6 Ihw· fa lutn: co

\nl nlnr the bat d~ng thuc Jensen diroc trlnh bay d tren 1i:L m<,H trutrng hc;fP ri~n~ vdi nl = a2 = ... = lin = lIn. Cung wang tu nlnr tnroc, neu ban chua. bi6t nhieu vc cac kien tlnrc 6·!<'. hilm 15i hay dao ham, thi bi1.y glli nho t.inh chA.t sau day

46

H~ qua L8. Vdi cae so a"x; (i,,-; 1, n) tiiod man dieu ki~n tren, f lit m(jt ham xac d~nh t.,.tfl tap so th·!!!:. X tit bat dang tlnk

NlnJ v4y 11(; !;:!(lJn t ra I inl: k'ii ella f t.a .. hi (-An kif-m tra trong t.rlton!,; hop II - 2.

H';: qua 1.9. Cia EU f ; I_, R. XiI bU.1 rliing thiir:

Chuw) mi"llh rli"llfj Mt IJr1ny. flnlc tdn dung 11m tum: n va vai moi sci tlu«: dUring ;CI,X2, ... , In E I khi va chi khl no dung voi n = 2va vd'i moi £1,.1:2 E I.

CHl::'NG MINH. Day co the coi la OiN· dang phat bi~u khac cua bat dang thirc Jensen. Til. cimg sic dung plnrong phap chiing minh tl.tdng tl.,f, do Iii. phunng phap quy nap Cauchy. t~fich d)lmg: minh sau cHiy Cl-U)~ hoan t oan g:iAng: clnrng rninli bat dAu!!; tIn)" Akf - G M a phan tnroc.

Dieu kien dm ),\ hien nhieu, vi bolt Jilng thuc nfll dung voi n tuy )< vdi moi Xl, X2, ... , X" E J tiki cung dung voi n ;;;;;; 2\'1\ voi moi XI, X2 E' I.

DiCu k!en <Ill phai chUng minh kho han. ta slr dung quy nap.

Gill. SIr bat d3ng tlurc cia d~lng vtri U, nhu th~ hi~n nhien no s~ dung vOi 2n so hang, ta dli dn ap dung !!iii'\. thil~t qnv ni.!]) va ~i{~ th]ft bf" (tiU!l; tln'tl· dJ'mg voi n = 2. Nlur v,1y h,'a dhng t lurc ,h:lllg v0i IlLQ! ~-.o n <:0 d<;tng: 2", kEN.

Gia sl'r b~t dAng t.il{(1' (h1ng vOl moi n + 1 s6 ,1:1, ;1:""2,., Xu+l, ta c6

f(xll + f{X2) + ... + f(X'L) + j(;r) ? {n + l)/(x) =} I(T]) + f(:r.2l + .. + f(:.r,,) ~ nf(·T}

Di~u kii;n (ill (.ht(,1c chung minh xong thee plnrong phap quy 1l4P Ba: toan dii lhtl)c hoan thanh, 0

H~ qua LID,

a. Khdng dlnh if he qua 1.1 van dunq niu ta lhay lne'u !Me trung binh nhiltl "ling m9t bilu Ihue b1J.1!il binh. Viii ki .Qiilr! car bif.7I :r 1, :f2, ... , ;1:" chdng h(J,1! nh 1( Inmy /!in!! ron.9. truru; bin.h difu hurl. trunq blah nhan ...

47

b, N€u Mt dang thUe vdl 2 ,~t; d6i chiiu lM btl! ddng ih'ue udi 11. sJ d6i chieu.

CHlJ:-'-G r. ... INI!. Chung minh hoan toan tircng t~t nhu h~ qui\. L9. Vdi call (b) thirc ra ta oil xet vdi ham - f{.r), 0

H~ qua 1.11 (Bat d~ng thuc AM-GM). Viii m9i.')0 th1(C d1tdng (11, (12, _ -, (111 fa /uon co bitt ddng thaI.

CB(ING MINH. XCi f(x) = x. hien nhien bat dhg thirc dung vdi 11. == 2 nell thco h(· qn,i lIJ, no ("1-111!-', d,"nlj.', voi flloi Sl~ nr nhien n va voi rnoi day 56 time dirong i.l.1,([2".,.a". 0

H~ qua 1.12. ChUng minh vai moi s6 dl(dng .r I, £2, __ . , :r .. thl

1 1 1 112

-+-+ ... +->------Xl ,1:"2 x,~ - xI -+ .1:2 + .. , + Xu

CHliNG ~dINH. DAt f(x) = I/x, de th~y ham f(x) thoa man /(x) + /(y) ~ 21 (x;y).

Theo h~ qua 1,7, bAt drmg thuc dUng vai.Vn va rnoi day s6 ducng Xl,X2 •. ,.,x,... 0 H~ qua 1.13. Ch1Jnq minh ning vrn moi day 56 d'IJdng XI, X2, - - -, Xn thl

XI + X·l + __ . + I" -s In(x? +x~ + , __ +:r~),

CH (;:-;r; ;"·11 N H. Sit dung b~ qua L HI. chi can xct voi ham j{,T) = Vi- {J

San dii.y If! mot :>6 vi du nan gi;m dlil. bAt dimf; tluk Jensen. Thong thuong cac bai to an dang nay kh6ng qua kh6, nhirng din thift clio cac han ciing co Ii thuyet:

Vi d\ll.4.1 (1M 0 Shortlist). Ch ,m_q minh riJ.ng vai moi day .~6 Onle x I, £2, .. " x.~ ~ 1 lu. c6 Mt adng thur

1 1 1 n

-- + -- + .. , + -- < -;-----:-----::-;=== 1 + Xl l + X2 l + x, - l + \I<1:"I;.(:2-·X,~

LeI! G [AI. Then ItQ qua 1.9, bat dAng t:h~k F;C d(l[}!l; m;ll ta clnrng minh dl(Q"C va! 2 so

1 1· 2

-- + -- < -- Ii(l b > L

I + a2 1 + b2 - I + ab '-

Thy nnien bAt dimg thirc tren tuong uu-o.ng vOi (f! - bP(1 - ab) ~ 0 . hi~n nhicn dung, Dimg thuc xay ra khi va chi khi al = a2 = ... = a,~. 0

48

Cll!1'C/ug 1. B~I. clAllg thue n1 sO'

Vi d u 1.4,2. Chung miub rdnq 1101 17!(H d 1 • (L~l •...• a" La cac s6 ttiuc trong kho(llI!j ( 1/2, 11 til iuon c6

Llli GrAI. X6t ham so san voi x E (1/2, II.

/(.1:) ~ In x - !Il( 1 - x)

'{) t 1

=*1,1;:=:=-+--

.r 1 - .r

-1 1

~ t'{x) -'" ~ + (1 _ )"2 ~ I) (:l: E (lj2, 1]).

:r: :r

V6-y f ia ham 16i va theo bat doing that Jensen ta co di~u phai chimg minh

() (al+a'1+.,,+a,,)

!at + !(a2)+ ... +!(un);:::nj n .0

Vi du 1.4.3 (India 1995). Gia SIt oie so thvc dUcJn.1} .r I, XZ, ". In CO tdng bdng l. Chl"tn.Q mink

Xl x! X" ~

~+ ~+ ... + ~> --.

V 1 - .r I V 1 - .1:2 V I - Xn ~ n - I

Lei G! AI. Khong mfLY khri khan ta ch ung rni nh dlt('c f (x) ia ham toi voi

x

J(x) =.;r=x :r E (0,1).

VA do d6 ap dung bat ding thuc Jensen ta <:6 dietl phal chimg minh, 0

Sau dAy la. mOt tinh chat rAt. dcp ella ham loi, ban doc hay thir clnrng rninh trudc khi xern ticp phan sau.

Vi du 1.4.4 (B§t dAng t lurc Popoviefu). ChrfTl!; miuh. r/i:n..fl 1!f" f 10. ham .~6 !Oi trfn I thi. !)(li moi a, b, c E I

Sau ullg dung cua !Jim) loi u0 clnrng niinh bat ding tlurc dang churin (We Ih dil.ng tlnrc xay ra khi t~t ca n bi6n b~ng nhau) tao cung co mot irng dung rAt quan trong khac ella ham IOi. D6 Ii viec xii Ii bit ding tlurc voi cac phan tu (; bien.

1.4. BAt dlllig th uc voi holm /Oi

49

1. 4.2 Ham Ibi vdi kf t hu~t xet phiin t u {j bien

Hay bitt dau bAng bal tv;ill san day

Vi du 1.4.5 (Tap chi Twin hoc va 1\16i td:). Cho oie sa ttcuc a. b. c E [1. 21· CMtng mint: iJlJ! dang thlk

Lot CIA!. T'nruc khi sir dung tinh <;hAt manh cua ham l&i. ta !if' ul tun mot !oi giai thi)J sd ell.jJ dIU 'bAt dim!,; UI\'rc t.ruu. Dati 1 ion ta phai ~ia sii' a :;:. b '2 c. Vi a. b, c E ;1. 2111Cn d0 tha~-

a:1 +'~ :::; 50. ¢'} (r< - 2)(u2 + 2(t -- 1) ::; 0 (1)

5(~ .j.. b3 $ 5ab + I ~ (b -- 1)(1/( + b + 1 - 5a) :::; 0 (2)

5ab +c3::; 5abc+ 1 ¢} (c - 1){c2 + C 11 - 5ab):::; 0 (3)

C:k khftug d\nh tren hi0n nhWII ilI"mg vi.

b2 + b + 1 :::; a~ + a -!- 1 ::; 2u + a + 1 'So 5a, c2 + c + I 'So 0.2 + a + I ::;- 5a :::; 5(lb

Cong v6 3 bAt da-Ilg thuc (l}, (2) .:a (3) tel. ni di~u phti.i chimg minh. D&ng thuc x';y ra khi va chi khi a = 2, b = c = 1 0

Tit,p thoo (,llll?; Iii met vi du I'll t t hri vi trong vi(j<' rlanh p;i;\. I rung gran - ki t 11I!~l.t quen thuoc <;ila cac bat tllmg tha~~ 1)(1 cap.

vr du 1.4.6 (Olympiad 30-4). Ch{tng minh rJ:ng w:h ok .~IJ thuc a, b, c E [L 2]1.lJ. co Mt ddng IhUc

(1 I 1)

(a + b f- c) - + - + - 'S 10. a b r

LOI CIAI. Bat dAng Uuk timng du:ong vdi

a Ii c b c a

- + - + - + - + - + - < 7. b c a a b c-

Khong mAt tfnh tong quat ta gia sir a :::: b :::: c_ Khi do

1 a a b

((J - b) {Ii - c) ? 0 .;=> ab + be ;::0: b -i- w: 0:=;, - + I 2: -, +-

C J C

2 C c b

ab + be > b + ac G - + 1 > - + -.

- a -/} a

( 1 I 1 )

(al + a2 +." + an) ~ + - + ... +-

al 0.2 a"

< n2 + k (p - q)2

- 'I 4pq '.

50

Chuang 1. Bat dAng tblk cd !)(J

Do do

alb + blc + c/b + bt a :s ale + cia + 2,

abc b r rt (Q. C)

~'-+-+-+-+-+-<2+2 -+-

b (' a a /) c ': C a

D~~t x = ale, t.a co 2 ;::- .T ;:: I ncn (x - 2)(1' - 1) ::; 0 =e- x + l/x:::; fJ/2.

T(( do ta co dpcm. DAng thirc xay ra kbi va chi khi a = b :;;;;; 2, C "-' 1 hose a = 2, b = c = 1 hoac cac hoan vi nrong ling cua cluing. [J

Cil 2 I~i giiii UI((,11: trlnh hay d tren cieu kha kheo 1&0 t rong cac danh gia tnmg gian, Chung mirth hoan toan ph.1 hop veri trlnh ,ttl THCS vA lioan toan ('0 tlil~ t~n)l; quat diroc, Chimg h':l.ll vrri (bub gl,i trlcmg tl_( vi du l.4.G tu (;0 the t6rtg qu-it thanh

Vi' du 1.4.7. ChQ eric so thw a I . a2, __ , an E [p, ql (p, q 2 0), C}l1ln_Q minh

Cach danh gia trung gian nhir tren hoan toau tit nhian vii. don gian. ban hay SlC dung each nay df chung rninh bat dang thtrc 1.4,7. day Ia each so cap va hay nh&t_

Nhirng li';u phmmg phap d6 COn;'iii" dunglai dUQC trong bai toan sau hay khong?

Vi d1J. l.4.S. Cia sit xI, 2'2, ... , X2005 10. cac ,~6 th1,lc trong [-1,11. Hay am gia in nho nhdt 1Jfi !On nhfd run bie'u fMc

Lor CIA!. Khi nhln vso biit id!ng thlct: nay, ta cung 1.:6 "cam giac" la hi~u thuc s€d~t CI_Ir rbi khi cn 2005 sO bang l (df:), biGH thl.'lc sf: dar Cl)"C t.i~ll khi cac sO -1 vii. 1 xcn ke nhau trong da.y x I, X2, "., X2005 va co v~ day lit !n(}t dieu kha hl€n nhien. Thy nhieu, chirng minh uUQl: nhan xet hiJn nhien ay lai khcng don gian nhu ve ngoM cua hai toan, ::;i? rfu kho chirng minh n~u ban khong co mOt dinh hinrng t u trUUI.:_ Hay chu Y esc bilu thu:\; b~ uhat. I(;,:) ;;;;;; ax +b hoac bitu tlnrc bac hai J{x) = x'2 +cz + b vai x E [p, q] c6 mot tfnh chAt nit quan trong iii.

max I(x) =,max{J(p), f(q)} , xElp·q]

Vdi ham b~c nhat no con c6 them t.inh r.hAt

min J(x) ;;;;;; min{f{p),f(q)}, ;~Elp.ql

51

Su dung t inh chAt tren van hai toan ban da.u, chi can mQt nhan X!'t dun giiin la coi IItHr hili tuan ,-U·, UWl: g};ii quyet xling : Ntu ta ed dinh ttit cci elk bit" .1:2, :r:l, .,., X2005 1Ia cho xl b-itr: thien trow! [ - I, II Ihi P = P( X I) 1d mot bam b(tc nMt doi vai x j nen

max P(xd= max{P(-l},P{l)}, min P(:rd-=min{P{-l),P(l)} .

.(OJ'" I-l.l] xl€I-I,q

Va uu uu P U:;Lt max boac min khi vCli mot so Xk bat ki ta de.u co Xk E {-I, I}. Noi tom I~, P lAy gia tri nho nhat hoac ian nhiit khi XI<: E: {-I, 1} Vk -' 1,2005. 0

Til each chimg minh nay ta c6 thf thay thE bi6u thl'tc P Mng d.t nhieu bi6u tlnrc tlwng tl,t khac nhu

VI d 1.1- 1.4.9. Cia 5U Xl, T:;J, "., :r.c la (:lie so Ot1!r. trono [-1, II. H dy ti.m girl tri uho nMt va frln nMI CUll b-i€tl thUe

Ta co the khat quat phuong phap vita sir dung thanh mot dinh If t6ng quat la nO. dfl I (BM toan ve phin tli cue biM).

(i), Cho cae sa tAW Xl, ,1'2, ,." x., E I "" [a, b] va f ; I" ..... R La ham th\lc thoii man viii mQi i = 1, n, n~?1. t.(J djTlh n -1 bif.n x j (J oJ- i) thi f do-I qi(i tri llYn nhii.t khi Xi E {a, b} , Khi do f(XI, X2, ... , Xn) SC liiy giii try. tOn nMt khi lItl chi khi :<, E {11, b} V-i = l,I1.

(iiJ. s« qtui !"tidng tv neu ta thllY ,Qia try ldn nhat thilnh gia try nho Tl,hGt,

Cau Iil.Ji ui;l-t ra Iii. IllaIHg hiul! ~U uao till (;0 tilth I,:hiit eire LiEH nlur vf!.y': T!'tt nhien 117. khong phai moi ham so, chi ham bac nhiit rnoi co ca tinh chat chan trcn Ian chan dlItii. Dieu d~c bi~t la hoa ra kip cac ham nhu vay l~i chinh Iii lop cac ham 10i, 1:\1 the Iii Idp cac ham kha vi cap 2 va co dao ham khong am

B6 de 2. Nill f(x) la ham IOi tri'n [a, hi (r(x) ~ Olix E [a, bl) tid 't;I;r E [a, bl . f{:r) S max{J(a), f(bl},

Neu f(x) iii ham lsm lhi ta thu.y max b&i min.

TAt nhien plnmng phap trnn vitI! d!1"e sCt dung tJ\.(r~ t.i(~p nhrr bAt dAng t.b\i(~ Jensen. rna kh6ng .;an dung Wi dao ham. Su dung dinh Ii v@. phan ham loi va cae phan tit cue bien thl cac vi du vira trinh 'b~y a phan trinrc trCi nen rlit den gian,

Ch~ng han a VI du 1.4,7 ta co bi6u tMc

52

De tht..y vdi rnl}i .aj thi f la mot ham loi nen hifu thrrc s€ dat gill trj Ibn nha,t nell va. chi neu (.Ii E {p, q} \;/i = 1, 1t.

Coug vi';r ('1,6i dlll~ clUl liiru 1-" dAt j' la s() nil sf) (I, h~IIJo'; ]J roi dung tam t lnrc b~t 2. Oietl nav khoug kho nell ban (,:0 t.1I? tIj lam, Till nhien. phunng phiLp nily ~ gilip tao giai nhirng bal t08.1J c6ng kf~lIh hou. chang han:

Vi du 1.4.10 (TBng quat). Tim giu tri lrm nhal fua bdu th1)r;: sa1.i V01 Xi E [a, b] Vi = L 1~ tmng do b ~ a :::- 0

(Jl fh .1,,)

I·u,,:r,,) -+--!- ... +- ,

:1:1 Xl. I"

Tmng do n , Ii, C'I;, 3; Vi = r:-n Iil cac h/'i.n_q .~o duung cho iruac.

vt· Ii thuy(.t, bai toan tren luon giai c:tw:;c vi t.a c:hi phii xet rrong tnrong hop hiru han x, E {a, b} (Xj chi lAy 1 troug 2 giti. tri a, b voi rnoi i == 1,7':).

Vi du 1 A .11 (Tap chi Than hoc va Tu6i tre). ChI! cae so th1jC ;r I , X2, . -. ;1; .. E (a, b) _ Tim gid. In {fin nlult cuu

2 2 2 2 ~

{Xl - X2) + {Xl - x~ll + ' - + (Xl - x,,) 1- (:r:!-· XJ) + ... + (X,,-l - x,,) -

Trong hal toan trcn, VI xl. la mot bam 16i nell ta chi ctm tim gia tri Ion nha.t cua bi~u thirc khi 2', E {a. b}.

Sau diiy I~ mot VI du khac dira thoo ht t1tdng ham 101

Vi du 1.4.12. TIm ,qid tri n.hQ nhti.t cua biiu th11'c sau vm TI E N

f{x) = ~1 + xl + 12 + xl + .. - + jnl z], x E R

i.or CIAI. Dat h. = [-k, -k + I]Vk ~ 2'/"+1 = {-x, -n],I1 ::-:- [-I, +oc). Chri y rAng vci moi x E he ta co th(. hi) di d~c da.ll gla tr] t uyet doi cua f(x) va /(x) tnJ thanh mot ham b~c nhat vai x ncn de thii.y

minf{_T} "" min{J(-k),J(-k+ 1)}

:z:EI.

VCiI II va 1'1+: TO rang ta khong phai ~rt vdi cac can v6 cuug V'I hi~!I nhicn khi do / khong d~t giil tri nho nht.t. Vi;l.y f chi cl~~t g;ia tr] nll(i nllit khi ~-a chi khi a: E {-I, - 2, ... , -n}. Phan con 1000i cua hal to an la vice tinh toau v{ji moi :1: ;;;;;; -k (k :::: 1, n) sau do so sanh nen ::i-: gilmh cho ban doc nr hoan chinh, 0

15. Kho! tridl! Abel vh hAt dAJlg th!k hoA!! vi

53

1.5 Khai triiin Abel va bitt d£ng thuc hoan VI

1.5.1 Khai trj~n Abel

Khai tri~n Abel 13 c6ng thiro khai trien sau day

Djnh ly 1. 7 (Khai triE!n Abel). Gia su Xl, I'l, .. , , Xu va VI, Y2, , .. , Yn ia cec s6 th'!!c tuy y. Di)t Ck = Yl + 1/2 + .,. + Yk 'tk ;;;; I, n, Kh: dc:l

XLIiI + X2!/2 + ... + X"Yn = (XI - X2)Cl + (Xi - X3)C2 + .. + (Xn-J - X .. )c,,-l + XnCn.

Coug tbuc trcn lit kha hi6n nhien, vice chUng miuh khfing co gi kho khan vi no chi don t.huan 11'1 dIDlg tln'(c_ That ki Iii la cong thl~C khai trien flay lai rt..t thuong xuysn dtrl)c !.Hmg trong roan hoc, dae bii;t HI trong dai ::;6 va bfit d.i\.ng thirc. Cong tllur kllA.J tri(~I1 tron !?,lin ih'in vai bit. dilnp; thirc Abel nhu sau

Vi dl,l 1.5.1. Cho 2 dii,y so thuc XI, X2, .. " Xn Va:\ll ~ Y2 ~ '" ::::: Yn. Ta d(u S~'=rl +XZ_+,,,+Xk,

M ;;;;; max S~ , m;;:: min SI<.

k = r:;:! k= Wi

ChUng minh Mt ddng thUc

mYl :$ XIYl + X2'Y2 + , .. + XnYn :$ MY1.

Lor CIAI. Ta su- dung khai trien Allel nhu da. trinh bay (j tren

xIYI + X2Y2 + ... + X"Y'I = (VI - Y2}Sl + (:<,12 - YJ)S2 -+- q.+

+ (Yn-I - y" )8'1-1 + ynSIl > (:1,11 - Y2)m + (Y2 - Y3)m + .. +

+ (Y't-l - y,,)m + Y;,m

> mYI·

Chung minh hoan to an tucng til d6i vdi bat cliug thuc ben pMi. 0

Sau oily li>. mOt $0 bAt <ling th{tc tieu bieu ::;u dung kh .. i tri.€n quan trong nay Vi du 1.5.2. Cho 2 day so dUdn9 ai, a2, .. , rlJ] 1)a hi. b2, , .. , bll thoa man dieu kifn

01 ~ b2 ~ .• ;:: b ... al ;:: bl

rl)!12 ~ blb2

54

ChUng minh. Mit ddng lhUe

L(:1I CIAI. Sit dung khai triim Abel ta co

Theo bAt ormg tlui'e AM - G M thl

Ngoai ra ti:r gia tbi6t Ol ;::: b2 ;:;: ••• ~ bn nen moi lib hang ij t61lg tren UC-ll khong 3m, V~y bat d~ng thlk duqc chung minh, Dang I lute xay ra chi khi (l.j "" bNi = 1, n, a

Vi du 1.5.3 (USA MO 1994)~ Cho cdc 56 t},'i,tc dWfng xl, X2, .. , 1:'1 Ihoa man

Vk 5 n, Xl + x2 + ... + XI; ?: Jk.

ChUng minh bc1t dang thUc sau

2 2 '2 1( 1 1 1)

xl + x2 + ." + III ~ 4" 1 + '2 + "3 + ... +;i" ,

LCI GIAI. Th c6 th~ sAp xtp lai day voi Xl ~ Xl ~ , .. ?:: :r,l· DM hI: = .fk. tiu dung plu!:p uifou JOi Abel

.1:~ + ::r~ + ... + x~ ='- (XI - ;r"llxl + {.'t2 - X;!l{,fl + :r2} + +

+ (X;,-1 - X,,)(.Tl + X2 + + X,,-.l) + In(XI + Xl + + Xii)

~ (XI - I:<) + b2(X2 - X3) + + b;l(Xll-1 ~ xn) + b,,~

~ biLl + {b2 - 01)X2 + (b3 - b2)X3 + ... + (bTl - bl1-l)xn ? {2bl - b2)xl + (202 - b1 - b3)(xl + I2) + .. +

+ (20,,_1 - bn-2 - b'I)(XI+ X2 + ,,, + ;1:11-.) + (b"-i - bn){Tl + ... + x,,) ~ (2b1 - ~) + ~(2b'2 ~ b1 - OJ) + .,.+

+bl1~I(2b .. -l - bn-2 - b;,) +b,,(bll-l- bn)·

55

};goai ra de thAY

(Jk + ~)(Vk+ v'k+l)'

1

Do 06 bd2bl,: - h.-I - Ok+ll "2: 4k Tir do d~ dang suy ra dpcm. 0

Vi du 1.5.4 (Lam m{llnh bilt diing thll'c Chebyshev). Oid su cae .sa thw ai, G2, ... , a", b 1, b2, ·"1 b7t !haa man dieu kie«

Chlmg minh rilng

LOI GIAI. Do bieu t.huc

n(a1b1 + a2b2 + ... + a"b,,~ - (al + a:2 + __ + (l,,)(bl + b2 + + b1t)

LU(J11 ui~u Jiell throe qua !:8.(.; ~0 hang (a, - (! i) (bi, - /;d nilu ta dl tbti dmg lJJt di J day (at) va (b;) mot lirong tirong (fig la (al .-I- a2 + .. _ + (.!u)!n va. (b1 + b2 + ... + b .. )!n, dieu do co nghia La ta co th~ the gia sit {l\lO"c

a I + 0.2 + ... + a,~ = 01 + b2 + .. _ + bn = O.

D{I,t Sk = a I + 0.2 + ... + ak, theo cong thrrc khai triJn Abel ta cO P = alb. + a2b2 + ... + anb"

= (b1 - b2)SI + (b2 -113}S2 + ,,_ + (b,,-l - b .. )Sn-1 + b"S"

. ( )S2 ( ){'- b )5,,-1 b S"

= (b1-b2)SI +2 b2-b~ -2 +- .. + n-l '!tt-l - rt -- +n ,,-.

n - 1 n

Clni y ditu ki¢n ghi thict {Sk I k} La. day giam non ta sU' dung ti~p khai tri~n Abei cho bi~u thUc tren

p = ( SI - ~2) (bl - b2) ~ ( ~2 - ~3 ) (bl + b2 - 2bJ) + ...

f (5,,-1 - Sn) (b1 + b2 ~ ... + bn-I - (11 - I )b,,) +

n -- I n

(S'I)

+ n (bl+~+ ... +bn).

56

Chuoug 1. Bitt diLlg til UC co ::iii

Thea ~ia thict ta co

b > bl + b2 > .. ? b 1 + 02 + ... + b~ ,

1- 2 - 11

Ma dii!u kien trcn hoan toan tmrng dirong v{Ji

W.y p ~ 0, Ta co dieu phai chung rninh. U

Vi du 1.5.5 (Romania MO, Singapore MO). Cia sot .1:1 ~ X2 ~ ' .. :::: In ?:: 1:.,+1 ;;;;;; 0 la day 56 d'l1ctng rho truce. Ch1tng minb. Mt diing !h.tc

n

";Xl + x ... + .n + X" :::; L 0.( Vii - \1",1:;+ I!·

i=l

Lal CIAL DM Ct = v'i - Jr='l va aj = y'Xi, BAt dAng thirc nrong dmmg

( )2> '2 '2 2

aiel + a2c2 + ... + (Inc., _ a1 + a'2 + ... + an-

Ta phiU chirng minh

alCI + (!2C2 + p. + anCn ;:. albl + (1.202 + ... + anbn <=> al(CI ~ bt} + a2(C2 -~) + ... +an(Cn - bot);::O: 0

<=} (<<'1 ~ {lZ){CI - b.) + (a2 - a3j(CI + C2 - b1 - b-z) + .n.+

+ (an-l - a.,){cl + Cz + ... + C,,-l - bj - b2 - , .. - b,,-iJ+ + a,,(Cl + C2 + ... + Cn - b1 - 02 - ... - b,,) ;::- O.

Bai VI theo bat dAng thue Cauchy - Schwarz ta co

Bat dAng tlnrc de. du.:;!.; chung minh xong, 0

1. 5, Kns: trjell Abel v <'1 bAt dArlg ~h tl'c ho8.11 "'i

f;7

Vi du 1.5.6. Cho 2 day so dUr1n(j ai, az, ", (L" IJ(l b:- bz, __ '. bn lholi man

Ch.Ung mmii rhn.,q

Lot G tAl. Ta ~1i dung crnrng minh bang quy nap, hi~n nhien bat ding thu:c dung vol n = 1. Gia. sir Mt ding thuc da dung voi n, ta chirng minh no van dung vdi n + 1 sO, Th~t v~y, theo bat uillg th((1; Cauchy - Schwarz til. co

Tu gia thiet suy ra

bI + b~ +,,, + b~+1 ~ alb1 + G2bz + _, + un+lb;Hl

~ 6}(1I1 - ad + b2(b2 - a:!l +,,, + b,,+l(b,.+l ~ a,~+d ~ 0

~ (b1 - ~)(bl ':'" ad + (b2 - b3)(bl + b2 - a2 - a:!l + '"

+(b'l - b,'+I)(bl + b2 + '" + b;j - u! - fl2 - '" - Gn)+ +bu+1(bl + b2 + .. , + bn+1 - al - a2 - __ . - at.+d 2': o.

Chu Y rang trong tong tren tat ca cac sO hsng t:rit s6 hang eual cung la. kh6ng dimng, do d6 ta phai co

b1 + b2 + ." + bn+1 - 01 - 0,2 - __ ~ a,!+l ;:: 0 <;;:> hi + 02 + '-_ + b'l+ l ~ G] + 02 + ", + a,,+ i·

Bat dang thlk dl1l,.lc chung minh xong, 0

vr du 1.5.7. Girl 811 0 ~ z ~ y ::;- z va (1, b, C 2:: 0 thOti man h(} dit1J ki(n

ChUn,Q mini: rang

58

Ldl CIAI, Ta co

j(i + Jb + .jC - (JJ + jlJ + .jZ)

'!"(~ -1)1 JY (fy -I) +v'Z( jf- I)

(fi y'ij) ( fx - I) + (JY - .jX) ( ~ + fy - 2)

+ v. ( ~.~ V~ + l- 3) $ 0

VI thro b1i.t dimg th(tc C{lllchy - SChW(lT Z ta (',0

t r"

a a abc

fa < 1, ~ + ~ 2, ~ + . 1- + ~ ~ 3.

V~ y; V; '[» V;

Dang thl1c xiiy fa khi va chi khi a = z, b ;.;;; y, c ;;= Z. 0

Hi vong qua cac vi du t.reu t;fu: ban thfiy ro dircc 'S' ughta (:111>. kI rhuat (1~(.: biet nay. D~ ket t hue, esc ban hay t~t. chung minh car bAt ding tll\1c salt

vr du 1.5.8 (D~ thi HSG TOng HQp). Cno e<k s6 thl/c dUdn,q a, b, c, X, y. z thoa man diiu ki(n

{

x;:::Y2:z,a'-:;;x a1 + b'l :; J:2 + y2

03 + /}3 + c:> '5 .oJ + 11:1 + .::3

ChUng minb. rang

HUONG DAN. Hay chrmg minb tnwe a3 + bS $ x3 + y~, 0

Vi du 1.5.9 (Russia 2000). Cbo -1 < ;1:, < .[2 < -- < J'~" < 1 v(ll}l < 112 < ". < y., saoctio

Chltn9 minh bilt ddng thUc

1,5. Khai triAl! A bel va. bat d~llg ~h !r(' hmill vi

59

1.5.2 Bitt dAng thirc hoan vi

Dlnh Iy 1.8 (Ba.t d~ng thU'c Roan YD. Cho 2 day s6 ddn die-v. tdng Ul, a"2, "" a" va 01,02, ... , 0". Gid. Ml" ('i\ J2, .. " in) ld m(it h{)an tit Mt ki c1lG (1,2, ... , n), ta luon c6

alb, + a2b2 + -_. + a"b;-L ~ U,ilil + azb.~ + .-- + a"bin

N,qoiii m nhl 2 day It L 't Q2, . _, n"rJ.J III, bz. ,. _, b" do» di~tI nquac r.hieu thi bat dang 1h'l1t tn-;n du'i chieu.

CHlJNG t.-fINH. XH tniung hup rae day dAll lit d(jll di~u tang. B§.t ding thuc !lJun~ duuug vui

adb;. - bil) + a2(lJ.l- bi2) + -" + a,,(b,. - bin) ~ 0

{...} «(q - a:!l(b, - bq) + (a2 - a:l)(b1 I 62 - OiL - b'2) + - +(a .. ~l - a,,)(b, + b2 + ... + b" 1- b;1 - bl2 - •• , _:_ b'n_l)+ +all(b1 + b2 + , .. + b71 - b;l - b;~ - ". - bin) ~ 0_

Vl bl ;::-: b~ ~ ... ~ bn nen vClj moi k = l,n thl bl + b"2 + --- + bk ;=: Oil + b;2 + .-. + bik· V~y m6i 80 hang d13 t6ng tren d~ll khong a.m. Ta co dpcm. a

I\"cu 2 diiy rlon di~u nguoc chieu thl bit dang t hac dili diu va cling chang minh hoan toan tuong tl,t,

Chimg minh bAt. rlA.n!1; t.!J\~C hOI\n V! bhILg kliai t-ri(.n Abel rih ng1i.n gon, co I1hi~ll each dl\~ng minh khac clio bat ding thuc nay nhung noi chung dell [Jiang nhieu tinh s6 hoc hoac uai doug hen llhif.u. B~t dAng tl:Jt'k hoan vi Iii. mOt bAt d~[jg tl'i(l'( rAt mauh, {;hiiug han no l:{J th~ !:iUY ra true ti~!> L~tt tlimg then; AAl - G M nhu sau

vr du 1.5.10 (Bilt dAng thJ1c AM-GM). ChUng minh vd'i moi 56 th1lC kh6n,q am (l"a2, ",a" ta co

U,iI GrAI. RO dUll!, vilt Llitllg tlnn trt>!l hoau toau tU0J1g ut(on!? vui

Xl X2 X,,-l T,t

- + - + + -- + - > n.

;1:2 XJ XU ·T.I-

Xet bfit !ling I hue hoan vi vdi 2 dliy dan dieu nguoc chieu sau

.{

.1:1, I2, .. -, ,Ttl,

1 1 1

----;-'",-,.,.,

Xl X2 ,Tn

Khi d6

Xl X2 x'~_1 Xn Xl XZ X"

- + - +." + -- + ~:;:> ~ + - +.,. + - = n.

Xz IJ :(;n.1:1 X,:l:2 Xl

B:\I d&ng thur chtQ~: ~hl'cng rninh. 0

60

Chuoug 1- BA t dAng th l?c CO sCi

Vi du 1.5.11. ChUn,q minh 1.>01 moi a, b, c khtmg lim ta itujn co a5 + b5 + c5 ~ a.:lb + b4e + c4a.

Ldl GrAL Ta tip dung bAt ding t htre hoan vi vf1i 2 bf,i ddn di~u 83U day (a,b.c) . (a4,b4,c4)_ 0

Vi d1J 1.5.12 (IMO 1984 Pro, 3). GiG.5U a, b, c iii dr d41 3 C(lnh mot tam .Q1dc, hiiy chitng tninh.

Ldl C)AL LU\l Y rAng n&u G, b, c 1ft do dai 3 canh mot tam gi5.c thi

Vi;i.y 2 bo s6 sau day Ie. 2 bo don dieu nguoc chieu

a2 + be , b2 + ca , c'l. + ob; III

a ' b ' c'

Theo bs.t ding thirc hoan V! ta Co

a2 + be b2 + ca c2 + ab a2 + be b2 + ea c2 + a.b

---+ + <--+ +--

abc ~ e a b

be ca ab a"i bZ (;2

$>-+-+-<-+-+abc - cab

¢:} a')b~ + b'2c'l + c'la'l ~ a3b + b3c + c3a,

Dily Iii. ui~lI Vli<i.i ..Jaing rninh. Liru y n\ug lieu khoug (lJ g;ia tltit-t a, b, c ia uQ uai 3 canh ~'ll<L t.am J",hk tbl brLt dr~uJ", HHi'~' kliC)JIg d{lIIg 0

Vi du 1.5.13 ([MO 1975). Clio 2 day s6 thl,tc Xl •• 1:2, ._-, z .. va Yl, Y"2, -, Y .. thoci man Xl ~ .'(2 :::; _-. ~ xn, Yl :::; 1/2 :s: ." :s: Yn' G91. Z" Z2, .'" z" iii. m¢t hodn vi cua 1/1, 1/2. - -, 1/,.. Chitng minhrdn_q

L<'Jl CIA], Day t!lI,tc chAt chi ia bAt dhng tlnrc hoan vi thong tllliang, 0

vr dl.11.5.14 (IMO 1978). Cho day so n_quytn d1trJng phan bi~t ai, U2, --, an' ChUng m'inh rdng

1.6. BAt d~llg tlul'c d6i xt1l1g 3 hi~rl

61

Lor G I AI. Sip xep day a l , U2, ,." a" thanh day dun dieu tang z i , ,T2, "', ;r n- Thco bat (lit-llg th UI: hoan vi t <1. <':0

Xl 1:2 X,, 1 1 1

l2 -I- 22 -+ ,.- 01- n2 ? I j :2 + ._. -I n'

Ta c6 cli(u phAi chrmg minh, 0

Vi du 1.5.15. Ch11nrJ mint: uoi 1n9i so dJ.tdng a, b, c thl

..

a2+bc b2+ca c2+ab.

b +---+--b->(~+b+c-

+c c+a a+ -

LOI GIA!. Do 2 day a 2:, b2, c2 va (,!c' clu' a~(, luon til. 2 day don di€'u cling chi~l1 neu theo LAt JA.ug thule hoau vi ta co

a'2 b'l r? + ab b2 c~ a2

-- + -- + --- > -- + -- + --.

b+c c+a a+b - b+c e+a {l+O

Do d6

a'i + be b'l + en cZ + ab b2 + be 2 + ea a'l + ab b

----::--- + + --- > ~~- + --- + = a + + c.

b+c c+a a+b - b+c c+a a+b

DAng t.hirc xay ra chi khi a = b ;;;;;; c. I:]

1.6 B.§.t d~ng thli'c d6i xirng 3 bi~n

BAt d~ng thl'J'c d6i xrmg la mot trong cae phi\n gUM trong nhAt cua hat dang tlnrc so ca.p. cimg Iil. dang bai quell thuoc trong cac ki t.hi hoc sinh ~ioi qtl5c gia va qt\tJC t~, Day In dang b5.t dAng t huc r~t (ht(,l'c yBli l.hich khOng chi vai rae ban d5. thanh thao rna con hap daII vdi uh1.1ng ban nll';i biit dAn, Co It> If do (I au gian la cac ba.t ding thuc dang nay d~\l rfu clt:'p va chuan ve mat hrnh thirc. Tuy nhien khong phil.i m¢t bai t,O.9.rt vdi hmh t lnrc piLat bi~n cion gUi.n rimg (~6 th6 ~iai (lit(jc then eAch dan gia.n, t harn chf nit nhieu bai toan nhu vay nhung ldi giai tAt lifi kho,

N6i cnung, eat; bat rifmp; tlnrc dM x\'rng 3 bi6n il dang t6ng quit lu6n co bi~u dicn duo! dang !;311

f(a, b, c) ?_ Q

1.6, Bil.t cUiug thue d6i X!i·llg 3 bi~1l

61

LOI G I A.I, S~p xCp da,y (.II, U2, .. " a" thanh day dun difu tang x I. X2, .. " Xu' Theo bat diiug: thu\C hoan vi ta 1;0

Xj)72 xn 1 1 1

2" + 2" + ,-. + 2' > - + - + ... +--

I 2 n - 1 2 n

Ta co dj~u phai cilt'"mg minh. 0

Vi d1,1 1.5.15. ChUng minli vai ~(Ji so dU(}ng a, b, c thi

a'1 + be b2 + ca c2 + ab

--- + +--b- > u+b+c-

b+c c+a a+ -

LOI GIAL Do 2 day aZ, b2, c2 va ~, 1;~tI' ~~b Iuon la 2 day don di~u cimg chi~n llf.n theu L.it uiug thirc hoan vi ta co

a2 b2 c2 + ab b'l. c2 a2

~~ + -- + --- > -- + -- + --, o+c c+a a+b - b+c c+a a+b

Do do

a2 + be b2 + ca c2 + ao 02 + be e2 + ea a2 + ab b

----,-----+ + --- > -- + -~- + =a+ +c.

b+c c+a a+b - b+c c+a a+b

Dang thuc xi1.y HI chi khi a ::::: b = e. 0

1.6 B&t d~ng thuc d6i xirng 3 bi~n

B~t dAng thuc db! xtrng IS. mea trong cae phArr quan trong nhiit cua bat dang thirc 50 cap, cung Iii. dang bai quell thuoc trong de ki thi hoc sinh r;ioi qu5c gia va qucc t~, Dfly la dang bfit cl~ng t ht'ie rflt duoc yell tluch khOng chi ... di cac ban da thanh thao rna con Mi.p clan vOl [lIlting ban nidi biil dAu, C6 Ie Ii do don gian la cac bAt ding thee dang nay dfll r~t (!!;'jl va chuan ve mat hmh tlurc, Tuy nhien khong phiii mot bai toan vdi hinh t!.(ft phfl.t bi~n d(JI, ginn rimg I~O tht giai d\l'C;k theo 1'01.r.h don giit.n. tharn (;hf rit nhieu bai toari nhu v{i,y Ilhl.tng loi giai d.t lal kho,

Noi chung, cAe b:lt c:lAn!l; tll1k df,j xirng 3 bir-n a dang t6ng quat luon c6 bi&u dicn diro: dang sau

f(a,b,e)~O

62

Chltong 1. Bat dAIJg thac co 80

trong do I(a, b, c) la ham doi xirng dla 3 bi(;n fl, b. c. hay noi each khac

1(0., b, c) = [tc, b, a) = fro, a, c).

Ching han f(a, b, c) = a,2 +- b2 + c'l + 3ab + 3bc + 3m + 5abc + a2b2c?

TInh chat quan trong nhat cua cac bieu thuc d6i xtmg ta vai tro binh ding giua. d,l; hi~tJ, va do do ta eo die xa,p x?p la: theo mot trat tu tu)' y gia t.ri cac bi~n :;5 uu t rcng {;hlt.lJ~ minli. Day lit HlV! ch(1 s :oi:: ,hrl,1{; :,;li' dung l<1t uhit-u.

Car: r inh ddlt vii. djnh Ilj!,hla nav ciLrOl' nlo ronl-', t liilliR tl,r voi r,k bi(~1l tli(w rua 11 bien XI, X2, ... , In· Trong phan nay dLl'\llg ta ~e tim hieu khai quat v~ cac bAt ding thtrc d6i xirng 3 bi~n va mot ~ ki thuat lieu quan. Ta phan chia c1il.: b~t dang thuc d6i xung th311h 2 dang chinh til. cac b~t dAng tlll'(1: 1;0 r1ieu kien va kh6ng co di~u kien nAn tirm t a ""~ xem xa. ('ae bAt rtilng tll\~c vdi d,r bit'n t.IJ do.

1.6.1 B&t d~ng thuc thuan nhcit khong co di~u kien

Tnnrc het ta coin dinh nghta cho mot ham lib thuan nhat Ham f(a, b, c) dltl}C goi Iii thuim nhAt voi cac bien tren mien I neu no thoa man di~u kiCn

f(ta, tb, tel == tk J((I.., b, r}

VOl rnoi t, a, b, eEl va k Ii ITlQt hang so khong phu thuoc vao a, b, e, t rna chi phu thuoc vao Lim than ham r Trong pham vi cua U1), thuc thl mot da thul; Iii timan nhat utlu no la 1ang eua cac don thuc d5ng bi).(~. Ch~Tlg han ta co da thire t huan liMit {khong dOl XOOlg) sau uay

J((1, I), c) = a.5bc" + a2b3r~ + ab(;c2.

Vi du mot ham s& thuan nhat khorig 180 da thl1l'

f(x) = r" + _x_.

IcQ + :r2 :x + c

Dinh nghia trsn co th~ rna rong cho ham so cua n bien Xl, X2, ... , Xtt.

Lop cae ham t huan uhAt trong bAt dAng t1ntc I:UIlg ia mot phan n\t fQOg ella bAt ding thirc, va.:,;~ dilQc nhac 1;;11 nit nhifu tau trong cuon sach vi t inh quaIl hong CUR no. Do do ban doc nen xern kl cac tinh l:hiit cua loai ham nay. trong do tinh eh5.t !lltq" Sl'[ dl.lfl"; tlll(tJng xuven flh!1.t lil Hnh ddl.l t.hnrin hfia duac. Tfnh chAt nay cung se uuqc ban I~J kl hen nguy trong ph fin -sau,

Md <lfLu va bAt dAn!!; thl~C dfii XI~Ilg: 3 bien timfln nlJAt la mJ)t bA.t diing t lnrc ('I_Ie ki n6i tieng vii. c6 nhieu irng dung, d6 Ie. bAt d&ng thtrc d6i )(tcng Schur

a(a - h)(a - c) + b(b - a)(b - c) + e(c - a)(c - b) 2: O.

63

Bitt dang thuc trim th,(ung ~hr[,)c pM .. t hi{.u du6i dang quem thuoc hun nhu sal!

D!nh 1:9 1.9 (B&t dA.ng thl1c Schur}, Wi rnoi s6 th~tc khuug am a, b, c ta IYDn c6 bdl. ddng thue

a3 + b3 + C3 + 3abe? ab{a + b) + bc(b + c) + c.a(c+ a) ..

CH(rXG :'\-[lNH. Co rAt nhiou riu:h clnrng minh bft.t r1Ant; I bl~ tron, fJ df~y t.a£: ?;i;\ S(~ gioi thicu mOt deh cmrng minh rna ~ con them nhicu (rug dung sau nay, Ngoai ra, da.y l:img la mOt each ch(tng minh di.t don giiiu va nga.n gC,m, Do tinh ubi ximg (;l~la biit d~ng tlurc ta co the gia sit (J. ~ b ?: c. D~t x ;;;;; a - b, y = b .- c, bAt d~ng thuc (hree vi~t lai thanh

c(.T + y))! - (c + Y}TH + (r: +.T + 11).1'"(.1: + y) :::- 0

,

{;} c(;r2 + Iy""l ]12) + ;r2(x + 2y) ~ O,

Bat U~llg tlnre treu lli01l nhicn l!iUI& VI (;(\1; bi011 C, X, Y 11~11 klJollg alIJ. D~llg tJllk xa.y ra khi va chi khi x = 1J -= 0 hoac x "" c = 0 hay a ;= b = C hoac a = b, c ;;co O. Do gia 51! a 2: b ~ c Q dA.u bai to an nen cac truong hap x '-' 0, y = z va y = 0, X ::: Z cung co dil.ng th(l'c. Vt. .. sail ta chi noi gQn la a ..:..:. b. c = 0 hoac cae hoan vi Bil.l toan cia (h(tJ(: chdnf.'; nunh. 0

MOt each tl.,l' nhicn, ta IT11ltm :xem xct mQt m~ rong t ncng tv

Vi d\). 1.6.1. Khdng d,nh hol).c phil dinh bUt dang thUc

alj + '!If) + cB 1 3a2b2c2 ;::: a5(b + c) + b5(c + a) + c5(a + b) ..

Loi G 1 A I. Rat tiUc. bAt. (tt.ng tl1l1c I ron khOng phai luon dung Cho b ;;;;; c = x

oY + 2x(' + 3a2x~ "? 3a5:r + 2/\a + x) *> aU + 3a2,.4 ~ 2a5x + 2x5(J.

<I;;? (.15 + 3ax4 ~ 2a4x + 2x5.

Tuv nlliC:n br..t dAn!,,; tli(!,· tren khong dim!!; n(~ll t.rI dJ{) (l = O. D6 ro hen. (~6 th~~ eho a = I va b ;;;- (" = 4. 0

!V!{lt. va.n at: khao duoc da,.t ru lit

Vi du 1.6.2. Lieu c6 tdn t(l,i mOt hAng 50 k sao cha Mt .ddng th~c san iuon dung

Vil n~u cd, hay !l.m Mng .';6 rot nMt (nho nhal) co the' d((~c.

64

C fHtc11lg 1. 8.1 t (Uing t hue co :sa

Lc11 G!A!. Dang ti~c lit khong tOn ti:\.i mot h~g 56 k: nao nhir vay di. va do do c(mg khong co hAn!!; So tot n hat. That khong may nuin la 2 kct qua. thli' nghiem mol Cull, chung ta Jell dUn den rnQt k~-t qua. phu dinh. Nhirng C{IC ban dung vi th~ ma e ngai viec tu ;o;;ing tao cac bitt d;ing thuc, VI nhung ~\t khong may man nhu v~y lit clieu k hong tbf tr anh khoi, thnrn chi 11\ rat nhieu, DOl voi mot bAt dang thuc, sf- kho hun r;\I nhieu w"'11 !~fi\l hrii d~t. 1'<'1 lil. khring dinh hoar; phi"i. dinA, chir chua ph iii 13. ch11n.Q mi-nh bUt rtrin,g t~uc, Thdrn chi trong khi ch,tng minh mot h5.t ding thnc khr5n,g dung ta hoau toan c6 th~ lam dung no d~ d~tQc mot bat (Iimg there t6t hon. 0

Chung ta. hay ticp tuc tr6 lai qua voi mot k6t qua kha hien nhicn 2(aG + b6 + (6) ;::: as(b + c) + b5(c + a) + {:5(a + b),

Muc dich bay gio la tot ~ thi'm d~i lllong chua a2bic2 vao hieu thirc ve trai, va giiim he so 2 den mire nh6 nhat eo the dWJC d~ co mot bat clAng thirc el(\ng. Theo cac suy Juan a- trcn ta khong thiS giam he 56 2 tci 1. Co mot ket qua kha dep mAt

Vi du 1.6.3. CMng minh »ai m9i a, b, c th1,fc

0,0 + b6 + ct' + a'l.b2c2 ~ ~ (a5(b + c) + b5(c+ a) + cS(a + b}).

Ll:Il Glfi.!. Chung minh bat dliIlg thuc tren kh{)ng qua kho. nhirng viec tim ra no lai la ruot C(lHg Vi~L klia vAt v~, Sit lh,mg bat 11iug tllll(; Schur' ta Co

aU + bti + cG + 3a2b'c2 ?': a<l.(l/ + c2) + b4(c2 + a2) + c4(a2 + b2), Thco hAl diing ttuk AM - OM tht

(ab + (14/)2) + (a6 + a4(2) ? 2a5(b+ C}, (b6 + b4c2) + (1/; + b4(2) ~ 2b5(c + (I), (c(J + c4a2) + (cG -+- c"b2) ? 2c5(n + b).

Cu6i cung, ci;\ng cit. 4 loiit dAng thUl: tl'l'Il lai ta (,:0 ..lieu phai elurug uiinh, 0

Mo ri'lng bAt dAng tlll~f' Schur vai bij.c nh{..t 0- tren ta 1'0 bAt dAng tl:nt Schur vdl b~ 2 sau day

Vi du 1.6.4. CMtn,q minh rhng l'oi mQi .96 thuc kh6ng am Q, b,c to: e(l

u4 + /;1 + .-:4 + (lb<:«(1 + b + (:) 2> a3(b + c) + b:l(c + a) + c:.l(a + 0),

Lor GIAI. Cae ban bay xcm lai .Ikh dn~ng -rninh rho btl ... ding tllllc Schur 0 dang co ban (bac nholt), ca 2 clurng minh den hoan toan gi6ng nhau, MOt di~\1 dang chu y Ill. hilt ding thtic: SchUT bac 2 dung voi moi a, b, c thW,. [J

Ti:t k6t qua trrm ta suy ra

100

C1lltOng 1. B~t dAlJg th!l'c co sa

BM roan 1.51 (Russia MO, Crux). Cho cae so IhW (ij,14k-,at1, XciI cae Mill.

b al + a2 + ,.- + ak wk

k = ~ ~ 1, n.

k:

, 2 2 2

C = (al ~ bil + (U2 - b2) + --- + (a,.. - bn} ,

D ~ (al --- b"f + (a2 - bn)2 + , .. +.{a" - b,~)2_ ChUng mmh bAt dang Unle

C$D$2G.

Bhl toan 1.52 (Viij!t Nam MO '2002). Cho cdc $6 lh7,lc z, 1/,.1: thad man ddu kif-n x2 + y2 + .0:2 = 9, ChUng minh rdng

2(x + y + z] - xyz ~ 10_

Bai toan 1. 53, ChUng minh rdng vrn cac 56 dlidng a, b, c, d ta luon co ~+~+2 ~+2+~ ~+~+~ ~+~+~

-----:- + + + > a2 + b2 + c2 + d2,

b+e+d c+d+a d+a+b a+b~c-

Bai toan 1.54 (Indian MO 2003). Chi1nq mink ran,Q x3y:5(X3 I y.l) < 2 trai cdc s6 th'llc khaug am x, Y co t6ng bang 2-

Bal toan 1.55. Cho x, y, z M cdc sa th1,JC thoci man die:u kifn x + y + ~ = 5 va x2 + y2 + z2 = 9, TIm gid tri tan nhllt va gia try nha nMt CUll bilu thUc

A = x'ly + y2z + z2x,

Bai toan 1.56 (Mongolia MO 1991). Cho cac s6 thl,lc c, b, e cd t6ng binh phtfrJng Mng 2. ChUnN minh. rdng

la3 + b3 + cJ - abel ~ 2 h.

BM tom 1.57. ChUng minh Mt ddng thUc ,_'Oat! 1)01 m9i a, b, e khOng am

a(b + c) b(e + a) c(a + b) 2

p+~+~+~+oo+~+~+~+~~ .

BRi toan 1.58. Cho cac so th!,(c audng ai, 0."2, '" > an cd fich bli.n,q 1. Chfrn9 minh

2 2 2

Ql + (12 + ... + aTi > -- + -- + ". + ---

- 1 +al 1 + a2 1 + a,;,

Bai toan 1.59. Gia $11 c, b, c id dQ dai 3 qmh cti(l m9t tam giae. ChUng minh nln!}

a3 + b3 b3 + 2 e3 + (1.3

-"----------,,, + + < > 2(a + b + c).

b2 - be + c:'.l ' (:2_ ea + a2 a2 - ab + b2 -

98

Bai toan 1.38. Chftn9 minh rang v&i mm a, b. c khu11y alII ttl ("u bM dang Ihuc

C1{b + c) b(a + c) c(a + b) 2

~--i-+ + > _

b2 + c2 a2 -t (.2 a2 + b2 -

HUO:-;G DAN. Khong mdt tfnh t6ng quat, gia su a ;;:> b ~ c, xet

Ding t.!n·rc xliy rn khi vii. clii khi u. - b, t = () hoac cac hoan vi. I::J BM toan 1.39 {Crux). TIm ch(j.n trfn LUn uhiAt cho bil1l f.huc sau

Troaq d6 X},X2, -'-, Xu lG cac so th'l,tc irony [0,11.

HIWNG [)Ai\. _ Su dung phirong phap ham 1Oi. tl m diroc chan t ren la n - 1 vdi n ~ 3_ D

Bai toan 1.40 (Crux). Chiin.Q minb. rdn,q vdi moi a, b, t: ~ 0 th~

Bai toan 1.41 (~p chi Toan hoc va Tu6i tre). Gia S1! a; b. (. la cae so Mltc kh6ng am co U5ng bO.ng I. Chli'n.g minh rdnq

ab + be + ca 2 2 ~

a2b'l + b2c2 -I- c2a'l ~ S(u + b + c ).

Bai toRn 1.42. Cho cdc sd thw kMng dmal,a2, ""' ar. thoci. man al+a2+.,,+an = 1. ChUng mink rli.ng

8M toan 1.43 (Poland 1998). Cho 'cac 36 thuc khbng am a, b, c, d, e, f co tang Mng 1 vii ace + bdf"2: 1/108, ChUng minh· bilt ddng that

1 abc + bed + cde + de! + e]a + fba S 36.

96

elmong1. B~t ([dug thue co .~a

EM toan 1.29. Gia .~it (I., b, c iii cae so th1,ir: dUM,1} tbo« man

111

.-.- + --- + ---- ;;;;; 2.

l+a l+b 1+c

ChUng minh. ran,q

1 1 1

- + - + - ~ 4(a + b + c).

a b r

HI.~dNG DAN. Gio.llli(.t. I~l'ta bii.i 1.0/1:(1 nmng d,tlJng .-di tfHl tai .r, y,:: dt a :=: -__::_ b =;; _y_ C '-- !: -

y+z' z+x' x+y

PiLAu l:un l~i (hr\,fl: I,:h((ng minh tuollg d0i t.le dang. 0

Biii toan 1.30. Cbo ole so Ihl,fC khonQ um a. b, c tbo« man (12 + b2 + (.2 = 1. Chlrng

mink rang

« + b + c < viz + Q(1DC

- 4

BAi toan 1.31 (Russia MO). Gia SU.T, y,.I: Iii r:ae so dUdn.q co Uing bdng 3. Ch11n.q mink ntn.Q

vx + JY + .fi ~ xy + "z + ZX.

Bai toan 1.32 (Romania TST 1993). Tim s6 th1,tc d,tdng (.I ton nMt dl vat dang thiic 81.1ll dung mJi In(,!! X, y, z E R-+-

X + Y + z > a.

Vy2+z2 ';",-2+;('2 Jx2 + 112 -

HVClNG DAN. Gia tri t6t nhlit ella a Ia. 2, Sfr dung b§.t d!ng thirc AM - OM. 0 BAi tom 1.33 (Crux), CMmg tninh. Mt ddng tMc sau 1Jd'i mo-i s6 th1jc dUdng

x,y,z

x " Z

--""""F'=~:::;=== + . + ::: 1.

x+ j(.'L'+Y)(x + z) y + ..j(y -+ x)(y + z) z + J(z + x)(z + y)

HtJONG DAN, Ta e6 hai each clnmg minh nhu sau

each 1. TlW(I bat uAng thuc Cal1chy - Scliusar z t.hl (x + y)(,r; + z) ~ {x + .,ftii)2

x x

~ < ,

x + V(x + y)(x + z ) - 2."C + y']iZ

Do d6 ta phai chimg rninh nhl abc"" 1 thi

1 1 1

--+~-+--<1.

2+a 2+b 2+c-

94

Cll!1C1Ilg 1. BAt dAuB' thue ca so

1Il_:'0KG DA:-<. BAt uing thl~C btong otl;Jng

St( dung khai trifn Abel ta c6

ta, _ t ~ = I: ((~ - /+,.) (L:,._k._'l at - ~",kl b~) .

i=-l i'" I' h=l 1 ,

Ding thuc xay ra khi va chi khi (J.~ = hi Vi "'" 1,Il. r:J

BM toan 1.19 (Crux). Cia stt n ;::: m ~ 1 va x ? !J :;:: 0 thoo man x,,+l + y,t+l ~ z" _ ym. ChUn9 minh rlin,Q z" + lin -:; I.

BM toan 1.20. ChUng minh vai moi .r. y, z E [O, 1] Ih1. x2lJ + i/< + z2:.c 2: 3/4.

Loi CIAI. Sli dl,l!lg bat ding thirc Xli ;::: x/(x + y _ xy), <Ilia bat dang thUc v~

(--=--)2 + (_}j__)2 + (_z )2 >~. 0

x+y y+z, Z+,T _ 4

Bbi toan 1. 21. CM"tng m inh ranq vdi 7n9i a, b, c "2: 0 t a . co

a2 + b(: I} + Ca ('2 + ab 3

~-~+ + > -r- •

(b+C)2 {c+ a)2 (a + b}2 - 2

HVd:-iG DAN. Sfr dung btu d:!ing thuc Chebyshev. 0

Bai toan 1.22 (Tap chi Than hoc va Thai tre). Cho cec so ths«: kh{Jnq am ;1;, y, z thrui miin df~U kifn .r + y + z = 1. Ciuln(j nunh. r1J.ng

:r.2 I 1 y2 + 1 Z2 + I 7 --+--+--<-.

1/2 + 1 z2 + 1 xl + 1 ._. 2

Bai toan 1.23 (1M 0 Shortlist 2004). Cia sU' a, b, c Iii cae s6 th!,lc dttdng sao cho ab + be + ca = 1. Chf~nq minh rdnq

if~+-'~b + ;jr~·~ + fr~ ~.: < ~b·

Bid toan 1.24 (IMO 1995). Cho a, Ii, c ~ 0 1/(i abc;;;;;; L Ch11n,q minh

1 113

. +'. + . >-.

(lJ{b + c) !P(c + a) c1(a + b) _ 2

92

Clutoug 1. BM (1i-1lg th uc co so

11 V(lNG DA N. Ta phjii chltng m inh vdi a. b, t: E [0. k 1 thl

a(k - b) +- b(k - c) + c(k - a) S' e.

Tuy nhion v~ trai ls ham 16i vdi moi bien a, b, c nen ta chi dm xot khi a, b, c E {O, k}. Kbi du Lilt tliing thlk hiJn nlusu dung.

N goai ra ta c6 thi chimg minh true ti6p khong dung ham If-i nlnr sau

. ';'

(k-Il)(k-b)+(k-b)(k-c)+{k-c)(k-a) ~ () => 2k(a+h+d- ((lb+bc+<:u) ~ 3k .

Do ao neu a + b + c ~ k ta co ngay dpcm. Tnrcng hop ngucc lai, de thay k(a + b + c) - (all + be + co.) :::: k(a + b + c) '5 k2. 0

Bai toan 1.11 (APM 0 2001). Cho cac ,.,6 th~1<: dlidng G, b, c thOli. man 1/ a + 1 [b + lie = I. Chimq minh

va + br: + vb + ca + -Ie + ~b :? JabC + Va + Vb + .;c .:

HUONG DAN. Sfr dung ba.t ding th(rc

Ie + ab;:: ;C + /¥.

Chu y ding thee gia thit~t ta co

it _ f¥b (!;C. fi~a-

vaoc- -+ -+ -,

cab

Sau do ap dung btu dftng thitc AM - GM cho 2 sO. 0

Bai toan 1.12 (Math. Chantleges). Cia 8uul,a~,. " a" Iii cae $0 11I\(r; thw)c dur,m, [-2,21 Tun. gld tri nit..; nhfit etta bdu thur:

HUdNC DAN. S11 dung phttdng phap ham 161. 0

Bal toan 1,13. 11m Mng 50 k = k( n) !an nhdt saO cno Mt dang tMc sau dung ,lrji moi day 36 khi'mg am al,a,}, "., a"

mJdNG DAN. D~ dang tim utioc k(3) = 3 va k(4) = 4. Vdi n;:: 4 thl k(n) = 4. TMt ~y, cho al = (.12 = 1 va ui "" 0 'ij # 1,2 ta suy ra k(n) ~ 4, D~ clnmg minh k(n) ;;;;;; 4 thoa man ta dung chirng minh bang quy nap 0

90

CllItoJlg 1. B.§:t dAug Ihue co sO

Bai tQan 1.1 (Balkan MO). Cbo 3 .,6 thuc duaru; a, b, e n-l tich bdnq 1. Chml9 minh IJJt duny fink

a+b b+c e-t-a ( 1 1 1)

--+--+--+6>2 a+b+c+~+-+- .

cab - abc

HVO;';G DAN. TBn tai cac 56 dirong .r, Jj, Z sao cho a = x/yo b :;;;;; y! z , c = z/x Bat dAng t lnrc tra t hanh

;,[:3 + y3 + z3 + 3xyz 2': .TY(X + V) + yz()/ + z) + zx(z + x), 0

Bal tOO" 1.2 (Russia MO 1999). Cho cae 86 thJ,tc dllrJng .r, y, z co tick bdng 1. ChUng minh rdn9 nlil

1 1 1

- + - + - 2: x + y + z, x y z

Thi Il{n m9~ 1>"6 n.9uyt'fn dUr1nq k ttl co

HL'ONG DAN. Dj~u kj~n cua bai loan Luong dirong VOl

(x -.l)(y - 1)(z - 1) < 0

Sur ra vdi mci so nguyen duong k thi

(xl.; - l)(yk - l)(zk - 1) ::; 0, 0

Bili toAn 1.3. Cha cdc so thl,tc x, y, z thu¢c { -1, 1), ChUng minh rdng

1 1

~~-----,-------,--""""""'---:- + ' > 2 >

(1 - x)(1 - y)(l - z) (1 + x)(l + y}(l + z) -

Ht'O:-"C DAN, Sit <hlDg tIl,ie tif'p ht~t di..ng, t InrI' AM - G!l1 cho 2 sa hang 6 vc trai ta c~ dieu phai chirng minh. hru -:; r:i.Ilg

(1 - x)(l - y}(l - .::)(1 + x)(l + y)(l + z) "" {I - x'l)(l - y2}(1 - 2~) :-::; 1. 0

Bal toan 1.4. ChUng mint: nln1 vai m9i so dlldng a, b, c !I!. ivon co a4 + /)4 + (A 3abe 2 (2 62 2) ----+--->-a + +e.

ab + be + ca a + b: + c - 3

BM wan 1.5 (USA MO 2002). Vat a, b, c la cdc so th1,fc d:!tdng, eMtnil minh (a5 _ a'J. + 3)(b5 - b'l + 3)(c5 - c2 + 3) ;;:: (0. + b + c}3,

1_ Y. D{io hAm vA ung dllllg

87

Yo. Ute do g(yo) ~ g( ~) = li~-

') 15 -.

Tu (10 k~t iH)lJ vui {3 !;1I)' ra Pi», y, z) ~ _q(y) :;:: g(Yo) = 2' olan~ then: xay ra vci

y = ~,:r = 3, z = ~ hay u = 1/3, b ~ 4/5, c ;;;;;; 3/2 _ 0

T6ng quat t:ua bai toan nay da duqe giai b~ng phu'l1ng phap can b{.ng h~ so a rnuc tnroc Nhirng n~l\ xet thea mot each uao do, chimg minh su dung dao ham v~n la dun gian nhat ve mat y -t.u0ng N6 J.:1Lfig Ill. mot vi du tieu bi~u cho phuong phap giiinl bitm - m(H kl thuat qnnn rrong trong vi~(~ rliwt'; d~) ham <1~ giAi bAi tOfln nrc tri ham s6 nhieu bien. Ta hay xcrn bAt ding t.hti'c Schur eo th& chirng rninh bAng phirong phap nay nhir t ~~ nao don gi';u tM nao?

Vi du 1.9.6 (Bit ding tht1c Schtlr), Ch11ng minh Va,b,c ~ 0 ta cO a;,i + b3 + c3 + 3abc ~ o.b(u + b) + bc(b + c) + ca(c + a).

LOI G I Al. Khcng mAt tinh t.6ng quat, gia l;tl It.ng a 2: b 2: c. X6t ham s6

f(a) ;;;; a3 + 1/ + c:~ + 3alx- - ab(a + b) - bc(b + c) - ca(r + a) t(u) = 3a2 + 3bc - 2ab - b'2 - 2ac - c2

DC thiy 3a'2 ~ b2 + Zac va 3bc ~ c'l [len t(a) ~ (] vdi a ~ b ;::: c. Vi;ly /(a) ;:::: f(b) "" c;,i + 3a2c - 2ac(a + c) = c(u - c)'i :::- 0,

Bat !lAng thuc ,hi!;!C chung minh xong. DAng Hit'll' xay !.<t chi khi a "" b == c hoi).c a;;;;; b,c = 0 bose cac harm vi. 0

1.9.4 Md' rQng rn9t bhl thi toan q u&c tii 2004

Moi bai torin co ITu)t dJ:ic thil ri<':np;. Co nhimg bai toan rns d~r. thfl tim n6 chinh la co so de cae clnrng minh mung tinh k5' thuat (Ghing han phirong phap can bang he so) tl'O npn hitn d •. lIIg_ Thucng 1101 esc chirng minh do d.t bap dAn. co l€- boi VI tfnh dun gian e.;lhl n6. Tuy nhif.n VIf'C tim ra Gac chung minh dep de I heo nghla nhu v ~.y trong ua so cac tnrong hop la rat mo bb. T'r<ii lai plurong phap sit dung dao ham dl ve e6ng kenh nang vii: tinh loan co th~ I;;\i la con dU:ong de thuc hi~n nMt. Xin earn on han ~ guyen Quo;; Khanh dii gnip dci hOM thanh bai vi~t nay.

Vi du 1.9.7 {IMO 2004 Pro.2). Cia su n iii mot s6 N nhien iem han 2 va n so thuc duong .171, Xl, _",:r" thoo. man

(1 1 1) 'i

(Xl + x1 + ". + Xu) - + - + -" + ~ < n + 1.

Xl X2 xn

ChUng minh rllng m9i b~ ba trong so n so d6 Iii d(J alii:) c(lnh cua mot tam gidc

88

Ch,tdllg 1. H~t dAng thuc co .'iO

Vi dJ.,1 1.9.8 (Timg quat). Gia su 11 vii. k la hai s6 tv nhie.n thoa man 11 ;:: k > 2.+ Tim 56 th1tc Ian nhiit {J (n, k) co tinh eMt : bat ky k tr-on.9 n 56 thljc dU:dng Xl , X2 •...• In S~ ia do diii k cq,nh eua m<)t da gicic LJi ntu

( 1 1 1 )

(Xl + X2 + -- + ];n) - + - + ... + - < g(n. k).

Xl X2 In

LOI G IAI. Chung ta bj6t ding a~ ::::: GI;_ I ~ -. - - - - - ~ a I Ii'! dQ ua.i k canh cua mot k giac loi khi va chi khi al + a2 + . __ + ak-l > (J.I.;. Do d6 bai toan SftU co the di~n dat 117-1 1:\: vdi rti(~u kien Xn ;::: Xl + X2 + ... + X!c- 1 va I" ~ In -1 ;? _ .. ? Xl, tim gia tri nho nhat eua ham 56

( 1 1 1 )

(XI+X:l;+ ... +xn) -+-+ ... +~ =g(n.k).

Xl X2 X"

D~ lam di~u d6 ta se thj~t I~p h~ thuc lien h~ g(n + 1, k) va ;11(71, k)-

Gia. BU" rrmg gia tri g( n, k) dii. xae dinh va dAng t h(r(_; (:() xay ra tai (1'1 •. 1'2, • - -, Xu) vdi :tn·;:: Xn- 1 ;:- _. - ;:: XI· XCt. oi~u kien

D:;lt

A = XI + X2 + -" + X,I-l + X,,,

1 1 1 1

B = - + - + -" + -- + -.

XI:1:'j X,,_I In

Xct ham 56 J(x) = (x + A) (~ + B) vdi x > ()

I 1 x+A A

f (r};;;; - + B ~ -- :;; B - -,

x x2 x2

, fA

f (x) == 0 $} x = V B = Xo > o.

Do A ::;- nXn+l> B ~ nIx,,+! => Xo ::; Xn+l. Tai Xo, ham f dat cue tieu VI W)Y

Thco gia. thi6t AB ~ g( n, k) v~~ llfulg t lure xiiy ru (.hiljt: W:"lI

g(7I + 1, k) ;.:: ( Jg(n, k) + 1 f .

Ldl 01..\1, DAt x ;;;; 1/a, '!.J = 2jb, z = 3/c b3.~ toan chuven thanh :

Xit cac s6 thuc d1idnl} x,y, z tho« man 12x)/,;: ~ 2x+8y 1 21z, TIm_Q'ia tri nhii llMt ctia bdu thUc P{x, y, z} = x + 2y + 3z-

Ti't gia. thict z(12xy - 21) ~ 2x + 8y > a t\'t do z?: 2x + 8y v(li x > 47 (1). Suy ra

12xy-21 y

2x+ By

P(x, y, z) _~ x + 2y + 4xy _.., (2}

s. f ( ) 2 x + 8y 4 x2 11 - 5x + 8y .. b· < 7. 1 ~ h <

Xet ham so a: = x + =' , VOl ion x > - va y a t am so

4xy - 7 4xy - 7 4y

thirc duong

I'( ) = l5x2yZ - 56xy - 32y2 + 35

~ x (4xy- 7)2

7 7 ' V32y2 + 14 , J

Then (-, +00) t hl t(z) = 0 {;;;) .r ;;;; XO == - +. va qua IO tIll f (x)

4y 4y 4y

dOi dim tit am S'I.l:Jg dinrng [len f(x) dat cue ti~ll t~i XO, Suy ra !(:.r:} ~ I(xo) = 5

2xo - 4y => P(x, y, z) ;:: f(x) + 2y?: I(xo) + 2y = g(y) (~)

Xct ham so g(y) = 2y + _2_ + ~ /32y2 + 14. Sau khi tinh g'{y) ta co

4y 2y

.r/(y) ;;;;; 0 ¢'} (8y2 ---; 9) J32y2 + 14 - 28 = 0-

Dl'It t = J32y2 + 14 vai t > 0 thl phuong trtnh tren tr0 thanh t3 - SOt - 112 = O.

5 s

Phuong trinh nay c6 duy nhat mot nghicm dunng t = 8 ¢::} Y = Yo = 4· Wy 9'(-4} =

o. Yen y> 0 va qua Yo th) 9'(y) d6i d!u tit lim sang duong nen g{y) dat CI,lC ti~u tai

86 ChtWllg 1- Biit dli.ng thttc co sa

1.9.3 Khao sat ham nhieu bi~n

Cae.; hal roan mot bien M,lI nhir luon co th~ giai diroc bang dao ham t.heo t:8.t.:l! nay hay each khac, nhung cOng vise tuong I\t dbi '1m C<:'II; hai toan nhieu hii?n ";0 hUH thi khoug <:011 Ut· tH.\ll)!, ulnr trut.k Doi. v(li dit: ham Hhj(·u bi(;n. each lam thong thuong 13 thay thi cac bien tI1 dieu kien bai toan d€ co mot bi~u thuc moi mil. de bien khong con dH~u kifn rang buoc voi nhau sau d6 tim cue tri theo ti:(ng bi~n. B~ dl)c c6 th~ thll,y ra t u lUdng nay trong dlling minh hai toan sau day,

Vi du 1.9.5 (Vi~t Nam TST 2001). Xt!t cae so th'!tc dltdllg a, b, c thoa man diJu ben 12 ~ 21ah + 2bc + 8ea. Tim g~(i try nho nhdt clia lJ'iP1t thut

1 2 3 Pta, b. c) = - + -b + -,

a c

84

Ciutcug 1, Bli t (Uing tht1r: tx! sa

Them chi t i61. cac ban ncn xcrn trong Uti lieu sach giao khoa lop 11 vc gidi han va 12 \'l' duo hiun. Car, ki{'ll I lnrc vi:! dluh ll ul Gall lien qll<!.ll Wi U~~U Ii~ull halll ~b kh6ng chi l'§.t quan trong d6i voi bUt dllng t.hiic rna hflU nhu luon (;(J anh ltilng Ion den llhi{-u nganh khli{: nhau cua toan hoc No cung quan trong voi cl"ilnh cae ban, nhat lit khi ban phfii trai qua cac kt thi toan quail trong.

1.9.2 Khao sat ham 86 m¢t bi@n

Dinh Iy Fermat (hay con g9i Iii nguyen 15' el,tc tri) Iii dieu kl~1I d.ll. thuung dlrtjc ap dung trong cac bai loan tl In cue tri _ Ta lam qUCJl voi phuong phap chirng minh nay qua vi du den gian "iJ.U,

VI du 1.9.1. Tim ,qid tn Ian nlult va nho nhrit elta bie'u thitc

x2 + 2

/(.T) = ( )2 l'

;1: + 1 +

Loi CIA!. Ta co

/1 ) 2x((x+ 1)"2+ l} ~2(x+l)(x2+2l

(x = -~-____:_-..:..--::...._~__:~-.....:.

((x + 1)2 ... 1)1

t(x) -= 0 ¢=> x(x + 1)2 + X "" 2x + 2+ x2{x+ I) .g" 2x· -1 2x .z;: 2 + 2:r + .1:2

~ :r2 = 2 ~ x = J2 v -J2,

TI'c do suy ra gi<i trj nhe nhll.t cua ham s5 l~ (I +~)2+ I ' con gill. tr] Ion nh!t cua ham s51a (l-Al~+l" 0

Vi d'1,1 1.9.2. Tim giti tri nh6 nMt Clill. biifu tMc x;£, vdi x ia m9t s6 lh'!,tc dUdn,q,

LOI CIAI. Xct ham

J(.T:) = x" = e"IUI

j'{x} = e:t"l"l:(lnx + 1} f(;r;)=O~lnx-= -1 ¢=>-T=l/e"

Lap bing bien thien ta suy ra

1

f(x) ;::: f(l/e) = 1" 0

c"

82

CJliidlJg 1 B~ t dAllg tt: ((c co l>"d

Vi du 1.8.14. ChUng minh bIlt dang thuc sau 1!{n moi day so thl,lc Xl, X2, .. ', X"

( )2 ( )2

2 Xl +:1:2 Xl + X2 + ... -j Tn :l 2 i

Xl + ---2- + --- + --- ---n--- < 4{.TJ + X2 + --- + x,.).

01) do

(.TJ+X2: +.1:k)2< (fJ+rt2+---+n.k 2 1''l+rt2+···+ftk 2

k2 Xl + k2 X2 + .,.-j

O:j 0-2

I1'J+rr2+---+(lk 2

+ z Xk-

k rrlo:

Cho k chay tu 1 tOi n roi CQJlg v6 n bfi.t ding thuc dang tren 1(1.i ta dl.tQ"c

'2 (Xl +:1:2)2 (X1+X2+.,,+X.,)Z 2 2 i

Xl + 2 + ,.- + _ n ::; IITJ + ')'2X2 + .. - + I"X"

1'rong rio c~ic hf~ sf, ')1, dlf(W xae \Ii rib bOI

u,+n2+, .. +uk oj+nz+·'·-!-O'k+1 _ Hj+u2+ ... +Un

II.: = k20:1-; + {k + 1)2(1" -t- --- + n2n-k

Ta chon

1 ( 1 1 1 )

i~ = etA- k;;j2 + (~~ + 1)"/2 + ... -+ n:.J/2 .

Chu y rAng

1 M-/k-~

-----====--'--r===~==-

M V(k - !)(k +~)

1

Ju. - ~)(k + ~)(Ik -~ + M)

1 >--:2F!2

1 1 1 1 1

=> - >--'+ + +--

r;--t ~ - 2kJ/2. 2(k + 1)3/2 ., 2nJ/2

Vr-,-2 yTl+'2

~ 11;- :S 2 = 2( Vk + Jk=l") ::; 4.

CltJk-~ R

Khi 06 giA tri nhc nhat cua bi€u th\i'i,: 13. nam-1, xay fa. vdi

a a

Xl= ,~,X2= 1~,--,X7L=

no~al m-{<U2

o 1'?t-~'

a

80 CllltOllg 1_ BAt driIJg tnur ca so

1.8.3 Can b~ng h~ s6 vdi ba.t da.ng thuc Cauchy - Schwarz - Holder

Vi' d \1 1. 8.11. GiQ st!' z, 1/, z ;:::- 0 va x _._ y -+- z = 3. hll!! tim _qid try nhJ nMt eua

r ,(11 CIA 1- CbQD cac sO a, b, c duong va a + b + c ;;;;; 3. thea biit d.iing thl'i(~ Holder (.t.4 + 2y4 + 3z1)(a4 + 2b"~ + 3c4)" ~ (aJx + 2bJy -t- 3c3z)4_

Cbon 0" b, c sao cho a3 ;;;;;; 2b3 = 3c3 = k3, khi d6

(3kJ }4

(a4+- 264 + 3c4)J·

Xet dit',1 ki¢n dang thirc thi ~;;;;;;1j_;:::~=x+y+z=l_

abc a+b+c

Do vtfr._y ta c6

Ti't do de dang suy ra k{.t qua hal toano []

Vi d \1 1.8.12. Gia.stl Xi f Xz, "., X'l iii. dic s6 tin«: dudng co tang biing n, aI, 0.'2, , .. , an ld cae hang s6 dUring cho lrr.tr}c. nm gid try nho nMt CUll. bilu thuc

T1'07l-g dO m > 1 iii mgt so nguyen duonq cho tT1JrJc.

Lor GIA!. Bai to an t5ng quat trong trireng hop nay cung duuc chung rninh hoan t01\i1 tll'on~ tl,t nho bAt d:lng: th,-W H older, Ta chon

n

(I, "'" 1 1 - I'

?n -V<'l + '" vai + -,- + ~

Bay gia chung ta hay xet df!.n mot v i du kho hem

78

Lei GIAI. Sa dung hitt diillg t hrrc AM - GAt ta c6

x~ + a'l > Zax y2 + u2 :::: 2aN zJ + b~1 + bJ :::: 3b2 z

f)<ing thU:t: xuy ru khi va rhi khi

{ ~{

.r = 11 = u. z -= b 2a = 3b2 xrl,l+z=3

.u = y = a, Z = 2b 2a = 3b2, 2a + b = 3

Do d6

b2 b b -1+V31

3 - =3-:=;. = 6 '

3-b 19- J37

u::::;:--=---

• 2 12

San nily Ill. m(lt bAt (Hin!-\ tlurc trong fl} tin V:'ItEO. 1:\ bttt rlimg tluk rl;;Ul).!, tAn)!, quat cua kl thi chon doi tuyon loan Viet Xam nam 2002.

Vi" du 1.8.9. Cho a, b, c liJ. oh: s6 th1/c duoru; va :r,]j, z ~ a la cdc biln so thou man ax + by + cz = TY"- Ch1'fng min" rdm) tAn tlli duy nh&t so 11011(: dudn,q d sao clio

2 1 1 1

- = -- + -- + --. d a+d b+d c+d

Va khl do hay chUng minh 9-id In nho nMt cua birfu th1~C x + y + z Mng

J d{d + a)(d + b}(d + c).

Lor GIAI. Y tll'dng chinh trong chirng rniuh bai roan trcn Iii. dira v11.O lide lucng

(a.T + liy + cz){.o:: + Y + .if I!JZ

Hong bAt ding t hiic AM - GM, tuy nhien phil Ii). bat dfing tllue AM - GAt suy rong voi $6 mtl tllI,(C Khi do neu gia IIi nho ulda cua bi~u thtrc !rbl bang C t hi gia trj nho nhiit cua x + y + z trong mien xac dinh cua bai toan cung bang C.

Khi <16 gj:'i. tr] 11M nhat cua bl€U thUl; la. no,m-l, xay HI. vdi

a a

XI = ---,X2 = ---,-_.,X" = -----==

m -{fa! "'-~

o :r'I~-vra;'

a

1.8.3 Can bllng h~ 55 VOl b§.t d~ng thtrc Cauchy - Schwarz - Holder

Vi du 1.8.11. Gia ~,ti' z . y,;; ~ 0 va x + y + ;; = 3, hay tim _q-id tri nhJ nMt eila ;r4 + ',1y4 + 3z·1_

J "til G IA I, Chon cac so a, b, c d ircng va a + b + c ;;;; 3. thea M.t dAng thtl'c H older (~4 + 2y4 + 3z4){a4 +2b4 + 3c4)J ~ {aJx+ 2bJy + 3c3z)4.

Chon a, b, c sao rho aJ -= 2bJ = 3c3 = k3, khi do

(3k3)4 fa"4":t-2b4 + 3c4):1·

Xot di~l.I kifn ditug t lurc t hl ~:;;;;;~=,::=x+y+z=l.

abc a+b+c

Do ~_y ta (:6

Tu do de dang suy ra k~t qua bai toan, 0

Vi du 1.8.12. Gia 5U' Xl, X2, .. " Xn lil. cric s6 thllC dudn,q co t6ng biLng n, aj, (.!2 > •• " a" lli cdc hiing 56 dUrJn9 (:ho 1,,«0('. Tim gia try nho nhat C1ill biiu thUc

Tnmg do m > 1 la mpt so nguyen dU!ln.q cho truac.

Loi G IA I, Ba: to an t.Ong quat trong trltung hop nay cung duuc chimg rninh hoan toan Won!!; t '.i nilo bitt <tAn p, t I nrc H a! d er _ Ta cho II

n

«= 1 1 1 '

m ,y<ii + ." Ya2 + --- + ~

Bay gid chung ta hay x(>t dOn mot VI du kh6 hen

78

Lo: GIAL Sll dung bat clA.ng there AM - GM to. c6

x2 + 02 ;:::: 2(1X y2 + {Ii ~ 2ay Z:l + If! .._ bJ ;::: 3hzz

Ding thu~ xay fa khi va chi khi

{ ~{

.r;;;;;;.1/- a, Z = b 2a"" 3b2 .T+y+z=3

.1: = y :;: (I, ;: = 2b 2a = 3b~, 2(1 + b = 3

Do d6

3b2 - b "'" 3 ~ b = -1 + J37 6 '

3-b 19-V37

a=~-=--~-

2 12

San nf~~' lil nt(ll bAt rl:hn!-!, t,lu"rr t rnn~ M t.hi V:\JEO. I;;, b~t dAnJ.!. t IH-r~' ct',!.n"" t";n~ quat cua kt thi chon dOl tuycn toan Viet :'\arn narn 2002,

vr du 1. B.9. Giro (1, b, c [(i cdc s6 thl/c dIJdn,q va J:, y. ~ >- () iii f:lic b'i€n s6 Ihoa man (u + by + C2 = :ry z , CMrn.Q min h rdnf) Ion !9i dU1J nMt 86 tlu«: dUdng d sao rho

2 I 1 1

- = -- + ~- + --.

d n+d b+d cld'

Va khi do hdy ch1ftlg minh gid tq nho nMt ctia bi€'U th,uc x + y + z bling

Jd(d + a)(d + b}(d + c).

Loi G IAI. Y Wang chinh trong chirng rninh bai toan tren la dira villi I1cJe hrong

{ax + by + C.l)(x + )/-1- Z)2 xi(Z

bAng bAt ding tll\lc AAI - GM. tuy nhicn plu\i l1l b~t d&ng t lnic AM - GAl suy rong vdi $() rnu thirc. Khi UO liEu gla tri nho nhat ella bi6u tlurc tren bl:ing C thl gia tri nh6 nhat ella x + 11 + z trong mien xac dinh cua bai to an ding bilng C.

u5

Vi dl,l 1.6.5. ChUng minh ri'mg udi mot s6 ihuc khOng am a, b, c

Cb(mg tao hay xern xet them mOt dang bat d~ng thuc d6i x(tng 3 bien khac. Bai toan sa, I day kha c6 di;'in va co nhi~u (ntg dung

Vi du 1.6.6. ChUng minh vUi 11I<J~ a, b, c khong lim ta luon 1;0

abc? (a I-b- c)(b+c-a)(c+a-b).

Lo: CIA!. Day Ill. mot bat dr-tUg thuc ra.t quell thuoc va. don gian VI chung minh chi din dua vao bat ding thuc AM - GM. Hien nhien bat dAng thirc dung n~ll mot thira ::;6 nao do iJ ve trai am. Ta xet khi ca 3 ~ hang do deu dirong. DE} tbay hAl dhnp; tluk t.r't:1\ (htW suy ra bi:ing deh nhan r.i'I 3 b<'i.t. drlnK t.hdr. san theo tJ'tng vt

(a+ b , .. c}(b+ c - 0) < b2 (b + c - a)(c + a - b) ~ c2 (c+a - b)(a+b - c)::; a2

Co the noi VI du i.6.6 kha clan giaij so voi b~t dAng thlie Schur, nhimg thM ki la la 2 LA.t tlil.ug tlnre nay lioa ra lai iA !HOt khi ta chudn hOa hoac khai trj~n chUng?! 0

Cac bat ding thuc thuan ntdu 111 d6i tl.tt;lng chu y{·u cua bat ding thm.:, Mu het cac bai thi toan Olympiad d~u xuat hi(>11 dlf(jj dang nay. P hAn con laj la cac btu dit.ng I.luk ldu'lnf.'; timAn nh!u. ((ti~i xlmg hn~~: khong) tltong d{ji ft va dtng dlfQ<" ft chi! y hen Tharn chi nhieu nguo-i con tin riing u~ mot b&t dd.llg thlic dung t hi chung buoc phai thuan nhtit (d6ng b!).cJ· Cac b~t d:ing thuc khong thu~n nhli.t Iuon rat d~c hiet vs dep mat. Hai VI du sau dliy ~ lam to tllf.1I nay

Vi du 1.6.7. Chung minh vdi mpi a, b, c thr,tc

ai + b2 + ~ + 2<1bc + I ::::- 2(ab + be + ca).

VI du 1.6.B. ChUng minh viJi m9i a, b, c thuc

Salt day chung ta se xern xet cac b§.t u&ng thuc d6i xirng co dieu kien, va mot ki thll(il quan trong d~ chung rninh bAt dang t lll~~~ : kl thu~t chuifn hoa.

Jab + be + ca < 1 (a + b)( b + c)( c + a) .

3 - 8

1.6.2 BAt ding thirc d6i. xUng co di~ll kien

Cac b~t da.ng thuc doi xirng c6 dieu kien va khong co dieu kitn La 2 dQi hLQng rieng rf tOn tai doc I~p nhung that ra lei ('6 mai quan h~ chat che voi nhau. Chinh mOi quan III; Hay larn nay sinh mi)t ki thuat lIloi dHrllg minh L&l ua.ug thlt~ : UU li\ ki thuat Ch1tdn hmi vO'i bAt. ding thu\: thua.n nhut ubi X U-II !!;_ Sl;I.U <.w.y la. 11I0t s6 vi du

Vi du 1.6.9. Chlmg minh rCln.q vdi nwi a, b, c kh/jn,g am thi

LOI GIAI. vr du t ren iii. m¢t bfi.t cling thuc I-lit n5i ti~ng va khong h~ de. Cach thong tblfi'Jng la luy thira mil 6 ca 2 vA rc1i khai tr!cn. nhirng each nay ra.t dai va khong nhicu .y nghla, tharn chi rat de nhAm Ian. Ban hay ki xcm clurng m inh sau

Cia SLr ab + be + ca = 3. khi d6 a "1"" b + c ~ 3 va abc :5 1_

(a + b)(b + c}(e + a) = (a + b + c)(aD + be + cal - abc __:. 3(a + b + c) - abc :2: 8

jab + be + m 1;> I«~- + bY{b + c){c + a)

~ =1< -

3 - 8

Bit Jillg; tlnrc t.lut)~ t:hifllg iuinh, Dang tlnre xa'y ra klii va chi khi a -= b = c. 0

Di~m dang chti y trong Ibi gilLl trsn Iii vi~ gia su- ab + be + ea = 3. Ta gia sit dtio"c nlur v(iy vi bAt dim~ tlnrc t ren Ih thuan nMt- Th~t v;ty, It..y aJ = «it, b' = bit, d = cft

1'6i chon t d(i; alb' + b(d + da' ;;;;; 3. Ta tim diroc t = J rJ.l>+";,+ro _ B§.t dAng tMc dung voi a', b', c' nen hi~n nhien n6 cung dung voi a, b, c sau khi nhan ,/, b', rl vdi t.

Co the each li Iuan tren kh6ng hoan toan de hi€n d6i vdi ban doc chua quen vdi phmmg phap nay, nhung thuc HI (_jay Ih. m¢t .. .all d~ [al don gi';'_n y~ I;a.t.: b.,.11 dill phiii n~m roo Ta hcan toan c6 the; gift sit b1lt ki met bieu thuc nao khac, chang han a + b + e ;;= 3 hoac (1 + b + c = 1 'hoac abe = 1 ... Dit·u d()c dao va cung Iii dicu kho nhat cua ki tbu~t Hay la viec dlU.1n hoa bie\) thuc nao cho hop If nMt de co chung minh don gian nhat , Chang han trong vi du trtln, ta hoan toan co thJ giii su- a + b + c ;;;; 3 hose abc = 1 boac ((l + b)(b + c)(c + a) :::::; 8 nlnrng cac each chuan hoa nay, hcac khong the 1"(1. (iltt1{;, hoac phai clurng minh ra.t dai,

Vi d1,1 1.6.10. ChUn9 mmh. nl:ng 'Urn mpi u, b, c khOng lim ta i/Lon cd

Lot GIAI. Y tuang til nhien khi giai bai toan nay Iii. tim each loai b6 dau call b~c 3 lieu ('6 th~. BAng each do, ta thay d6i b§..t ding thuc tren thanh mo-t bAt clAng Ihue

67

co diil-u kien nhir sau :

Cia $I( (a + b)(b + c){c + a) = 8, hay chung minh .

(11(b + d + bZ(c + a) + c2(a + b) ~ 2(ab + be + c~).

Cong \'i¢~ dJil Iai !ra non UtllJ gian han. du Iii chirng minh 2 bat d~ng thl~!;

ab+ bc+ cu S 3,

a2{& + c) + b2(c + a) + c2(a + b) ;::: 6.

Th~t vay, voi bAt dil.ng thuc thir nhat hay chi! y rang

8 + (Lbc

8 = (a + b + c)( ab + be + m) - abc ~ ab . f be + fa ;;;;;; b

a+ +c

Theo vAt uimg thul: AM - OM thl

Do d6

8;;;;;; (a + b){b + c)(c + a) 2': 8abe => abc '5 1,

(2(a +h+ (J):J

8= (a+/))(b+r')(r'+a}:$ 3 =}a+b+c~3.

ab + be + cu "5 (8 + abcl/3 -:; 3 .

Biit d&ng thU'c dii duoc chung minh KOUg. 0

Vi du 1.6.11 (USA MO 2003). Chl1ng minh Mt diing thuc

{2a + b + c)"2 (2b + c + a)2 (2(' + a + bj2 8

--"""'"-,-+-- + < .

2a2 + (b + c)2 2b2 + (c + a)2 2('2 + (a + b)2 -

Trong d6 a, b, c ld cdc $6 th1jC khong dm-

Lor GIAI. Ta chuan hoa a + b + c ;;;;;; 3 di\ Flit ~'/n cac s(j hang vo t.rii.i tnJ thanh de bit'll! thin- rlon giti.n han dQi voi 1 bi~n r{la a, b. c. B&t dAng thU'c tuong chiang vdi

(3+a)1 (3+.6)2 (3 + c)2 <8

2a2 + (3 - ap + 2b2 + (3 _.b)2 + 2'C2 + (3 - c)2 -- .

Chi'! Y vOl di~l.I ki(,n a -i- b + c == 3, ta sc tim mot 56 tht;Cc k sao cho

{3 + n}2 8

2a2 + (3 - up :::; :3 + k(a - I).

68

Khi d6 Ddt dii.ng thirc sf" duuc chung minh VI

VT :::: 8 + k{a + b + c - 3) = 8.

Ta co

3(3-+a}2 a2+6a+9 80.+6

2a2 + (3 - (1)2 - a2 - 2(.1 + 3 =- 11 (a - If + 2 8a +6

< I + . - '-' 4a + 4.

- 2

W.y k ~ 4/3 va bit dimg thrrc diroc chirng mirth. 0

Vi du 1.6.12 (Japan MO 2002). ChUng minh rli.nq 11m m{)"i a.b.c kh8ng am ta co Mit a&nq fMc

LO! GIAI. Ta chuin hoa a + b + c == 3 va xet fieng tu so hang cua bi.Eu thirc

(b+c-a)2 (3-2a)2 9-12a 1-4a~ (b + c)2 + (.12 = {3 ~ a)2 -+ a2 ;_;;; "9=- 6a + 20.2 9

;=2- .

2a2 - 6a + 9

Ta phil chirng rninh

l 1 1 3

----=~-- + + < ~,

2a2 - 6a + 9 2b2 - 6b I- 9 2c2 - 6c + 9 - 5

Ta ding lam thco phucng phap tlwng tlJ HI. tim s6 thuc k sao cho

1 1

-2(.!-::2'---6a-+-g < "5 + k{a - 1},

Trong bsi nay, sa k khong d~ dang tim nhu truce, Hay d~ y each nnh sau day

5 -1- 2a'l-6a+4 _ 2(a~ 1)(a-:2)

2!l2 - 6a + 9 - 2a2 -.6a + 9 - 2a2 - 64 + 9 '

Khi a = I th'l bi.3u thirc (a - 2)/(2a'l - 6a + 9) c6 gia tri b~ng -1/5, vf;.y ta siS du dean rang

2(a-l)(a-2) -2{a-l)

. . ... _- > -.~-.--.

2(12 - 6a + 9 - 5

126

V~y hAt ding thlic ducc chirng minh xong.

each 2. DM k = a + b + c. Su: dung hilt da.ng thuc quen thuoc abc. ;;:- (a + b - c)(b + c - a)(c + a - bl => abc ~ (k • 2a){k - 2b)(k - 2c).

Rut gQn lai dUt,ic

9

4(ab + be + ca) - k2 ~ "kabc (~)

Bat dang thuc ~~~i.a bai toan wong ,-hwng vtJi

(IL + b + (;)2 + 2abc + 1 ~ 4{ab + be + cal <rl- 4(ab + be + cal - e ::; 1 + 2abc.

5u- dung (;I<) , ta chi can chimg rninh bat d~ng thU'c sau

(~ - 2) abc < 1.

k --

Thoo b~t llft.ug th(n; AM . G M ta c6

(~ _ 2) abc < (~ _ 2) k~ = (9 - 2k)k2 < L

k - k 27 27 -

V~y ta c6 di~u phal chung minh, DAng thuc xay ra khi va. chi khi a = b = c = L 0 Nh~n xet. Mot bai toan 511 dung phirong phap nrong til nhu tren cua cac tac gia Marian Tetiva, Mircca Lascu, Gabriel Dospincscu

a2 + b2 + c2 + 2abc+ 3 ~ (1 + (1.)(1 + 1»(1 + c),

Bld toan 2.30' (Pham Kim Hung), V6'i a, b, cia cac so th1tc dl.tong dwng tu.ff !i, cht?ny minh bUt dang th1k

a + 2b + b + 2c + c + 2a > 3.

c + 2b a + 2c b + 2a -

Ldi CrA!. Quy uong mau !:la rbi tilt gqn cho ta bdt da.ng thuc 2(a3 + b3 + c3) + Jabc ;:: 3(a;2b + b2c + c?a).

Khong mAt Unh 1608 quat, giA su e = min( a, b, c). Dnt a = e + p, b = c + q{p, q ~ 0) > Thay vao bat d~ng thue tren ta dli<;1c .

VT = 9x~ + 9(p + q)x'2 + {6p:i + 6q'1. + 3pq)x + 2{p3 + q3), V P = 9Xl + 9(p + q)x2 + 3(p + q)2x + 3p2q, .

VT - V P = (p2 + q2 - pq)x + (2p3 + 2q3 - 3p2q} "2:': ° vi p, q ~ 0, x ~ 0,

124

C'hlMng:1. Sang tlW bAt dAng thtk

N~u xy ~ () thl ta c6 ngay di~u phai chimg rninh thee bAt ding there AM - GM X4 + y4 + 2x3y :::-: y4 + 2x3y = ~IYI4 + ~ lyl4 + 2lx:iyl ;::: ~lxif!1 ~ 2~y3.

Nell xy ~ 0 til. ermg r.lll'mlo'; nunh t.IKJn~ t l.t. khi do

I

X4 + y4 + 2x3y _ 2J;y~ 2 .:rA + 21xy31 - 21x3YI :::>: o.

V~y bt..t ding thac tli!n da d\1l;lC chirng minh, tit d6 suy ra ket qua. cua bai toan, 0 Nh~n xet. Bit ding tht'tc sau vau dung, val lC1i giru hoan toan nrong tI!

Ngoai ra n ;;,;; 3,5 la 2 s6 nguyen duy nhat c6 tinh chti.t

a(a + b)" + b(b + c)" + c(c + a)" 2: a 'Ita, b, e E R.

Ban doc hay tu elnmg rninh 2 tinh chAt nay.

Bai tolm 2.27 (Mathlinks Contests). Ch'lffi9 minh rli.nq neu a, b, c Iii cae sli th:l1c

d?Iang co Uch bdng 1 thi .

/ffi+c i2t+a 3

+ --+ --> .

b+l c+1-

LOr GIAI. Su- dung tnre ti~p bit ding thd'c AM - GM cno 3 s6 hang tre-n, ta sf chtmg rninh

(a+b)(b+c)(c+n) ~ (a+l)(b+l)(c+ 1).

Th~t vay, cung voi di~u ki~n abc == I, hAt dang thuc tren tuong dinmg voi

ab(a+ b) + bc(b+ c) + ca(c+a);::: a+b+ c+ ab+bc+ca.

SIi: dung bAt dimg that AM - G M v(!i nhorn 5 so

o?b + (J2b + a2c + a2c + be:::: 5a, b2a + b?a + b2c + t?c + cc :::: 5b, c2b + r?b + ~'la. + c2a + ab ~ 5c.

Cong 3 M.t dil.n~ thuc. tren l~i

2. VT ~ 5{a + b + c) - (ab + be + ea) (1)

123

Si1 dung cac kft qua rren, ta co

1 1 1 I 1 1

{l + a)2 -+- {l -+- b)2 + (1 + c)2 -+- (1 + d)2 ~ 1 + m' + 1 + n ;;; 1.

D6 18 dieu pbiii chang minh D~ng that xay ra khi va chi khi a ;;; b = c = d =- 1. [J BM toan 2.25, (IMO Shortlist). VGi a, b, c, z, y, z lil cae .,,6 th1,tc kh6ng tim thoo man a + b + c = x + 11 + z , ChUng minh Mt ddng th1Zc

ax':! + by2 + cz2 + xyz "2" 4abc,

LOl CIA!. Tnroc t){;n ta sc chUng rninh rAng

BO dli. N!.u ax2 + by2 + (;Z2 + xyz ;;; 4abc tlti :r + y + z ~ (l -t- b -+- c. Thoiit ~y, vdi dang thue hen ta co

x'l. y'l. z2 xyz

4bc + 4ca + 4ab + 4abc = 1.

W.y ton tl;\i ID¢t tam giac ABC thoa man

x y z

cos A = . JT::' cos B =-- rx:: (:Q~ C = rit:

2v be' 2yca 2vab

u..y a) = Va, b) = -Ib, CI = .;c. Bat llimg thlli: dua vf: J:;mg quen thuvl: a~ + b~ + c~ ::?: 2(a1bt case + b1Cl cos A + Ctal cosB),

• . . • k1 . x + y y + z z + x . L _H

DAng tht'rc xay ra khi va ell! 11 c s= -2-,b;;;;;; z.: -2-'c = -2-' Bal toan uan <..IJ:I.\1

lit h~ qua true I i~p dla. bAt liang ttnk tren Q

Bal tOM 2,26 (L, PanaitopoI, Mlcea Lascu, V. Bandlla}, Chitn_q minh nin9

vrJi mQ1 a, b, c E R ta l1Liln co .

Lo: GIAl- Dat z = a + b, y = a + c, z = b + c, t.a. phai cmrng minh x~(x +y - z) -+- y3(y + z -;I.') + z~(z + x - yJ ~ {)

~ x4 + y4 + z4 + ;;?y +.y~z + z3x - xy3 - yz3 - zx3 ?': O.

Ta tim each nh6m thanh esc t6ng nM hdn ciia 3 sO hang. 0.,1 the: ta. S6 chrmg minh bat ding th lee

69

Rat dang thirc tren du<,)c chung minh nhu ::;<\1I

((1- l) (5(a - 2) + 2a~ - 6a +9) :::: 0 ¢;> (a - 1)(2a~ - a - 1) 2: 0

~ (a - 1)2(2a + 1} ;::: 0 (dungl).

Tit do suy ra

1 1 2

2a2 _ 60. + 9 :S "5 - 25 (a - 1)

Thay a bdi b, c rbi cong vt> cac bat dimg thuc ta c:.h.r($<; dpcm 0

Cac vi du tren la cac vi du di~n hinh efta ki th11(tt l;hu~n hoa giup chung ta co (hl~c 1I1i gih-i ngiin mn vit (t(11] giiin- Trong httn h(~t rfie trH?ing hop k!J{u: t hi ('{u· clil,u kien mang tmh htnh thuc nhieu hen, We lit d~t di~u kien truce dJ co mot bat uling thrrc dl nhl.n han.

Vi du 1.6.13. Gid su cae 56 th?Jc a, b, c thoa man a2 + b2 + c2 = 3, Hay r;hJ)ng minh

LOt GIAI. Th dira bfi.t d§.ng thuc vl: dang ihudn nh&t la.

o,3(b + c) + b3(c+ a) + e3(a + b} s ~(a2 + b2 + e~)2 ¢:> 2(a4 + b4 + c:4) + 4(a2b2 + b2c2 + c2a2) :::: 3ab(az + b2)+

+ 3bc(b2 + e2) + 3ca(c2 + a2).

Val bAt tli.ng tb{(t; trf-n ta chi dill hh:h ra clnrng miuh don gian thea 2 bien

a4 + b4 + 4a2b2 ~ 3ab(a2 + b2) $} (a - b)4+ab(a - b}~ ~ 0,

Bllt dil.ng thirc Jil c:hr(k: chung rninh xong. Diing thirc xay fa chi kbi a = b == e, 0

Vi du 1.6.14. Chltng mink vdi cac s6 thuc dlld'ng (I, b, c thod man ab + be + ca = 3 ta co Mt dang thuc

a2{b+c) + b2{c+a) +c2(a +·b} ~ 3V'(a + b){b+ c){c+a).

Leu GIAL BI;I-n dQC de dang nhan ra day chinh la la vi du 1-6_10 va viec them dif!u kicn ab + be + ca = 3 chi mang tinh hinh th (rc , Rieng vCl'i bat d~ng thirc tren con (6

70

CJnWJJg 1. B~ t dlfllg !h de cd !iii

them mot each ch ung mi nh 1:.5 dii:n mra hl'mg each ~0: d \lng hat dang th!k Holder, Chu y r~ng 3 bifu thuc sau boan toan nhu nhau

a2(b + c) + b:l(c + a) I c2(a + b), 62((" + fl) + .;2(a + b) + a2(b + c), ab{a + b) + bc(b + c) + I':a(c + a),

Chi cAn ap dl,lng bat ding thuc Holder cho 3 Il(? s6 tren ta co dpcm. 0

Vi du 1.6.15. Chu;n9 minh rdng n~11 (I, b, c Iii cdc so tht,fc khanq am co t6ng Mtul2

ta lu6n co

La! GIAI. Theo hilt dAng thrn- Schur b:;ic 2 ta co

a4 + b4 + c4 + abc(a + b + c) ;:: aJ(b + c) + bJ(c + a) + c3(a + b)

¢:} 2(a4 + b4 + c4} + abc(a I- b + c) ~ (a3 + b3 + c3)(a + b + c).

Thay a + b + c = 2 vao bat dii.ng tlllt(~ cnfJi ta l>U_V ra k6t q1lio bhi toan ril'l ,v rAnI!; ding thirc co x<iy ra 0 bai toan tren khi a ;;;;;; b "" L c = 0 hoac cac hoan vj (ngoai truong hop t~lJl dntang a "" b = c = 2/3). 0

Vi du 1.6.16. Ch11n,q minh rdn,Q voi cdc -1'0 kMng am a, b, c cd fling Mn.g 2 thi a3 + /} :-I- c3 + 15abc/4 2:: 2.

Ldl CIA!, Thirc chat, ta dn phal chlrng rninh bai toan

J bJ 3 15ahc (a+b+c)J'

a+ +c+--> .

4 - 4

D5i voi bat dAng thuc tren, kbai tri~n d 2 ve. r61 rut gon ta dUQ<: bAt dAng thin:

Schur qUC[) thuoc: 0

Thy nhien dOi vOl r~t nhi~u ba! toan ban g{lp trong cu6n sach nay thl di~u kifn khong chi Ie. dl@.u kl~[) rang bucc voi cac bien, rna vi~G dua v~ dang thuan nMt khong ph;\] luon gil'tp bAt dAng tJI11c t,TO n(~n ddn !l;llln han, Vi du nl111

V ( du 1.6.17. VOi cae so khOn.{l am a, b, C co t6ng MnQ 2, hay chung minh (a2 + ab + b2){ll + be + ('2)(c2 + ca + (2) ::; 3.

Vi du 1.6.18. Vdi cae so tlqtc khOT!9 am «, b, c co tdng bdilg 2, chUng minh din9 28

(a2 - ab + b2)(b2 - be + c.2)(C2 - ell + a2) :-:; 35.

Trang ca 2 VI du trcn, vi¢c tIman nhat 2 v6 khong co y nghta, chung ta can 511 dung cac phircng phap khac, do la d&n bi~[j va phuong phap cO dien. Cac phuong phap nay .'iii dircc xeru xet ki luang trong chuong IV,

1. 7. BAt dAng thll'C vii ntt. Qfl th,k d(Ji XIi':!Jg sa cAp

71

1.7 Bdt d§_ng thuc va cac da thirc d6i ximg so cAp

1.7.1 LI thuyi-t v.§ cac da thtrc d6i xtrng sd d.p

K IIi noi t.(Ji 11 hii'ng bieu t IIl(C' rlQi xl·rn~ np.;lIui t /l t.lnrn-n!'; hay xoru xN t.iJi di.e r\a thtrc ubi xirng so cap D6 Ill. cac da tlnrc cua cac bien XI, X2 > •• ,' x" co dang sail

Sl = Xl + X2 + ... + X"

S2 = X1I2 I X,:Ca l ... i ,T,3:" j. X'.!X3 + --. + X2X~

SI; "" L XqXq·· .. 1"l. ('1,;2 .. · ',;k)C { 1.2 .... ," f

Noi cadi kliac, clruug iii. h~ t::u rrong khai tri~1I cua elk da tlnrc

f(x} "" (x - xti(x - Xi)--·(X - ~tl)

"" x" - s,x71-1 + ... + (_l)kSkXn-1r + ... + (-Its". g(x) = (x T :rJl(:r + :r2) .. ,(:r + :r.,,)

= ,1:" + S,x';~1 + ... + SI;X,,-k + ... + Su.

Viec nghien ciru cac bj~u thitc dbi xirng co th~ lht(,!c quy v~ bang vii,lc nghien cuu cae da thirc dQi xung drive dl«(ng minh qna dinh Ii sA-II dAy

Djnh Iy 1.10 (D~nh 11 ell ban cira dai 5&). M9i do ihUe dO-i xtrn9lu017. co thi /.Juju di.[n IjU(l. (.:Ur:: au tiL-lie: d(ji ;iring sa eJp.

Dinh If tren khong chi co nhieu irng dung trong bAt diing thirc rna no ton co rAt nhi~u ung clung trong dai W noi chung Tht nhien. dinh Ii co th~ chimg minh vci ki'~'11 th(k ~d d.1J ulurug khoug t huoc v~ Lat lIaug' t lnrc Jl~n tac gia khong ui?u l~H ;j da.y, coi do nhu mot hili t{\P dal s6 cho cac ban. Tli dinh If nay, tu (.;6 th~ xem vi~c chimg minh mot bat dimg thtrc doi ximg ve vi~c.; danh gia giUa eac da thirc dbi xung so ciip. Ri€ng vOl esc da thirc nay ta c6 mot s6 chu y quan trong.

V f d J,l 1.7.1 (BAt dAng thu-c N ewton), Wi m(Ji Xl, X2, -- " Xn khong am ta c6

72

C1HfO'ug 1. BAt clAug thuc ca ~a

Vi du 1.7.2 (BAt dAng thac Maclaurin). V6i m91 XI, ;1:"2, ••. , Xu khong am to. co

Cl'I (!NG M I!':H. Chirng minh c3. 2 bM ,iAn!'!; tln~' trfn hhn!l; elm)!, mot plllmD!l; phap nhir nha.u, d6 la phirong phap giam bien dira VEIO dinh li Roll.

Xet da. thuc

g(x) = (x+ xd(x + X2) ... (:r +:r,~)

== .T" + SI.T,,-I + . __ + SkX,,-k + ... + Sa'

'( ) ,,-I + ( 1\(.' .. -2 (k}S ,,-k-l S

g X :::: nx n - I.JI;r. + _ .. + n " Ir:X + --- + ,,-I

= n(x»-l + S~X"-2 + __ . + S~xn-k. I + .. + S~-d, nS~ = {n - k)SJ.;.

Tlieo dinh Ii Roll th'i phirong thlg{x/ =-.: 0 co dll n-l nghiem thuc -X'I' -x~, --', -X~._I va do do

g'(x) ::= n(x + X'I )(x + x;) ... (x + x:.).

Vij.y 3S~ Tfk = 1, n - 1 Ih efi.e rIa ti1l1r dr.l xltng ~('j rap dla. Xl' x~, ... , X~'-l' No. each khac, tan tai d~ sao cho d~ ;:;;;; d~_I' Sau mot s6 hihJ. han Ian giam bi~n nhu v~.y, ta chi dm xtt 2 bit dAng thrre trcn trong trtrong hop co 3 &6, Nhimg trllOng hop nAy rit don glaD vi ta co ngay

3abc(a + b + c) S; (ao + be + ca)2,

a" b + c lab + br +.ca ~rTb

> V > v coc,

3 - 3 -

CA. 2 blit ding thirc tr€n a& dlr~c clnmg minh xong. 0

Ti1 phirong phap chirng minh biit d~ng thul: t ren ta rut ra hi: qua sau

BO de. xet Mt ddng thlic f(dl, d2, ... ,d,,J ~ 0 trong do n ~ k va d, to' cae da thuc d61 TfIn_1} r:~m cae bi~n .w~ dl(I1rI..1} XI, X2, ... , Xfl. Khi do nf1J Mt danf) thue dii:n g uni mQi C:c I, X:l; •.... x,,), Xi 2 0 th1 ciJng dung 'Udi mQi (Xl, X'::" - -', x,,+ d, X1 ~ 0_

1.7.2 Da thuc d6i xU'ng so d.p va cac U"ng dung trong giai toan b.§.t d~ng thuc

Chung ta hay bit dall voi vi du san clay

Vi du 1.1.3 (Bulgari MO 1998). Cho a, b,c '2: [) va abc;;;;;; L Ch1i'n.g minh nlng

1 1 1 1 1 1

---+ + <--+--+--.

l+a+b l+b+c l+c+a - 2+a 2+b 2+c

1.7. BAt dAng thuc vii cac da thtl'c d0i XllllX Sri dp

73

L(11 GIAL D~ cho gen. ta dat S = a + b + c, P = ab + be + ca, Q = abc. Ta co

VT=

1 1 1 01

--~,+ + -'-'-

S + 1 - a S + 1 - b S + 1 - C 0:2

LO + a + b)(l + a + c) "" S2 + 45 + 3 + P

,'111m

0:2 = (5 + 1 - a)(S + 1 - b)(S + 1 - c) = 52 + 25' + PS + P

vp=

'" 1 12 + 45 + P L...J 2 + a = 9 + 45 + 2P

.1:I~)'n~

Ta phal clnrng minh

S2+4S+3+P 12+4S+P

~-----<--=----:--

S2 + 28 + PS + P - 9 + 45 + 2p

P-3 PS-2S-3

<rl- < -----:::---~--

9+4S+2P - S2+2S+PS+P

~ (P - 3)(S2 + 2S + PS + P) S (PS - 2S ~ 3)(9 + 4S + 2P)

** (3P - 5)S2., (5 1)p2 + 6PS ~ 24S + 3P + 27.

Vi abc = 1 nen S, P ~ 3, do do

VT;:: 482 + 2P? + 6PS ? 12S + 6(P - 1)5 + 6S t 2p2 ;::: 245 + :W + (p2 + 6S) :::: V p.

Ddng thlk xiiy ru chi khi S "" P = 3 hay a = b = c = 1. 0

Vi' dl,l1.7.4 (Vi~t Nam MO 1996). Gia su a,b,c,d iii cdc sO thl,tc dUdng thod man dUn ki~n

2(u1J + be + cd + au + ar. + IJd) + abc + bed + cda + dab = 16.

Ch1m(J minh Mt riling th({('

2

a + b + (: + d 2': "3(ab + be + cd + da + ac + bd).

LOI GIAL Thea dinh If Roll, t6n tai cac so x, y. ~ de

ab + be + cd + da + 0.0:; + od = 2(xy + ,1.JZ + zx) abc + bed + cda + dab ». 4:1'y.z

. 4

a+b+c+d= 3(x+y+z)

74

CllU'Gng 1. BAt dillg thrrc co sCi

BAt dang thuc !n) t hanh : Neu :!:JJ + lIZ + z» + T.IIZ "" 4 tid.

J' + !/ + z 2: ,]:11 + liZ + z.r.

Co nhieu each chimg minh bAt dAng thuc ~reIt San day lb. mot chirng minh dung cl'i(~ da thire d6i x(tnp; SC1 d.p.

Vd'i ki hleu d;, d2, d:) nlnr tn!OC ta co 3d'). + d3 "" 4. Thee M.t dling thuc Schur df thay TAng

:l

3dl + d:! ? 4dld~

22 3 22 2 ~

=> 3d1d2 + d3dl :;:-: 3d'.d2 + d').d3/dl ~ 4d2

=> {d1 - d2)3 + 3(df - d~) :;:-: 0 --> d, ~ d2,

Bit diiug thuc um.,lc clnrug minh xoug. D4ug t!J(!t: xay HI. khi a = b = c = d = 1, 0

SCI dung c6.l; da thue d6i xung trong chirng minh h~t diing thirc eo uu di~m la khong tun 111Ji~lI thci giau u':: llghi 1Il0t (all! dl\ln~ minh u{l,c biet uau do. khong !lui hoi nhieu kt nan!l; nhung cling co nhiroe di~m Ill. ue nham Ian va neu 55 cac bien co sa mli cao, tli bac 2 trd len till clnrng minh ~e rlit phric t<;LP, THy nhien, elil sao day van 13. mot ki t hu.(l.t. call thiet di d(; sit dvng. Cal' han hay t huc hanh ki Il11.llg rtlil rnlnh qua bar dimg tlurc khii quon t.lmoc tiA rhrc::w r:hlrng ruinh {) n11_lC triroc.

Vi du 1.7.5. Chung minh u6i ear sa ths«: dltr:tng a, b, c ta /uon co

lab + be + ea < V ((.I ., b){ b t c)( (' -+ (I,}.

\ 3 - 8

1.8 Phu'dng phap can b~ng h~ s6

1.8.1 Bai toan rna dAu

Trong nhieu bai toan bat dang Ul\~( v~ c1,l'c.: hi, dfij lchi viRI: gh~p va. sit dung eae bAt dfulg thuc c6 dien kh6ng clu(,!c thuan lC)i va de dang. Khi su dung lien tie.p nhisu bat dhnp; tllll(' tn luon pliai I~h(l Y Wi c1jl~11 kiOn fIr: bft t di:i.ng I hl{l~ xil.y fa, ni; niGu ki¢n nay luon dU'Qc thoa man su6t qua trlnn ta ~U dung cac bat diwg thirc trung gian, Trang nhieu truoug hop thl h~ 80 uic bat dAng th\'jt lt1t l';-ch, khong d61 xirng nen I,:ung, vi~t.: tre1J trLr Hell fA I kll0 khAn, VI I h~ lIf' chat; ch~n (;0 !!lOt Illi gi~i dl~t. ch& vi! Illj\,t t.oim hoc, dl(Ul)!; t;l b110C' pbAi cht:-t thf:m dw bil~n trnnK gian rili gihi h~ plurrrng trl.nh ella cac dAng thuc.

Vi d u LB. 1. Gu~ su eric ,~(5 thuc khong rim z , y, z tho(l, lIuin di~lI kien xy+yz+zx = L ch'llng miuh Mt ddng th'ue

1,8, Phuong phap CaD bAng h¢ sa

75

UH GIAI. vc mat hinh tbrrc, ta co thE trinh bay ngt..n gall loi giai cho bai toau trcn U~Jlg Lb.t U[\lIg: t lnrc AM . G M nhu sau

2X2 + 2y2 ~ 4xy 8x2 + l/2;;:2 ~ 4.n Hy2 + l /2:;2 ~ 4yz

Cong vi; d 3 bAt ctAng thlk triO-II I~i tli\n d'~Jl

lOx2 + 101,1;> + :;:1 '2: 4(xy + yz + Z'-I;) = 4, Ding t lnre xay ra khi va chi khi

{

.T = 11 4x = z 4y .:= z

<=>

{

x;;u;;1/3 z = .1/3

Dll.y l~ mOt loi gini dep, ngdn gon nhimg c6 v~ hoi thieu tu nhien: Cac ban ~e th~c mac t<;l.l sao lai tach 10 = 2 + 8, clay la ffi(Jt su ngiiu nhien hay may mlin" va neu tach thea each khac, ching han 10 = 3 + 7 li.;ou co giai dinrc khong? Tat nluen, ruoi dich t;i.d~ klH1.r rt('n khfJ[]~ d!Ln Mn kt:t q\la, va. trich 10 =;; 2 + 8 cling khong phai 111. 51! may man ...

Vi d 1,,1 1.8.2. Giri. ,~1[ ('(ie sri tlua- .1', y, z tho« mnn 2.r1jz =;; 3:r2 + 1.vz + 51'2, Tim {Iii! in nhci nhO.t c:ua

P =;; 3x + 2!J + z .

LOI G IA I, Dil,V ,'.ill1~ b. mot bai to an khf\. kl 1;,1. dtc hii s0 I('(:h nhau nlnrng gia I f\ nh6 nbut lui dat duoc khi ,r = !f = z = fi l? 0

Ki thuat can bAng he so ia mot ki tllll~t rilJ cAn thi6t, thirong Stt clung mac cill doi lux til. co du: phuong trlnh kha phirc tap. Nhung vOl each bai toan khong a dang chuan, v{$i cac bi6u thuc tech nhau till tUllg vi¢c nay dirong nhu bitt buoc. Trong kI thuat can bAng h¢ s6, ta eo th6 phan chia lam 2 nh6m trong thuc hanh, do la l:all \.l~llg b~ !:ill vui Mt !.liing thlle AM - GM va din Lt.llg h~ sO vui !Jat !.limg; th(!\:

Cauchy - Schuiarz.

1.8.2 Can b~ng h~ s6 voi b.it d§.ng thuc lien h~ trung blnh cQng va trung blnh nhan (AM - GM)

Voi m¢t 56 ban. ki thuat nay COil kha xa la nhirng no cling rat quell thuoc voi nhieu ban doc khac. Tuy v~y hilu nhu cac ba: toan sit dung phuoug phap nay dhl

76

Chuang 1 BAt aAllg th d"c co sO

khong don gian vOi ha.t kl ai Trtr lai hai toan 1.8,1. Cb(lllf', ta foe h giai cho dll."rng ruinh 0 do. va July xz.t thorn iuot vi JU kha« twmg hJ

Vi du 1.8.3. ChUng minh rJ:ng n€u xy + yz + ;>;.T "" 5 thi 3xi + 3y2 + Z2 ;::: 10.

LOI G14L Lui giai Jon giii.n va ng~n gon eho VI du tren la 4;C2 + :;:2 2': 4xz

4y2 + z2 ~ 4yz

2x2 + zl > 4xy

Do (to 2(3x2 + 3y'l + :;:2) ~ 4(xy + yz + z.r]. til. l:("J dh~1l p!u'd dl\·(ng rninh. Dling thuc xfiy ru k hi ;r. = y:;;;;; I, Z = 2. 0

Bay gio ta sf tim li do ella vice tach 10 = 2 + 8 a hai loan lIla d§.u va 6 = 4 + 2 " vi du tr~n. Ta e~n xem xet bai roan dang t6ng quilt

Vi du 1.8.4. TIm giri trj nho nhfi.t eua

k(x2 + l) + :;:2

Trong do cac ~O thuc z , y, z !hoG. man. xy + yz + zx = 1 va k l{L m(jt hdng 56 dltanfj.

LtlI GL4L Ta hay tach k == 1+ (k - I) (vci 0 :::; I ::; k) vii ap dung bat u~ng thik AM - GM thoo phuong phap tuong tu nlnr trcn

Ix? + !y2 ::::: 21xll

{k - l)x2 + 1/2.<:2 ~ J2(k -l)xz (k _l)y'2 + 11hz "2:' J2(k - I)yz

Do J6 k(x2 + )/) + z'.1 ~ 21x)f + V2(k':" l)(;r.z + l-p).

Trong t.rlton).!; llcjp nay, t.a khi'JIl!l; phAi din bi'i.n~ I!i{:n kiC~lI rlAng t lurc ma 1 a phfi.i (·i\n bAng di~l1 kl(:n gia thiet, tire la tim 1 so dirong I sao cho 2l 0:: J2(k - I). Khi d{,

k(x'l + y2) + z2 ~ 2l{xy + yz + zx) = 21. 56 I dime chon a trcn thoa man phuong trinh

2{2 = k - l <=*" 212 + I ;;;;; k

~l= ~1+v'l+8"k. 4

Va to- suy ra kct qua

k( 2 2) 2 -1 + Jf+8k 0

x +/1 +Z ~ 2 .

1-10, BM tap ap d~ng

101

Bai tom 1.60. Cho cae s& dJJong a, b, c thod. man at + be + ca = 1 vii z, y, z La do da'i 3 (;{inh cuu m(jt tum yiii(; TIm gill tl1 nh6 nhdt L1i~ bitt> ~hue 8au day

Bai toan 1.61. Ch*n9 minh. riln9 vai m9i a: b, c. khUng am !hi

a J 1),2 + 2bc + bJb2+2co, + cJA 2ab ~ V3(ab + be! r.a).

Bai toan 1.62. ChUng minh v6'i m{Ji 56 thvc khOng am a, b,;:: to: ll.lan co

a2 b2 e2

=-----:------;:2 + " 2 + " ~ > 2.

62 - be. + c c~ - ca + a a < - ab + b- -

Bai toan 1.63 (India 2004). Cdc so th1,lc dUdng Xl, x~, .. " J.., thod. mdn I~; _ xi+d <1 vax; ~ l'v'i;;;;;; Ln, Xn+l "" Xl. ChUng tninl: btit dii.ng Ihuc

Bb,i toan 1.64. Ch» cae s6 th-l,ic dUdng a, 0, C E [-1, 11 thoa man ab + oc + ca =;; 1. Chlm.g minh rdng

a2 + b2 b2 + c2 c? + (22 9

-;-:--~;-:---;-:;:;;- +. + > -

(1 - a2}(1 - 6i) (l - b2)(1 - c2) (1 - (2)(1 - a2) - 2'

BOi toan 1.65 (Poland 1991). Gici sa cdc .~o th1,tc z, y, z thou man x2 +y2 +z2 = 2. Chltn,9 minh:

x + y + Z "$ 2 + :cyz,

Bili toan 1.66. Ch"iJng minh rang niu a, b, c ;:: 0 vii a + 0 + C = 2 thi

a2b2 + b2c2+ C2(1,2 + abc :$ 2.

8M toan 1.61 (Poland 1997). Cac 86 thttc a, b, c thoa man a, b, c ~ -3/4 va a + b + c = 1. ChUng minh rlil1g

abc 9

--+--+~<- 1+a2 l+b2 1+c2-10·

Bill toan 1.68. Chitn,,! mink tnn mpi a, b, e khong tim thi

1 I 1 a+b .b+c e+a

-+ -+- > --+--+~-.

abc - c2 + ab ,,2 + be b2 + 4c

Bal toan 1.69 (Math. Changelles). Chim_9 minh 1JUi mQi a, b; c ~ a 3 ~ + la, - bj + Ib ~ cl + Ie ~ a) ;:: a + b + c.

102

Bai toan 1.70 (IMO 1986). Cbo X" X2 ~ 0 va Yh Y2. ZI, Z2 E R sao cho X1Yl :::! :or, X2Y2 2:: z~. Chtcny iui1l.h but dang thUc

1 1 8

_-..-------,c+:> .

$1.YI - zr ;L'2Y"l - z~ - {.TI + X2){YI + m) - (Zl + z:d2

Bai toan 1.71. Ch11ng minh vrn m{J1 11-: b, c kMng dm 1M

abc abc 5

~+--+--+ :>-

b + c a + C a + b 2{a3 + b~ + c3) - 3'

Bal tosn 1.72 (Balkan 2005 Pro. 3). Chung minh bJ! dang lh-tJC suu vai mQl

a2 b2 c2 4{a - b)2

~+_+_>a+b+e+ .

be a- o.+b+c

Bit.i toan 1.73. ChUng minh rang vd'i m(Ji a, b, c :::>: 0 till:

( 4a) ( 40) ( 4r)

1 + _ _ 1 + - - 1 + - - "2: 25.

b+c u+c u+b

BM toan 1.74 (A, IMO Shortlist). ChUng minh lJoi mQi a, b, c ~ 0 th1

. 1 113

a3(o + cHe + a) + b3(c + aHa + b) + c3(a + b){b + e) '2: "4'

Bai tORn 1.15. ChUng minh V01 11, b, e;:?_ 0 va a + b + c = 2 thl,

ub be Cfl 1

--+--+~<~,

c+2 a+2 b+2 - 2

fcIUCNG OA.N- Quy d6n!l; rH?1.1l t;itJ va bi~n o.6i bAt diing t,ll\Ir: t,r,) thhnh

2(ab + be + ea)2 + 14(ab + be + cc) ::; 16 + 21abc {=} 2(ab + be + C(l. - l){ub -+- be + en + 8) :$ 21abc.

abc 2: ((I. + b _ c){b + c - a)(c + a - b) ;;;.. 8(ab + be + ca - 1) :5 9abe.

Thay b~t d~ng th(tc tr€:n vao (*) va·c~6. y rAng ab + be + ea ~ ~ ta c6 apcm. 0 Blti tom 1.76 (Math Camp. 2000). Gia. su a, 0, C Iii d¢ ddi 3 C(tnh cua m~t tam giac nh(ln- Chitng ffl1nh Mt ddng tMe sat.i

(a + b + c) (a2 + b"- + (2)(a3 -t b3 + c3) ?: 4{a£ + bt'i + e6).

1.11. MOt ~O bai (oall d~l!g eM .Y

103

BAi toim 1.77. Cbo a, b, c ld cae s6 thuc d1tdn9 thoa man abc = L CMm.9 minh

rdng

, < a(3a+ 1) b(3b+ 1) c(3c+ 1)

3 - (a + 1)2 + (b + l)~ + (c + 1)2 ~ a + bf. c.

Bai roan 1.78. Ch(m,g mmh bUt diing thuc sat! lJo-i mm a, b, c > 0

Bai toan 1.79. Chu nk srl th1jc khOng am a, b,e, d thaG. man a~ + b2 + c2 + d'l ;;; 4.

ChUng mink nInq .

a" + bJ + c3 + dJ + abc + bed + cda + dab :So 8.

Ba] toan 1.80, ChUng minh nlng vih moi s6 th1,lc khdnq am a, b. c ta co

b+c c+a a e b 6

_._+ + > .

2a2 + be 2112 + Ca 2c. + ab - a + b + c

Ba! toan 1.81 (IMO 2005 Pro.2). Cho n E N' va Xl. X2 •... , Xn !d cdc 86 lh!,!c. L Ch((n.g minh rtlng

1.11 MQt 56 bai toan dang chti y

Truoc khi xern th~p chuang IL u:I.C han hii.y thu sue voi dt: hai toan eho sail day.

Clniug dtrt)t: chon ra h! cac uai t.ofu] U l,:(u; clnnmg SHU va mi('l\ ta. klui JAy ttll IlOi dung dm cac btl.t dAng thl1c: a d6. V~n Ill. cac bai toan c6 hmh thuc rAt dan gian: nhirng khong co bai nao don gi';n nhu ve ngoai ella no, mat bai toan deu dac trirng cho mot phinrng phap rieng: C6 mOt s6 bai con diroc clnrng rnmh thea r§.t nhi~u didl kha~ nhau, Ch1k dli\n clll"mp; sr. rAt co y nghia cho (:3.r: ban nam dw:]c noi dHng cua cu6n each nay. 1\'3.0, bay giO moi cac ban hay th{r suc ll

Problem 1 (Romania TST IMO 1998). Vo:ti car s6 th1,lC duang al ,a2 •. -, I},;, cOo

uct, Mng 1, hall chung minh .

1 1 1

----+ + ... + < l.

n - 1 + a I n - 1 + '<"2 n - l + an -

You might also like